Download as pdf or txt
Download as pdf or txt
You are on page 1of 433

3-2x

1_ z

?p-' f.^;

: ULKO
PtiysicsL"
O Licence ategory B1 & B2 7, Issue number: 2 / Volume Number: 1
r/

>

5^
V-.-'
\ •

2.1: Matter 2.2.2: Kinetics 2.3: Thermodynamics


2.2: Mechanics 2.2.3: Dynamics 2.4: Optics
2.2.1: Statics 2.2.4: Fluid Dynamics 2.5: Wave Motion & Sound
- §&>* •/T, " --
W,
Module 2.1 Matter
Copyright notice
© Copyright. All worldwide rights reserved. No part of this publication may be reproduced, stored in a
retrieval system or transmitted in any form by any other means whatsoever: i.e. photocopy, electronic,
mechanical recording or otherwise without the prior written permission of Total Training Support Ltd.

Knowledge levels — Category A, B1, B2, B3 and C Aircraft Maintenance Licence


Basic knowledge for categories A, B1, B2 and B3 are indicated by the allocation of knowledge levels
indicators (1, 2 or 3) against each applicable subject. Category C applicants must meet either the category
B1 or the category B2 basic knowledge levels.
The knowledge level indicators are defined as follows:

LEVEL 1
• A familiarization with the principal elements of the subject.

Objectives:
• The applicant should be familiar with the basic elements of the subject.
• The applicant should be able to give a simple description of the whole subject, using common
words and examples.
• The applicant should be able to use typical terms.

LEVEL 2
• A general knowledge of the theoretical and practical aspects of the subject.
• An ability to apply that knowledge.
«
Objectives:
• The applicant should be able to understand the theoretical fundamentals of the subject.
• The applicant should be able to give a general description of the subject using, as appropriate,
typical examples.
• The applicant should be able to use mathematical formulae in conjunction with physical laws
describing the subject.
• The applicant should be able to read and understand sketches, drawings and schematics
describing the subject.
• The applicant should be able to apply his knowledge in a practical manner using detailed
procedures.

t pvPI
V 3
■■■t Imw Kwh lw»

• A detailed knowledge of the theoretical and practical aspects of the subject.


• A capacity to combine and apply the separate elements of knowledge in a logical and
comprehensive manner.

Objectives:
• The applicant should know the theory of the subject and interrelationships with other subjects.
• The applicant should be able to give a detailed description of the subject using theoretical
fundamentals and specific examples.
• The applicant should understand and be able to use mathematical formulae related to the subject.
• The applicant should be able to read, understand and prepare sketches, simple drawings and
schematics describing the subject.
• The applicant should be able to apply his knowledge in a practical manner using manufacturer’s
instructions.
» The applicant should be able to interpret results from various sources and measurements and
apply corrective action where appropriate.

1-2 Total Training Support Ltd


Issue 2 - September 2016
Module 2.1 Matter ©Copyright 2016
Module 2.1 Matter
Certification statement
These Study Notes comply with the syllabus of EASA Regulation (EU) No. 1321/2014
Annex Ell (Part-66) Appendix I, and the associated Knowledge Levels as specified below:

Knowledge
Part-66 Levels
Objective
Reference
A B1 B2 B3
Matter 2.1 1 2 2 2
Nature of matter: the
chemical elements,
structure of atoms,
molecules
Chemical compounds;
States: solid, liquid and
gaseous
Changes between states

Total Training Support Ltd 1-3


©Copyright 2016 Module 2.1 Matter Issue 2 - September 2016
Moduie 2.1 Matter

Intentionally Blank

1-4 Total Training Support Ltd


Issue 2 - September 2016
Module 2.1 Matter ©Copyright 2016
Module 2.1 Matter
Table of Content
Nature of matter______________________________________________________ 9
General___________________________________________________________ 9
Dalton’s atomic theory_______________________________________________ 9
Structure of atoms__________________________________________________ 11
Protons__________________________________________________________ 11
Neutrons_________________________________________________________ 11
Electrons_________________________________________________________ 11
Periodic table of the elements________________________________________ 13
Elements, mixtures, compounds and molecules_________________________ 15
Elements_________________________________________________________ 15
M ixtu res__________________________________________________________ 15
Compounds______________________________________________________ 15
Molecules_________________________________________________________ 16
Chemical formulae_________________________________________________ 16
Chemical reactions ________________________________________________ 20
Atomic number and mass number______________________________________ 23
Atomic number_____________________________ ._______________________ 23
Mass number_____________________________________________________ 23
Atomic mass unit (AMU)____________________________________________ 23
Isotopes__________________________________________________________ 26
Electron arrangements______________________________________________ 29
Shells____________________________________________________________ 29
Subshells____________________________________ ;____________________ 29
Ionization_________________________________________________________ 30
Examples of electron arrangements ___________________________________ 30
Valency__________________________________________________________ 32
Conductors, semiconductors and non-conductors (insulators)_______________ 34
Summary_________________________________________________________ 36
Chemical bonding__________________________________________________ 41
Adhesion and cohesion_____________________________________________ 41
Noble gases_______________________________________________________ 41
Covalent bonding__________________________________________________ 42
Allotropes________________________________________________________ 46
Ionic bonding______________________________________________________ 48
Metallic bonding (electron cloud)______________________________________ 54
States; solid, liquid and gaseous______________________________________ 57
Solids____________________________________________________________ 57
Liquids__________________________________________________________ 57
Gases ___________________________________________________________ 57
Plasma___________________________________________________________ 57
Changes between states____________________________________________ 58
Brownian motion__________________________________________________ 60
Thermal expansion of solids, liquids and gases___________________________ 60
Kinetic particle theory_______________________________________________ 62

Total Training Support Ltd 1-5


©Copyright 2016 Module 2.1 Matter Issue 2 - September 2016
Module 2.1 Matter
Acids and alkalis____________________________________________________ 63
Acids____________________________________________________________ 63
Alkalis ___________________________________________________________ 63
The pH scale_____________________________________________________ 64

Issue 2 ~ September 2013 1-6 Total Training Support Ltd


Module 2.1 Matter ©Copyright 2016
Module 2.1 Matter

intentionally Blank

Total Training Support Ltd 1-7


©Copyright 2016 Module 2.1 Matter Issue 2 - September 2016
Module 2.1 Matter

Different types Single type


of atoms of atoms

• Atom: the smallest possible piece of an element


• Element: a substance in which all of the atoms are (almost) identical
• Molecule: a combination of two or more atoms connected in a fixed ratio that act as a
single unit.
• Compound: a molecule containing two or more elements.

1-8 Total Training Support Ltd


issue 2 - September 2016 © Copyright 2016
Module 2.1 Matter
Module 2.1 Matter
Nature of matter
General
Atoms are the smallest particles of matter whose properties we study in chemistry.
However, from experiments done in the late 19th and early 20th century it was deduced
that atoms were made up of three fundamental sub-atomic particles (see table opposite).

The early Greek philosophers proposed that all matter is made up of incredibly small but
discrete units (like the bricks in our wall example). Democritus (460 - 370 BC) was the
first to call these units ‘atomos’. From this phrase came the term atom that we use today.
Atomos is a Greek phrase which means ‘not cut’ or ‘that which is indivisible’.

Atoms can neither be created nor destroyed during chemical reactions.

All atoms are, crudely speaking, the same size and can be thought to consist of two main
parts. The outer part is composed of one or more orbits of electrons.

These orbits make up most of the volume of the atom yet contributes practically nothing
to its substance. The other part, located at the centre, is extremely small compared to the
atom as a whole, yet essentially all of the real substance of the atom can be attributed to
this small speck. We call this speck the nucleus.

Further investigation revealed that the nucleus is actually composed of two kinds of
particles of a roughly equal size and substance packed closely together. These nuclear
particles are the proton and neutron. When we refer to the amount of material or
substance in an object, we are really talking about the number of protons and neutrons in
that object.

Also, what we perceive as the mass of an object is related directly to the number of
protons and neutrons contained it.

The simplest atom is hydrogen which has a single proton for a nucleus. An atom of lead,
on the other hand, has 82 protons and 125 neutrons in its nucleus and so has 207 (125 +
82) times as much material or substance as an atom of hydrogen.

The size of an atom bears no simple relation to the number of particles in its nucleus. A
sodium atom, for example, with 11 protons and 12 neutrons is approximately the same
size as an atom of mercury with 80 protons and 121 neutrons.

In general, we can say that the size of an atom is determined by its electron orbits, its
substance is determined by the total number of protons and neutrons in its nucleus.

Dalton’s atomic theory


Around 1805, John Dalton proposed a theory to explain the nature of matter that is
remarkably similar to modern atomic theory. While this theory is stated in a variety of
ways, the key points of this theory are:

• All matter is composed of indivisible particles called atoms.


« All atoms of an element are identical, while atoms of different elements are different.
® Molecules are a combination of any two or more atoms. Compounds are a
combination of atoms from two or more different elements.
• Chemical reactions involve the rearrangement of atoms. The atoms themselves do
not change.

Total Training Support Ltd 1-9


© Copyright 2016 Module 2.1 Matter Issue 2 - September 2016
Module 2.1 Matter
The law of conservation of mass is a direct consequence of this last postulate. The law
of constant composition is a result of the fact that molecules generally contain fixed
ratios of atoms.

Neutron 1 0 (zero) In the nucleus


Proton T/ +1 (positive)
Electron 1/1850 -1 (negative) Arranged in energy levels or
shells around the nucleus

The sub-atomic components of atoms

1-10 Total Training Support Ltd


Issue 2 - September 2016
Module 2.1 Matter © Copyright 2016
Module 2.1 Matter
Structure of atoms
Atoms cannot be seen with the naked eye, only recently has this been possible, with very
powerful microscopes. However, scientists have a good idea of how they behave in
different situations. Based on these ideas, they have developed a model of what the
atom looks like, to help us understand atoms better.

The modern model of the atom teaches us that all atoms are made up of sub-atomic
particles. Sub-atomic means ‘smaller than the atom’.

Protons
The protons are deep inside the atom, in a zone called the nucleus. The protons are said
to be positively charged.

® When two protons get near each other, they push each other away.
* When an electron gets near a proton, they attract each other.
® Two electrons will also push each other away.

Scientists use the word ‘charge’ to represent the property these particles have. We
observe that:

® like charges repel (meaning the same charges push each other away)
• opposite charges attract

Neutrons
Neutrons are particles that are neither positively nor negatively charged. They are
neutral. The neutrons together with protons form the tightly packed nucleus at the centre
of the atom.

Electrons
Electrons are the smallest of the three sub-atomic particles. Electrons are nearly 2,000
times smaller than protons and neutrons. The electrons move in a zone around the
atomic nucleus at extremely high speeds, forming an electron cloud that is much larger
than the nucleus. Have a look again at the diagram which shows a model of the atom to
see this.

What Is An Atom?
https://youtu.be/o-3l 1 JGW-Ck

What Is An Atom - Part 1 I Chemistry for All I FuseSchool


https://youtu.be/7VZApOzxYC4

What Is An Atom - Part 2 - Isotopes | Chemistry for All I FuseSchool


https://youtu.be/51zQgewbjEs

Totai Training Support Ltd 1-11


© Copyright 2016 Module 2.1 Matter Issue 2 - September 2016
5 ®
_J
Modu le 2.1 Matter
■c CM
o *-
CL .£3
Cl O)

CO £
□ Alkali metals
.£ O
II Rare earth vin f ©

SI Non metals
M Alkaline earth
Other metals 1VB
HIB VB VIB 5 1IB ■■iSKB ■
91 Halogens ■ -pocn'
□ Transition metals 6 7 ' 5 ■ ■ 1Q \- .

c N 0 Ne
P $107 .14 SiiifiYl <liG»65!;33.
|.4hli-js ... ::i;S!K;wi- ■. ■ sif^.r: ;
15 16 OSS S8 -. ■.:

asa
IlIA IVA VA VIA VHA f -vniA- IB nis
w® P Ar
" - Xil !MSf ■
a! 'Ci :v 056
scandium-” ■: i:?anfum . < -vai®djum - . cbrotMiijs manganese / cobalt .- . tisetei ' ••••. copper ; , Z|hC
21 22 ■ 23 :■ ■■ ■ 24 ■-■■■ 25 ..:■ 26 " 27 ■ 28 ;' 29 30 " ;<31 ■■■■' it/; 34

Sc Ti
44.35591 ". 47.867 '
V ;Cr Mn Fe Co Ni Cu Zn Ga
' ■ 50,9415 51.9S61 54.33894!) .. 55.345 • 58.9332 /■ 55.6934 63,54$. . E5J-J '■ ffiWEi;"”.
Se
: : ■
yttrium. •••? zirconium •••.: niobium : molybdenum JadinetitBn •: ruthenium •:-. ihoifem pa&dium - stiver - - ': cadm -J?:- . /RttUili. •.iHri-m sr-j’nri .
39- . 40 ■' 41 . ■ 42. 43 ■; .44 45 46 47 48 7.?49-.-. .■■■■■■■so:. 52 ■' ■ 54-.' ■
$
Y Rh Pd Ag Cd Afc ■Sh
Zr Nb Mo Tc Ru rvii Te s®a. ;Xe; flj
5>
......... ij’iSs'SiiiSV:.'.-
88,30585 . 91.224 --92.MM338' 95.94 - (98S - 101.07 : .102.9055'' -< 106.42 - 107.8682 112.41! " .113.74 ■ CM T-
wsr.' ..." V CM
hafnium -tantalum'' ■■ '-liirigstan -.- 3 (bantam ■- • Gsmtgnv - ■:- iridium/ -. pSalirtem gold - ' merpjy ■■■■■thaiftrr. ■ ■ ./■ isad”-
72 : 73 . 74 .75 ;- 76 77 73 3--- 79 80 :.V ' S2> :: ■. i?3..' ■ ■ ..................... ■ ■ ■8S: '■ T“ .32
W;

. '
Hf Ta w Re Os
178.49 is?io;:>3
Ir Pt Au Hg TI Ph : " 2iV:>.: ■
3
*O
O
180.9473 - 193.84 -li?.£3.-2O7. 192.247 1S5.07S -iO6.5S»55 2O3.;:->
-tfiaitkttSttiitSn iiUKHlpeiitei's - itnnifcesxiiim- s.annBsep&iini: .unrii.techum itiftnBsfsriygt/ --Uftunnilttitfl-. .ttti&ftjwnii *
204 105 106 107 108 ■' 109 110 111 112 :
I Wi

Db sg Bh Hs Mt Uun tfiiii Uub


v i*
/■i 12611 {282]-• ■.."■"1263);..-.: i2rai ■ (265J (266! 12691 : (272) 127?! .

CO
o
CM

0
-C
E
0
a.
0
CO

The periodic table of elements CM


0
tft
tft
Module 2.1 Matter
Periodic table of the elements
The periodic table is a tabular arrangement of the chemical elements, ordered by their
atomic number (number of protons), electron configurations, and recurring chemical
properties. This ordering shows periodic trends, such as elements with similar behaviour
in the same column. It also shows four rectangular blocks with some approximately
similar chemical properties. In general, within one row (period) the elements are metals
on the left and non-metals on the right.

The rows of the table are called periods; the columns are called groups. Six groups
(columns) have names as well as numbers: for example, group 17 elements are the
halogens; and group 18, the noble gases. The periodic table can be used to derive
relationships between the properties of the elements, and predict the properties of new
elements yet to be discovered or synthesised. The periodic table provides a useful
framework for analysing chemical behaviour and is widely used in chemistry and other
sciences.

Dmitri Mendeleev published in 1869 the first widely recognised periodic table. He
developed his table to illustrate periodic trends in the properties of the then-known
elements. Mendeleev also predicted some properties of then-unknown elements that
would be expected to fill gaps in this table. Most of his predictions were proved correct
when the elements in question were subsequently discovered. Mendeleev’s periodic
table has since been expanded and refined with the discovery or synthesis of further new
elements and the development of new theoretical models to explain chemical behaviour.

All elements from atomic numbers 1 (hydrogen) to 118 (ununoctium) have been
discovered or synthesised, with the most recent additions (elements 113, 115, 117, and
118) being confirmed in 2015. The first 94 elements exist naturally, although some are
found only in trace amounts and were synthesised in laboratories before being found in
nature. Elements with atomic numbers from 95 to 118 have only been synthesised in
laboratories or nuclear reactors. Synthesis of elements having higher atomic numbers is
being pursued. Numerous synthetic radionuclides of naturally occurring elements have
also been produced in laboratories.

Hydrogen, 1, H, does not readily fit into any group.

A group is a vertical column of like elements e.g. group IA, the alkali metals (Li, Na, K
etc.), group VIIB, the halogens (F, Cl, Br, I, etc.) and group VIII (or 0), the noble gases
(He, Ne, Ar, etc.). The group number equals the number of electrons in the outer shell
(e.g. chlorine’s electron arrangement is 2.8.7, the second element down, in group 7).

Each element has a fixed position on the periodic table.

The elements are arranged in order of increasing atomic number.

Introduction to the Periodic Table


https ://youtu. be/Zg6 KeXs DVwY

How Does The Periodic Table Work | Chemistry for All I FuseSchool
https://youtu.be/P6DMEgE8CK8

Total Training Support Ltd 1-13


© Copyright 2016 Moduie 2.1 Matter Issue 2 - September 2016
Module 2.1 Matter
There are at least 118 elements in our known universe. They can form compounds by
bonding in millions of different combinations ~ far too many to discuss here!

An element consists of atoms that are A compound consists of two or more


ail the same kind kinds of atoms in a fixed ratio.

Atoms of an element Molecules of an Molecules of a Mixture of


element compound elements and a
compound

1-14 Total Training Support Ltd


Issue 2 - September 2016
ModuSe 2.1 Matter ©Copyright 2016
Module 2.1 Matter
Elements, mixtures, compounds and molecules
There are only two classes of pure substances, namely elements and compounds. To
understand the difference between the two, look at the two diagrams opposite.

Elements
Pure substances, made up of atoms with the same number of protons. An element is a
material that is made up of atoms of only one kind.

Note that an element:

• consists of indivisible, minute particles called atoms.


• consists of only one kind of atom. All atoms of a given element are identical.
® cannot be broken down into a simpler type of matter by either physical or chemical
means, and
• can exist as either atoms (e.g. argon) or molecules (e.g. nitrogen).
• atoms of different elements have different masses.
• is identified by the number of protons in its nucleus - the number of neutrons may
change (isotopes) and/or the number of electrons may change (ions) but the element
will retain its identity.

Mixtures
Mixtures have the properties of the different substances that make it up. Mixtures melt at
a range of temperatures and are easy to separate.

Element, mixture or compound ? I Chemistry for All I The Fuse School


https://youtu.be/MaZ7lsc5ub8

Note that a mixture:

• consists of two or more different elements and/or compounds which are not
chemically bonded, they are physically intermingled,
• can be separated into its components by physical means,
® often retains many of the properties of its components.

Compounds
A compound is a material that is made up of two or more kinds of atoms that are
chemically bonded together. The properties of a compound are different from the atoms
that make it up.

Chemical synthesis is the name given to the purposeful execution of chemical reactions
to obtain a compound.

Compounds can only be broken down chemically (i.e. by a chemical reaction). The
splitting of a compound into its constituent elements is called chemical analysis,
decomposition or breakdown.

Note that a compound:

® consists of atoms of two or more different elements bound together chemically,


® can be broken down into a simpler type of matter (elements) by chemical means (but
not by physical means),
• has properties that are different from its component elements, and
® always contains the same ratio of its component atoms.

Total Training Support Ltd 1-15


©Copyright 2016 Module 2.1 Matter Issue 2 - September 2016
Module 2.1 Matter
Molecules
A molecule is two or more atoms that have chemically bonded with each other.

The atoms in a molecule can be of the same kind (in which case it would be a molecule
of an element), or they can be of different kinds (in which case it would be a molecule of
a compound).

Not all elements have molecules. The metals on the left-hand side and the middle part of
the periodic table are solids at room temperature and so they exist as tightly packed
arrays of atoms like the previous examples of silver and copper.

Many of the non-metals on the right-hand side of the periodic table are gases at room
temperature that exist as molecules made up of two atoms each. These are called
diatomic molecules. Oxygen (O2), nitrogen (N2), hydrogen (H2), chlorine (CI2) and some
other elements from the non-metals all form diatomic molecules.

‘Diatomic’ refers to a molecule made of two of the same atoms bonded together, as in
oxygen (O2). ‘Di’ means two. ‘Triatomic’ refers to a molecule made up of three of the
same atoms bonded together, like ozone (O3).

Since water is such an important compound for organisms living on Earth, we will use
that as our first example. Scientists know that a water molecule is made up of one
oxygen atom and two hydrogen atoms. If we could see them, all water molecules would
look a little bit like this diagram of a water molecule.

All water molecules are exactly the same. We say the atoms are bonded in a fixed ratio:
two hydrogen atoms for every one oxygen atom. The atoms in the molecule are held
together by a special force that we call a ‘chemical bond’.

Atoms, Molecules, Elements and Compounds


https://youtu.be/AfXxZwN LvPA

Chemical formulae
Each element has its own unique chemical symbol. We can combine these symbols into
a chemical formula, for example, for water. The chemical formula is another very
important concept in chemistry.

The chemical formula for water is H2O. It shows the ratio of hydrogen atoms (two) to
oxygen atoms (one) in one molecule of water.

A compound is a material that consists of atoms of two or more different elements. The
elements are not just physically mixed, but chemically bonded together at the atomic
level.

Water (H2O), carbon dioxide (CO2) and salt or sodium chloride (NaCI) are examples of
compounds, while oxygen gas (O2), hydrogen gas (H2) and nitrogen gas (N2) are
examples of elements.

Reading a Chemical Formula - Breaking Down Molecules


https://youtu.be/G9Ta7bg5Tlk

a*1
1-16 Total Training Support Ltd
Issue 2 - September 2016 © Copyright 2016
Module 2,1 Matter
I

Modu le 2.1 Matte r


© Copyright 2016
Total Training Support Ltd

A water molecule representation


(oxygen and hydrogen)

Chemical symbol H2O


Mo dule 2.1 Ma tte r
'

Oxygen (O2)

Water (H2O)
Issue 2
- September 2016

Carbon Dioxide (CO2) Ethanol Glycol ((CH2OH)2) Aspjrjn


Module 2.1 Matter
Example:

The compound with the formula H2O2 also consists of hydrogen atoms and oxygen
atoms. The formula tells us that one molecule of this substance is made up of two atoms
of hydrogen and two atoms of oxygen. Is H2O2 the same as water?

Do not confuse H2O2 with H2O. H2O2 is a compound called hydrogen peroxide. Hydrogen
peroxide is similar to water in that it is a clear, colourless liquid at room temperature
(25°C) though not as runny, but it is different in many ways. The following properties of
hydrogen peroxide may convince you that it is not the same as water:

• Hydrogen peroxide has a boiling point of 150°C and it is a very effective bleach for
clothes and hair.
• Concentrated hydrogen peroxide is so reactive that it is used as a component in
rocket fuel!
• Hydrogen peroxide is extremely corrosive.
• We can drink water, but hydrogen peroxide is very hazardous and harmful.

Even though they are made up of exactly the same elements, the two compounds are
very different and should never be confused with one another.

The purpose of the comparison of hydrogen peroxide and water above was to show you
that the atoms in a given compound are always combined in a fixed ratio. In all water
molecules in the universe, there will always be one O atom and two H atoms bonded
together.

r
1-18 Total Training Support Ltd
Issue 2 - September 2016 © Copyright 2016
Module 2.1 Matter
ilodute 2.1 Matter

Hydrogen peroxide
moiecule (H2O2)

Total Training Support Ltd 1-19


©Copyright 2016 Module 2.1 Matter Issue 2 - September 2016
Module 2.1 Matter
Chemical reactions
A chemical reaction is a process that leads to the transformation of one set of chemical
substances to another. Classically, chemical reactions encompass changes that only
involve the positions of electrons in the forming and breaking of chemical bonds between
atoms, with no change to the nuclei (no change to the elements present), and can often
be described by a chemical equation.

The substance (or substances) initially involved in a chemical reaction are called
reactants or reagents. Chemical reactions are usually characterised by a chemical
change, and they yield one or more products, which usually have properties different
from the reactants.

Chemical reactions happen at a characteristic reaction rate at a given temperature and


chemical concentration. Typically, reaction rates increase with increasing temperature
because there is more thermal energy available to reach the activation energy necessary
for breaking bonds between atoms.

Sometimes, an additional chemical is added to increase the rate of reaction. This is


called a catalyst. With a catalyst, reactions occur faster and require less activation
energy. Because catalysts are not consumed in the catalysed reaction, they can continue
to catalyse the reaction of further quantities of reactant. Often only tiny amounts are
required.

Inhibitors are sometimes referred to as “negative catalysts” since they decrease the
reaction rate.

What triggers a chemical reaction?


https://youtu.be/8m6RtOpqvtU

1-20 Total Training Support Ltd


Issue 2 - September 2016 ©Copyright 2016
Module 2.1 Matter
Intentionally Blank

Total Training Support Ltd 1-21


© Copyright 2016 Module 2.1 Matter Issue 2 - September 2016
Module 2.1 Matter
Atomic number

6
Chemical symbol11 „.............

. 13:
A

Mass number

Atomic number and mass number notation

Atomic number and mass number


notation on the periodic table

Number of Number of Mass


Neutrons Protons number

Six protons and


six electrons

Electron
iflll Proton

e Neutron
Carbon atom
1-22 Total Training Support Ltd
Issue 2 - September 2016
Moduie 2.1 Matter © Copyright 2016
Module 2.1 Matter
Atomic number and mass number
Atomic number
The atomic number (also known as the proton number) is the number of protons found
in the nucleus of an atom. It is traditionally represented by the symbol Z. The atomic
number uniquely identifies a chemical element. In an atom of neutral charge, atomic
number is equal to the number of electrons.

Mass number
The mass number (A), also called atomic mass number or nucleon number, is the
number of protons and neutrons (also defined as a less commonly known term,
nucleons) in an atomic nucleus. The mass number is unique for each isotope of an
element and is written either after the element name or as a superscript to the left of an
element’s symbol.

For example, carbon-12 (12C) has 6 protons and 6 neutrons. The full isotope symbol
would also have the atomic number (Z) as a subscript to the left of the element symbol
directly below the mass number, thus:

The difference between the mass number and the atomic number gives the number of
neutrons (N) in a given nucleus: N = A-Z.

For example, carbon-14 is created from nitrogen-14 with seven protons (p) and seven
neutrons via a cosmic ray interaction which transmutes 1 proton into 1 neutron. Thus, the
atomic number decreases by 1 (Z: 7->6) and the mass number remains the same
(A = 14), however, the number of neutrons increases by 1 (n: 7 >8).

Before: nitrogen-14 (7p, 7n)

After: carbon-14 (6p, 8n).

Atomic mass unit (AMU)


In physics, the term for what we have up to now referred to as the amount of substance
or matter is ‘mass’. A natural unit for mass is the mass of a proton or neutron. This unit
has the name atomic mass unit (AMU or amu). This unit is useful in those sciences
which deal with atomic and nuclear matter.

An atomic mass unit is defined as precisely 1/i2 the mass of an atom of carbon-12. The
carbon-12 (C-12) atom has six protons and six neutrons in its nucleus.

In imprecise terms, one AMU is the average of the proton rest mass and the neutron rest
mass. This is approximately 1.67377 x 10-27 kilogram (kg), or 1.67377 x 10-24 gram (g).
The mass of an atom in AMU is roughly equal to the sum of the number of protons and
neutrons in the nucleus.

In measuring the mass of objects which we encounter daily, this unit is much too small
and therefore very inconvenient. For example, the mass of a bowling ball expressed in
AMUs would be about 4,390,000,000,000,000,000,000,000,000.

Total Training Support Ltd 1-23


© Copyright 2016 Module 2.1 Matter Issue 2 - September 2016
SS1P Module 2.1 Matter
One kilogram equals 602,000,000,000,000,000,000,000,000 AMU. Since one amu is the
mass of a proton or neutron we know immediately that a kilogram of anything has this
combined number of protons and neutrons contained in it.

Atomic Mass: Introduction


https://youtu.be/7fYpEnxhKQk

What Is An Atom?
https://youtu.be/o~3l 1 JGW~Ck

What’s the Difference between Mass Number and Atomic Mass?


https://youtu.be/m1 SDWkkGeO

1-24 Total Training Support Ltd


Issue 2 - September 2016 © Copyright 2016
Module 2.1 Matter
Module 2.1 Matter

Intentionally Blank

Total Training Support Ltd 1-25


©Copyright 2016 Module 2.1 Matter Issue 2 - September 2016
Module 2.1 Matter
Isotopes
Isotopes are atoms of the same element with different numbers of neutrons. This gives
each isotope of the element a different mass or nucleon number but being the same
element, they have the same atomic or proton number. There are small physical
differences between the isotopes, e.g. the heavier isotope has a greater density and
boiling point. However, because they have the same number of protons they have the
same electronic structure and are identical chemically. Examples are illustrated below.
Do not assume the word isotope means it is radioactive, this depends on the stability of
the nucleus i.e. unstable atoms might be referred to as radioisotopes.

11H
1 ■, and
2H* are 31 H
* Mhe three isotopes of hydrogen. They are called hydrogen,
deuterium, and tritium respectively. How do we distinguish between them? They each
have one single proton (Z=1) but differ in the number of their neutrons. Hydrogen has no
neutron, deuterium has one, and tritium has two neutrons. The isotopes of hydrogen
have, respectively, mass numbers of one, two, and three. Hydrogen-1 is the most
common, there is a trace of hydrogen-2 naturally but hydrogen-3 is very unstable and is
used in atomic fusion weapons.

2® lv anq 2S iv are |W0 isotopes of helium with mass numbers of 3 and 4, with
1 and 2 neutrons respectively but both have 2 protons. Helium-3 is formed in the Sun by
the initial nuclear fusion process. Helium-4 is also formed in the Sun and as a product of
the radioactive alpha decay of an unstable nucleus. An alpha particle is a helium nucleus,
it picks up two electrons and becomes the atoms of the gas helium.

and 11 Q are the two isotopes of sodium with mass numbers of 23 and 24,
with 12 and 13 neutrons respectively but both have 11 protons. Sodium-23 is quite stable
e.g. in common salt (NaCI, sodium chloride) but sodium-24 is a radioisotope and is a
gamma emitter used in medicine as a radioactive tracer, e.g, to examine organs and the
blood system.

There are three isotopes of carbon found in nature - carbon-12, carbon-13, and carbon-
14. All three have six protons, but their neutron numbers - 6, 7 and 8, respectively - all
differ. This means that all three isotopes have different atomic masses (carbon-14 being
the heaviest), but share the same atomic number (Z=6).

Chemically, all three are indistinguishable, because the number of electrons in each of
these three isotopes is the same.

So different isotopes of the same element are identical, chemically speaking. But some
isotopes have the ability to circumvent this rule by transforming into another element
entirely.

What are Isotopes?


https://youtu.be/EboWeWmh5Pg

1-26 Total Training Support Ltd


Issue 2 - September 2016
Module 2.1 Matter © Copyright 2016
Module 2.1 Matter
isotopes of carbon

Total Training Support Ltd 1-27


© Copyright 2016 Module 2.1 Matter Issue 2 - September 2016
Module 2.1 Matter

The atomic structure of helium and neon

Electron shell (orbit) designation

1-28 Total Training Support Ltd


Issue 2 ~ September 2016
Module 2.1 Matter © Copyright 2016
ModuSe 2.1 Matter
Electron arrangements
Shells
The electrons are arranged in energy levels or shells around the nucleus and with
increasing distance from the nucleus. The shells are lettered from the innermost shell
outwards from K to G. There are rules about the maximum number of electrons allowed
in each shell.

® The 1st shell (K) has a maximum of 2 electrons


• The 2nd shell (L) has a maximum of 8 electrons
• The 3rd shell (M) has a maximum of 18 electrons
• The 4th shell (N) has a maximum of 32 electrons

Our knowledge about the structure of atoms depends on the mathematical formulations
predicted by Neils Bohr. He suggested that electrons are distributed in orbits and the
number of electrons held in the orbit depends on the number of the orbit. The orbits are
counted outwards from the nucleus. Higher the orbit number, the further are the electrons
in that orbit from the nucleus. If the orbit number is “n”, then the maximum electrons held
in the orbit is given as 2n2. The first orbit has n=1, and will hold a maximum of two
electrons, the second orbit has n=2 and is capable of holding a total of eight electrons;
similarly, the third orbit will be able to contain 18 electrons and so on. This is known as
Pauli’s exclusion principle.

Electrons within an atom have definite energies. The electrons closest to the nucleus
(n=1) are most tightly bound; the reason is because of stronger electrostatic attraction
with the nucleus. Electrons in the highest orbit are least tightly bound. Electrons in the
same orbit have same energies. The electron orbits are also called as electron energy
levels or shells. Electronic shells are known as K shell, L shell, M shell, N shell
corresponding to orbit number n=1, 2, 3 and 4 respectively. Higher number orbits are
assigned shell names in alphabetical order after N.

Subshells
Shells also have subshells. The subshells define the energy levels.

‘s, p, d, f are the names given to the subshells that hold the electrons in the shells of
atoms. These orbitals have different shapes (e.g. electron density distributions) and
energies (e.g. 1s is lower energy than 2s which is lower energy than 3s; 2s is lower
energy than 2p).

Physicists and chemists use a standard notation to indicate the electron configurations of
atoms. The notation consists of a sequence of atomic orbital labels (e.g. for phosphorus
the sequence 1s, 2s, 2p, 3s, 3p) with the number of electrons assigned to each orbital (or
set of orbitals sharing the same label) placed as a superscript.

For example, hydrogen has one electron in the s-orbital of the first shell, so its
configuration is written 1s1. Lithium has two electrons in the 1s-subshell and one in the
(higher-energy) 2s-subshell, so its configuration is written 1s2 2s1 (pronounced “one-s-
two, two-s-one”). Phosphorus (atomic number 15) is as follows: 1s2 2s2 2p6 3s2 3p3.

Total Training Support Ltd 1-29


© Copyright 2016 Module 2.1 Matter Issue 2 - September 2016
Module 2.1 Matter
For atoms with many electrons, this notation can become lengthy and so an abbreviated
notation is used, since all but the last few subshells are identical to those of one or
another of the noble gases. Phosphorus, for instance, differs from neon (1s2 2s2 2p6) only
by the presence of a third shell. Thus, the electron configuration of neon is pulled out,
and phosphorus is written as follows: [Ne] 3s2 3p3. This convention is useful as it is the
electrons in the outermost shell that most determine the chemistry of the element.

Drawing electron configuration diagrams I Chemistry for AH I The Fuse School


https://youtu.be/hSkJzE2Vzw

Ionization
When the atom loses electrons or gains electrons in this process of electron exchange, it
is said to be ionised. For ionisation to take place, there must be a transfer of energy
which results in a change in the internal energy of the atom. An atom having more than
its normal number of electrons acquires a negative charge and is called a negative ion
(or ‘anion’). The atom that gives up some of its normal electrons is left with less negative
charges than positive charges and is called a positive ion (or ‘cation’). Thus, ionisation is
the process by which an atom loses or gains electrons.

• Cation - A cation is a positively charged ion. Metals typically form cations.


• Anion ~ An anion is a negatively charged ion. Non-metals typically form anions.

Examples of electron arrangements


The diagrams opposite show some examples of electron arrangements in the shells of
the respective atoms.

Subshells are not shown.

1-30 Total Training Support Ltd


Issue 2 - September 2016 © Copyright 2016
Module 2.1 Matter
Module 2.1 Matter

On period 1
S = electron
helium (2) 2
(atomic no.)
or
(electrons)

Electron arrangement of hydrogen and helium

On period 2
carbon (8) 2.4

Electron arrangement of lithium, carbon and neon

On period 3

sodium (11) 2.8.7 chlorine (17) 2.8.7 argon (18) 2.8.8

Electron arrangement of sodium, chlorine and argon

On period 4

Electron arrangement of potassium and calcium

Total Training Support Ltd 1-31


©Copyright 2016 Module 2.1 Matter issue 2 - September 2016
Module 2.1 Matter
Valency
Hydrogen is the simplest element. It has one electron. Its outer shell only holds two
electrons. Let us use hydrogen as a standard to see how other atoms combine with it.
The table opposite lists the simplest compound of selected elements with hydrogen.

Valency can be simply defined as the number of hydrogen atoms that an element can
combine with. In the table, helium, neon and argon have a valency of 0. They do not
normally form compounds.

Lithium, sodium and potassium have a valency of 1 because they combine with one
hydrogen atom. Beryllium, magnesium and calcium all have a valency of 2: they combine
with two hydrogen atoms. Note that the valences of all these atoms are equal to the
number of outer electrons that these elements have.

Boron and aluminium combine with three hydrogen atoms - their valences are 3 - and
they have three outer electrons.

Carbon and silicon combine with four hydrogen atoms. The valency of these elements is
4. It will come as no surprise that they both have four outer electrons. Any element with
four electrons in its outer shell is known as a semiconductor

What about nitrogen and phosphorus? They have five outer electrons. But they normally
only combine with three hydrogen atoms. Their valences are 3. Note that three is five
less than eight. These atoms are three electrons short of a full shell.

Please note that both nitrogen and phosphorus can also have a valency of 5. Some
atoms are capable of having more than one valency. That will confuse the issue so we
will talk of normal valency.

Now to oxygen and sulphur. Both have six outer electrons. Six is two short of a full shell.
Their normal valences are 2 and they combine with two atoms of hydrogen.

Finally, fluorine and chlorine - seven outer electrons. This is one short of a full shell.
They both combine with a single hydrogen atom and their normal valences are 1.

As a side note, chlorine can also have valences of 3, 5 and 7. The reasons are well
beyond the scope of these notes.

The rules above can be summarised as follows:

The normal valency of an atom is equal to the number of outer electrons if that
number is four or less. Otherwise, the valency is equal to eight minus the number
of outer electrons.

The atoms with full electron shells (helium, neon, argon) are chemically inert forming few
compounds. The atoms don’t even interact with each other very much. These elements
are gases with very low boiling points.

The atoms with a single outer electron or a single missing electron are all highly reactive.
Sodium is more reactive than magnesium. Chlorine is more reactive than oxygen.
Generally speaking, the closer an atom is to having a full electron shell, the more reactive
it is. Atoms with one outer electron are more reactive than those with two outer electrons,
etc. Atoms that are one electron short of a full shell are more reactive than those that are
two short.

1-32 Total Training Support Ltd


Issue 2 - September 2016 ©Copyright 2016
Module 2.1 Matter
Module 2.1 Matter jjjgf
Atoms with only a few electrons in its outer shell are good electrical conductors. Atoms
with eight, or close to eight electrons in their outer shells are poor conductors (or good
insulators). This is why atoms with four electrons in its outer shell are semiconductors.

When a semiconductor (such as silicon or germanium) atom bonds with another similar
atom, it does so covalently. Each atom shares one electron with four neighbour atoms.
Thus all its electrons are used up in what becomes a solid lattice of semiconductor
atoms. The solid material has, therefore, no free electrons (and no holes for electrons to
fit into).

—7T

Helium He Full None

B|^Hum Be BeH2

CartJon C A

Oxygen H2O
ISIS j HF
None
■IF......
Magnesium Mg 2 MgH2
...
Silicon
toon Si 4 SiH^
■as
Ci ft LKC
*---•Sulphur
: V s ' **• .it" r.?
Cl 7 HCI
Argon Ar Full None

Calcium Ca 2 CaH2

Electrons in outer shells of some common elements

The following names are given to ions of the specific


number of electron bindings (valence):

• 1 electron binding - monovalent


® 2 electron binding - divalent
• 3 electron binding - trivalent
® 4 electron binding - tetravalent
« 5 electron binding - pentavalent
® 6 electron binding - hexavalent

Total Training Support Ltd 1-33


©Copyright 2016 Module 2.1 Matter Issue 2 - September 2016
Module 2.1 Matter
Conductors, semiconductors and non-conductors (insulators)

Conductors
In a conductor, electric current can flow freely, in an insulator it cannot. Metals such as
copper typify conductors, while most non-metallic solids are said to be good insulators,
having extremely high resistance to the flow of electrical charge through them.

“Conductor” implies that the outer electrons of the atoms are loosely bound and free to
move through the material. Most atoms hold on to their electrons tightly and are
insulators. In copper, the valence electrons are essentially free and strongly repel each
other. Any external influence which moves one of them will cause a repulsion of other
electrons which propagates, “domino fashion” through the conductor.

Simply stated, most metals are good electrical conductors, most non-metals are not.
Metals are also generally good heat conductors while non-metals are not.

Insulators
Most solid materials are classified as insulators because they offer very large resistance
to the flow of electric current. Metals are classified as conductors because their outer
electrons are not tightly bound, but in most materials, even the outermost electrons are
so tightly bound that there is essentially zero electron flow through them with ordinary
voltages. Some materials are particularly good insulators and can be characterised by
their high resistivities

Semiconductors
Solid materials are classified by the way the atoms are arranged within the solid.
Materials in which atoms are placed at random are called amorphous. Materials in which
atoms are placed in a highly ordered structure are called crystalline.

Semiconductors are crystalline or amorphous solids with distinct electrical


characteristics. They are of high resistance - higher than typical resistance materials, but
still of much lower resistance than insulators.

Their resistance decreases as their temperature increases, which is behaviour opposite


to that of a metal.

Pure semiconductors have only a small number of free electrons available and pass a
limited amount of electrical current. In general, their valence electrons are tightly bound
within their crystal (lattice) structure.

Semiconductors are extremely important in modern electronics because they can be


used to control the amount of current in an electrical system. Their conducting properties
may be altered in useful ways by the deliberate, controlled introduction of impurities
(called ‘doping’) into the crystal structure, which lowers its resistance but also permits the
creation of semiconductor junctions between differently-doped regions of the extrinsic
semiconductor crystal.

Although some pure elements and many compounds display semiconductor properties,
silicon, germanium, and compounds of gallium are the most widely used in electronic
devices. All these elements have four electrons in their outer shell (tetravalent).

Conductors and Non-Conductors I Chemistry for All I The Fuse School


https://youtu, be/U8l Fo7ykHuU

1-34 Total Training Support Ltd


Issue 2 - September 2016 ©Copyright 2016
Module 2.1 Matter
Semiconductor crystal materials and
light emitting diodes (LEDs), in
which semiconductor materials are
used in their construction and
operation

An electrical wire has both a conductor


material and an insulating material

Each atom in a silicon lattice has its valence


electrons tightly bound within the lattice
structure. Doping with impurities can create
specific amounts of holes and free electrons,
thus controlling the conductivity

Ceramic insulators are commonly used in


electrical pylons

Total Training Support Ltd 1-35


©Copyright 2016 Module 2.1 Matter Issue 2 - September 2016
Module 2.1 Matter
Summary

Hydrogen
Helium
Lithium
Beryllium
Boron
Carbon
Nitrogen

e z

Pggum ............... ... 2^ 8 1


2
Scandium .... >. .. . .. ... .. .......... 9 s 2
T ' 2 8 10 2
23 Vanadium O
2 &
a 44
11 2
24 IOII1KS 8 13 1
r* Chromium
25 Manganese 2 8 13 2
26 Iron 8 14 2
27 Cobalt 2 8 15 2
11IMIWI 8 16 2
28 Nickel
29 ' " Copper 2 8 18 1
30
Zinc : ■ 8 18 2

Electrons per shell

1-36 Total Training Support Ltd


Issue 2 - September 2016 Module 2.1 Matter
© Copyright 2016
Module 2.1 Matter

2 8 1| 14

45 Rhodium 2 8 18 16
|e..
47 Silver 2 8 W 18 1 ■

.... 3. .... ....................... ..... 3..........

«£«5BBS8i
51 Antimony 2 8 18 18 5
■ •.
53 iodire I 8 jj.® **.....„.7

55.. .......... Caesium ....................................... 2 8 18 8 8 1

iiiiais^^
57 Lanthanum 2 8 18 8 9 2
OllISiBlilllW^
59 Praseodymium 2 8 18 20 9 2
0

Electrons per shell

Total Training Support Ltd 1-37


© Copyright 2016 Module 2,1 Matter Issue 2 - September 2016
Module 2.1 Matter

81 Promethium 2 8 18 22 “9 2!|
62 Samarium 2 o
© 16 23 9 2 I
83 Europium 2 8 18 24 9 2
84 Gadolinium 2 8 18 25 9 2
65 Terbium 2 8 18 26 9 2
88 Dysprosium 2 IH88B8 18 27 9 2
67 Holmium 2 8 18 28 9 2
68 Erbium 2 8 18 29 9 2
69 Thulium 2 8 18 30 9 2
70 Ytterbium 2 8 18 31 9 2
71 Lutetium 2 8 18 32 9 2
Halniitm 2 8 18 32 10 2
73 Tantalum 2 8 18 32 11 2
atiSSfe Tungsten 2 fll||fl ■ill 32 12 2
75 Rhenium 2 8 18 32 13 2
76 BWiSBOtSS^^ BSllll 48 32 14 2
77 Indium 2 8 8 32 15 2
78. O^BKIlliSllSSSIlBlUjBIOS 8 32 IRS 2
79 Gold 2 8 8 32 18 1
80 Mercury : ■ o ■■■ 39 ts Si 2 '
8

Lead 9 ft ft ift
Misfed
83 Bismuth 2 .. .. | ,..... 8 .. 5
84 Potoniui 1
' 2 " "" 8....... 8... ^32... 18.. 7
85 Asatine
Electrons per shell

1-38 Total Training Support Ltd


Issue 2 - September 2016 © Copyright 2016
Module 2.1 Matter
Module 2.1 Matter
.. .: ..■■■■■■■■*;-/ S' ■. .”

S<Eieo£t$
fey;cL:i
J>K
. '. . '.H L : M
86 Radon 2 8 8 32 18 8
37 Francium 2 8 8 32 18 8 1
.As i r, i, • .• !.• l J I;.' ! ■ ■, 'aL'L8"' /.
88 IS BBS! 8 ItlilBSB
.:. ■<■ f,.-, r.‘ r, r.r. r,.^ 8 2
89 Actinium 2 8 18 32 18 9 2
90 SI 8 8l!Wi OHBSSS 9 2
91 Proactinium 2 8 18 32 20 9 2
WHS Uraniuni . • iias«$ 8 iO! OSililB 9 2
93 Neptunium 2 8 18 32 22 9 2
94 aoKiassiiiBii 883# S^BB !H8W®i
9 2
95 Americium 2 8 18 32 24 9 2
98 SOurium Si 8 eofey iiasi

97 Berkelium 2 8 8 32 26 9 2
|8 lasiiiiiaiiiiifii
li ©SIB Slllll 32 27 SUM;

99 Einsteinium 2 8 8 32 28 9 2
100 li 8 ibbsBes BISSIWll
101 Mendelevium 2 8 18 32 30 9 2
IlliS 2 8 liliS BOSSM! SfiaglBa
103 Lawrencium 2 8 18 32 32 9 2

Electrons per shell

Total Training Support Ltd 1-39


© Copyright 2016 Module 2.1 Matter Issue 2 - September 2016
Module 2.1 Matter

Water molecule (H2O)


Magnetic balls

Decomposition of H2O molecule and


recombination to H2 and O2 molecules

(Atomic number) and electron arrangement

1-40 Total Training Support Ltd


Issue 2 - September 2016 © Copyright 2016
Module 2.1 Matter
liodute 2.1 Hatter
Chemical bonding
The photo opposite shows different arrangements of metal balls. These balls are magnetic and
this allows you to make different patterns by sticking them together.

Magnets attract (or repel) each other because of a magnetic force between them. When atoms
combine, they do so because they also experience an attractive force. The force is slightly more
complex than the force between magnets, but it works in the same way: The force holds atoms
together as if they are stuck together with glue. The forces that hold atoms together are called
chemical bonds.

In the water molecule, chemical bonds between O and the two H atoms hold the whole
molecule together.

The water molecule has two identical O-H bonds. What would happen if we had enough energy
to break those bonds?

What would we have if we separated water molecules into their atoms? Theoretically, we would
have hydrogen and oxygen atoms. What actually happens is that the hydrogen atoms
immediately combine to form H2 and the oxygen atoms immediately combine to form O2.

When atoms separate from each other and recombine into different combinations of atoms, we
say a chemical reaction has occurred.

In the above chemical reaction, the water has ‘decomposed’ (broke up) and ‘recombined’ into
smaller molecules. We say that water has undergone a decomposition reaction in the example
above. Of course, not all chemical reactions are decomposition reactions. There are many
different kinds of chemical reactions.

Adhesion and cohesion


‘Cohesion’ is the intermolecular force between liquid particle types (for example, it is what
makes water molecules stick together, or ‘cohere’, to make a rain drop). ‘Adhesion’ is the
intermolecular force between dissimilar atoms (for example, it is what makes the rain drops
‘adhere’ to a washing line). These types of bonding are temporary. Atomic bonding refers to the
permanent bonding between atoms which holds all materials together.

Noble gases
Some atoms are very reluctant to combine with other atoms and exist in the air around us as
single atoms. These are the noble gases and have very stable electron arrangements e.g. 2,
2.8 and 2.8.8 and are shown in the figures opposite.

How Do Atoms Bond I Chemistry for AH I FuseSchool


https://youtu.be/IQKC3BFZK6w

How Do Atoms Bond I Chemistry for All I FuseSchool


https://youtu.be/wOclvkRjOmw

How Do Atoms Bond - Fart 2 1 Chemistry for All I FuseSchool


https://youtu.be/cllKifjMhXRo

Total Training Support Ltd 1-41


© Copyright 2016 Module 2.1 Matter Issue 2 - September 2016
Module 2.1 Matter
Covalent bonding
Covalent bonding is a form of chemical bonding where electrons are shared to form molecules.
This type of bond is usually formed between two non-metallic elements in a molecule.

The molecules might be that of an element i.e. one type of atom only or from different elements
chemically combined to form a compound.

The covalent bonding is caused by the mutual electrical attraction between the two positive
nuclei of the two atoms of the bond, and the electrons between them.

One single covalent bond is a sharing of one pair of electrons, two pairs of shared electrons
between the same two atoms gives a double bond and it is possible for two atoms to share
three pairs of electrons and give a triple bond.

Small covalent molecules


Sometimes called molecular covalent bonding.

The simplest molecules are formed from two atoms and examples of their formation are shown
below. The electrons are shown as dots and crosses to indicate which atom the electrons come
from, though all electrons are the same. The diagrams may only show the outer electron
arrangements for atoms that use two or more electron shells. Examples of small covalent
molecules are...

What Are Dot and Cross Diagrams - Covalent Bonds - Part 1 [ Chemistry for All I FuseSchool
https://youtu.be/7tHM-40iK7E

Example 1 - two hydrogen atoms (1) form the molecule of the element hydrogen H2

® and ® combine to form


combine to form VJAL7 where both atoms have a pseudo helium structure of
two outer electrons around each atom.

Example 2 - two chlorine atoms (2.8.7) form the molecule of the element chlorine CI2

00
and combine to form where both atoms have a pseudo-neon or argon
structure of eight outer electrons around each atom.

Example 3 - one atom of hydrogen (1) combines with one atom of chlorine (2.8.7) to form the
molecule of the compound hydrogen chloride HCI

© and combine to form


where hydrogen is electronically like helium and
chlorine like neon or argon.

1-42 Totai Training Support Ltd


Issue 2 - September 2016 © Copyright 2016
Module 2.1 Matter
Module 2.1 Matter
Example 4 - two atoms of hydrogen (1) combine with one atom of oxygen (2.6) to form the
molecule of the compound we call water H2O

©and®
and combine to form so that the hydrogen atoms are electronically
like helium and the oxygen atom becomes like neon or argon.
HUIrt
\/
The molecule can be shown as O with two hydrogen - oxygen single covalent bonds.

What Are Dot and Cross Diagrams - Covalent Bonds - Part 2 I Chemistry for All | FuseSchool
https://youtu.be/QzytnZY67J8

Example 5 - three atoms of hydrogen (1) combine with one atom of nitrogen (2.5) to form the
molecule of the compound we call ammonia NH3

O
©
Three of and one combine to form so that the hydrogen atoms are
electronically like helium and the nitrogen atom becomes like neon or argon.
N
/|\
The molecule can be shown as n H n with three
nitrogen - hydrogen single covalent bonds.

Example 6 - four atoms of hydrogen (1) combine with one atom of carbon (2.4) to form the
molecule of the compound we call methane CH4.

Four of © and one of *0^ combine to form so that the hydrogen atoms are

electronically like helium and the nitrogen atom becomes like neon or argon.
u
rl
I
c
/1\
H h H
The molecule can be shown as with four carbon ~ hydrogen single covalent bonds.

All the bonds in the above examples are single covalent bonds. Below are three examples 7-
9, where there is a double bond in the molecule, in order that the atoms have stable noble gas
outer electron arrangements around each atom.

Total Training Support Ltd 1-43


©Copyright 2016 Module 2.1 Matter Issue 2 - September 2016
Module 2.1 Matter
Example 7 - two atoms of oxygen (2.6) combine to form the molecules of the element oxygen
O I O
O2.

The molecule has one double covalent bond 0— °.

Example 8 - one atom of carbon (2.4) combines with two atoms of oxygen (2.6) to form carbon
O C JO
dioxide CO2. >—

The molecule can be shown as 0“ C = O wjth two carbon = oxygen double covalent bonds.

Example 9 - two atoms of carbon (2.4) combine with four atoms of hydrogen (1) to form ethane
C2H4.
H
^C = C^
The molecule can be shown as H H with one
carbon = carbon double bond and four carbon - hydrogen single covalent bonds.

The electrical forces of attraction between atoms in a molecule are strong and most molecules
do not change on heating. However, the forces between molecules are weak and easily
weakened further on heating. Consequently, small covalent molecules have low melting and
boiling points. They are also poor conductors of electricity because there are no free electrons
or ions in any state to carry an electric charge. Most small molecules will dissolve in a solvent to
form a solution.

Large covalent molecules


Sometimes called giant covalent bonding.

It is possible for many atoms to link up to form a giant covalent structure. This produces a very
strong three-dimensional covalent bond network. This is illustrated by carbon in the form of
diamond. Carbon can form four single bonds to four other atoms. This type of structure is
thermally very stable and they have high melting and boiling points.

They are usually poor conductors of electricity because the electrons are not usually free to
move as they can in metallic structures. Also because of the strength of the bonding in the
structure, they are often very hard and will not dissolve in solvents like water.

What Are Covalent Bonds I Chemistry for All I FuseSchool


https://youtu.be/h24UmH38_LI

Covalent Bonding in Water, Methane, Ammonia & Hydrogen Fluoride I Chemistry for All I
FuseSchool
https://youtu.be/7mBokkBENWE

Covalent Bonding of Hydrogen, Oxygen & Nitrogen I Chemistry for All I FuseSchool
https://youtu.be/0HfN3CvXP2M

1-44 Total Training Support Ltd


Issue 2 - September 2016 © Copyright 2016
Module 2.1 Matter
Module 2„1 Matter

Intentionally Blank

Totai Training Support Ltd 1-45


© Copyright 2016 Module 2.1 Matter Issue 2 - September 2016
Module 2.1 Matter
Allotropes
Allotropy or allotropism meaning “other”, and tropos, meaning “manner” or “form”) is the
property of some chemical elements to exist in two or more different forms, in the same physical
state, known as allotropes of these elements. Allotropes are different structural modifications of
an element; the atoms of the element are bonded together in a different manner. For example,
the allotropes of carbon include diamond (where the carbon atoms are bonded together in a
tetrahedral lattice arrangement), graphite (where the carbon atoms are bonded together in
sheets of a hexagonal lattice), graphene (single sheets of graphite), and fullerenes (where the
carbon atoms are bonded together in spherical, tubular, or ellipsoidal formations).

What are Allotropes? Non-Metals | Chemistry for All I FuseSchool


https ://youtu. be/R-Q„00a Pq pU

1-46 Total Training Support Ltd


Issue 2 - September 2016 © Copyright 2016
Module 2.1 Matter
Module 2.1 Matter

Both graphite (above) and diamond (below) are


forms of carbon. The difference is the method
of bonding and their crystal structure. They are
just two examples of allotropes of carbon.

Total Training Support Ltd 1-47


© Copyright 2016 Module 2.1 Matter Issue 2 - September 2016
Module 2.1 Matter
Sonic bonding
Ionic bonding is a type of bonding where one atom transfers electrons to another atom. The
atom losing electrons forms a positive ion and is usually a metal. The atom gaining electrons
forms a negative ion and is usually a non-metallic element.

Ionic bonds are formed by one atom transferring electrons to another atom to form ions. Ions
are atoms, or groups of atoms, which have lost or gained electrons.

The atom losing electrons forms a positive ion (a cation) and is usually a meta,. The overall
charge on the ion is positive due to excess positive nuclear charge (protons do not change in
chemical reactions).

The atom gaining electrons forms a negative ion (an anion) and is usually a non-metallic
element. The overall charge on the ion is negative because of the gain, and therefore excess, of
negative electrons.

The examples below combining a metal from groups one (alkali metals), two or three, with a
non-metal from group six or group seven (the halogens).

What are Ionic Bonds? I The Chemistry Journey I The Fuse School
https://youtu.be/zpaHPXVR8WU

Formulae of Ionic Compounds & their Names - Part 1 I The Chemistry Journey I The Fuse
School
https://youtu.be/vfYnhnfdsDO

Formulae of Ionic Compounds & their Names: Part 2 I The Chemistry Journey [ The Fuse
School
https://youtu.be/JylJwHryeSU

1-48 Total Training Support Lid


Issue 2 - September 2016 ©Copyright 2016
Module 2.1 Matter
Module 2.1 Matter

Intentionally Blank

Total Training Support Ltd 1-49


© Copyright 2016 Module 2.1 Matter Issue 2 - September 2016
Module 2.1 Matter
Example 1 - A group one metal + a group seven non-metal, e.g.
sodium + chlorine => sodium chloride NaCI or ionic formula Na+CI'

In terms of electron arrangement, the sodium donates its outer electron to a chlorine atom
forming a single positive sodium ion and a single negative chloride ion. The atoms have
become stable ions, because electronically, sodium becomes like neon and chlorine like argon.

Na (2.8.1) + Cl (2.8.7) Na+ (2.8) Cl” (2.8.8)

A)
A [Na]'
One combines with one to form

Example 2 - A group two metal + a group seven non-metal, e.g. magnesium + chlorine =>
magnesium chloride MgCh or ionic formula Mg2+(CI")2

In terms of electron arrangement, the magnesium donates its two outer electrons to two chlorine
atoms forming a double positive magnesium ion and two single negative chloride ions. The
atoms have become stable ions, because electronically, magnesium becomes like neon and
chlorine like argon.

Mg (2.8.2) + 2CI (2.8.7) Mg2+ (2.8) 2CS - (2.8.8)

.24-

One combines with two to form see

(*Note you can draw two separate chloride ions, but in these examples, a number subscript has
been used, as in ordinary chemical formula)

1-50 Total Training Support Ltd


Issue 2 - September 2016 © Copyright 2016
Moduie 2.1 Matter
Module 2.1 Matter

Chloride ion Sodium ion


ci- Na*

Sodium chloride lattice structure

Total Training Support Ltd 1-51


©Copyright 2016 Module 2.1 Matter Issue 2 - September 2016
Module 2.1 Matter
Example 3 - A group three metal + a group seven non-metal, e.g. aluminium + fluorine
aluminium fluoride AIF3 or ionic formula AI3+(F_)3

In terms of electron arrangement, the aluminium donates its three outer electrons to three
fluorine atoms forming a triple positive aluminium ion and three single negative fluoride ions.
The atoms have become stable ions, because electronically, aluminium becomes like neon and
also fluorine.

A! (2.8.3) + 3F (2.8.7) Ai3+ (2.8) 3F“ (2.8)

34- { F I
€>
One <$> combines with three to form

Example 4 - A group one metal + a group six non-metal, e.g. potassium + oxygen =>
potassium oxide K2O or ionic formula (K*)2O2_

In terms of electron arrangement, the two potassium atoms donate their outer electrons to one
oxygen atom. This results in two single positive potassium ions to one double negative oxide
ion. All the ions have the stable electronic structures 2.8.8 (argon-like) or 2.8 (neon-like)

2K (2.8.8.1) + O (2.6) => 2K+ (2.8.8) 02“ (2.8)


2—
Mt®

Two O combine with one to form

Example 5 - A group two metal + a group six non-metal, e.g. calcium + oxygen => calcium
oxide CaO or ionic formula Ca2+O2~

In terms of electron arrangement, one calcium atom donates its two outer electrons to one
oxygen atom. This results in a double positive calcium ion to one double negative oxide ion. All
the ions have the stable electronic structures 2.8.8 (argon-like) or 2.8 (neon-like)

Ca (2.8.8.2) + O (2.6) =$ Ca2+ (2.8.8) O2~ (2.8)

1-62 Total Training Support Ltd


Issue 2 - September 2016 © Copyright 2016
Module 2.1 Matter
Module 2.1 Matter
Example 6 - A group three metal + a group six non-metal, e.g. aluminium + oxygen =>
aluminium oxide AI2O3 or ionic formula (AI3+)2(O2_)3

in terms of electron arrangement, two aluminium atoms donate their three outer electrons to
three oxygen atoms. This results in two triple positive aluminium ions to three double negative
oxide ions. All the ions have the stable electronic structure of neon 2.8

2AI (2.8.3) + 30 (2.6) 2AP* (2.8) 3O2“(2.8)

3d-
2
Two Sk combines with three to form

Total Training Support Ltd 1-53


©Copyright 2016 Module 2.1 Matter Issue 2 - September 2016
Module 2.1 Matter
The properties of ionic compounds
The ions in an ionic solid are arranged in an orderly way in a giant ionic lattice shown in the
diagram opposite. The ionic bond is the strong electrical attraction between the positive and
negative ions next to each other in the lattice. Salts and metal oxides are typical ionic
compounds.

This strong bonding force makes the structure hard (if brittle) and have high melting and boiling
points. Unlike covalent molecules, all ionic compounds are crystalline solids at room
temperature.

Many ionic compounds are soluble in water, but not all.

The solid crystals do not conduct electricity because the ions are not free to move to carry an
electric current However, if the ionic compound is melted or dissolved in water, the liquid will
now conduct electricity, as the ion particles are now free.

Properties of Ionic Substances I Chemistry for AH I The Fuse School


https://youtu.be/AGD5plXXt4o

Metallic bonding (electron cloud)


The crystal lattice of metals consists of ions, not atoms. The outer electrons (-) from the original
metal atoms are free to move around between the positive metal ions formed (+). These free or
‘de-localised’ electrons are the ‘electronic glue' holding the particles together. There is a strong
electrical force of attraction between these mobile electrons and the Immobile’ positive meta,
ions - this is the metallic bond.

This strong bonding generally results in dense, strong materials with high melting and boiling
points.

Metals are good conductors of electricity because these ‘free’ electrons carry the charge of an
electric current when a potential difference (voltage) is applied across a piece of metal.

Metals are also good conductors of heat. This is also due to the free moving electrons. Non-
metallic solids conduct heat energy by hotter more strongly vibrating atoms, knocking against
cooler less strongly vibrating atoms to pass the particle kinetic energy on. In metals, as well as
this effect, the ‘hot’ high kinetic energy electrons move around freely to transfer the particle
kinetic energy more efficiently to ‘cooler’ atoms.

Typical metals also have a silvery surface but remember this may be easily tarnished by
corrosive oxidation in air and water.

What are metallic bonds? 1 Chemistry for Ail I The Fuse School
https://youtu.be/S08qdOTdOwO

1-54 Total Training Support Ltd


Issue 2 - September 2016 © Copyright 2016
Module 2,1 Matter
Module 2.1 Matter

‘Electron cloud’ formation of ionic (or metallic) bonding

Aluminium

Metals are bonded by electron cloud bonding

Total Training Support Ltd 1-55


©Copyright 2016 Module 2.1 Matter Issue 2 - September 2016
Module 2.1 Matter

Intentionally Blank

1-56 Totai Training Support Ltd


Issue 2 - September 2016
Moduie 2.1 Matter ©Copyright 2016
Module 2.1 Matter
States; solid, liquid and gaseous
Solids
A solid object is characterised by its resistance to deformation and changes of volume.

At the microscopic scale, a solid has these properties:

» The atoms or molecules that comprise the solid are packed closely together.
• These constituent elements have fixed positions in space relative to each other. This
accounts for the solids rigidity. In mineralogy and crystallography, a crystal structure is a
unique arrangement of atoms in a crystal. A crystal structure is composed of a unit cell, a
set of atoms arranged in a particular way; which is periodically repeated in three
dimensions on a lattice. The spacing between unit cells in various directions is called its
lattice parameters.
• If sufficient force is applied, its lattice atomic structure can be disrupted, causing
permanent deformation.

Because any solid has some thermal energy, its atoms vibrate. However, this movement is very
small, and cannot be observed or felt under ordinary conditions.

Liquids
A liquid’s shape is confined to, but not determined by, the container it fills. That is to say, liquid
particles (normally molecules or clusters of molecules) are free to move within the volume, but
they form a discrete surface that may not necessarily be the same as the vessel.

The same cannot be said about a gas; it can also be considered a fluid, but it must conform to
the shape of the container entirely.

Gases
Gases consist of freely moving atoms or molecules without a definite shape and without a
definite volume. Compared to the solid and liquid states of matter a gas has lower density and a
lower viscosity. The volume of a gas will change with changes in temperature or pressure, as
described by the ideal gas law. A gas also has the characteristic that it will diffuse readily,
spreading apart in order to uniformly fill the space of any container.

Plasma
A plasma is typically an ionised gas. Plasma is considered to be a distinct state of matter, apart
from gases, because of its unique properties. ‘Ionized’ refers to the presence of one or more
free electrons, which are not bound to an atom or molecule. The free electric charges make the
plasma electrically conductive so that it responds strongly to electromagnetic fields.

Plasma typically takes the form of neutral gas-like clouds (e.g. stars) or charged ion beams, but
may also include dust and grains (called dusty plasmas). They are typically formed by heating
and ionising a gas, stripping electrons away from atoms, thereby enabling the positive and
negative charges to move more freely.

States of Matter - solids, liquids and gases I Chemistry for Ail j The Fuse School
https://youtu.be/21 CR01 rlmv4

What Is Plasma [ Chemistry for All I FuseSchool


https://youtu.be/94tReSbyPYc

Total Training Support Ltd 1-57


© Copyright 2016 Module 2.1 Matter Issue 2 - September 2016
Module 2.1 Matter
Changes between states
Solids can melt and become liquids, and liquids can boil to become gases. Likewise, gases can
condense to become liquids, and liquids can freeze to become solids.

Sometimes solids can become gases without ever becoming liquids. This is called sublimation.

When a gas becomes a solid without going through the liquid phase, it is called deposition.

1-58 Total Training Support Ltd


Issue 2 - September 2016
Module 2.1 Matter © Copyright 2016
Module 2.1 Matter

Sublimation

Freezing Condensation

Deposition

Total Training Support Ltd 1-59


© Copyright 2016 Module 2.1 Matter Issue 2 - September 2016
Module 2.1 Matter
Brownian motion
Particles in both liquids and gases (collectively called ‘fluids’) move randomly. This is called
Brownian motion. They do this because they are bombarded by the other moving particles in
the fluid. Larger particles can be moved by light, fast-moving molecules.

Brownian motion is named after the botanist Robert Brown, who first observed this in 1827. He
used a microscope to look at pollen grains moving randomly in water. At this point, he could not
explain why this occurred.

What is Brownian motion? I Chemistry for All I The Fuse School


https://youtu.be/NHo6LTXdFns

Brownian Motion
https://youtu.be/EZgbcQtnXME

Thermal expansion of solids, liquids and gases


All three states of matter (solid, liquid and gas) expand when heated. The atoms themselves do
not expand, but the volume they take up does.

When a solid is heated, its atoms vibrate faster about their fixed points. The relative increase in
the size of solids, when heated, is, therefore, small. Metal railway tracks have small gaps so that
when the sun heats them, the tracks expand into these gaps and do not buckle.

Liquids expand for the same reason, but because the bonds between separate molecules are
usually less tight they expand more than solids. This is the principle behind liquid-in-glass
thermometers. An increase in temperature results in the expansion of the liquid which means it
rises up the glass.

Molecules within gases are further apart and weakly attracted to each other. Heat causes the
molecules to move faster, (heat energy is converted to kinetic energy) which means that the
volume of a gas increases more than the volume of a solid or liquid.

However, gases that are contained in a fixed volume cannot expand - and so increases in
temperature result in increases in pressure.

1-60 Total Training Support Ltd


Issue 2 - September 2016 © Copyright 2016
Module 2.1 Matter
Brownian motion - a particle (such as dust particle) moves
randomly, because of collisions with air molecules

Buckled railway lines caused by thermal expansion during


excessive summer temperatures

Units °C
ii11'im'i'i'trnTnwTnwri!i'is 11 I'm1111rriinttriiTi iTTiTTi'TrTTrrnTTT'iTrmii sin mtiuurn 11

Bulb Capillary tube

A liquid thermometer works by thermal expansion

Total Training Support Ltd 1-61


© Copyright 2016 Module 2.1 Matter Issue 2 - September 2016
Module 2.1 Matter
Kinetic particle theory
The kinetic particle theory explains the properties of the different states of matter. The particles
in solids, liquids and gases have different amounts of energy. They are arranged differently and
move in different ways.

Solids
Properties Why they are like this
They have a fixed shape and The particles cannot move
cannot flow from place to place
They cannot be compressed The particles are close
or squashed together and have no space
to move into

Liquids
Properties Why they are like this
hey flow and take the shape The particles are free to
of their container move around each other
They cannot be compressed The particles are close
or squashed together and have no space
to move into

Gases
Properties Why they are like this
They flow and completely fill The particles can move
their container quickly in all directions
They can be compressed or The particles are far apart
squashed and have space to move into

1-62 Total Training Support Ltd


issue 2 - September 2016 © Copyright 2016
Module 2.1 Matter
Module 2.1 Matter
Acids and alkalis
Acids
Properties of acids:

• They are liquids.


• They are solutions of compounds in water.
• if concentrated they can be corrosive.
• Acids taste sour (for example, vinegar).
• Turn blue litmus paper red - this is an easy test for an acid?
® Usually, react with metals to form salts.
» Acids contain hydrogen ions.
® Turn Universal Indicator from green to red, and have a pH less than 7.

Examples of acids: are vinegar (ethanoic acid) and lemon juice (citric acid)

Some common acids used in and around aircraft:

® Hydrochloric acid, HCI


« Nitric acid, HNO3
• Sulphuric acid, H2SO4

Alkalis
Properties of alkalis:

• They feel soapy to touch.


• They are soluble bases.
® Like acids, they can burn the skin.
« They turn red litmus blue - this is how you test for an alkali!
• Alkalis contain hydroxide ions (OH~).
• They taste bitter.
® Turns Universal Indicator from green to blue or purple.

Total Training Support Ltd 1-63


©Copyright 2016 Module 2.1 Matter Issue 2 - September 2016
Module 2.1 Matter
Some common alkalis used in and around aircraft:

• Sodium hydroxide, NaOH


• Ammonia, NH3NH4OH
• Calcium hydroxide, Ca(OH)2

The pH scale
Acids and alkalis can be strong or weak.

The strength of an acid or alkali is shown using a scale of numbers called the pH scale.
The numbers go from 0-14.

On the scale:

• An acidic solution has a pH number less than 7


• An alkaline solution has a pH number greater than 7
• A neutral solution has a pH number of exactly 7.

You can find the pH of any solution using universal indicator. Universal indicator is a
mixture of dyes. It comes as a solution or in paper.

Universal indicator will change from green to a different colour depending on the pH of
the solution you place it in.

What Makes Something Acidic? I Chemistry for All I The Fuse School
https://youtu. be/RnyB2qbQtHk

What makes things alkali? i Chemistry for AH [ The Fuse School


https://youtu.be/zYGnq7-X9w

What is the pH scale I Chemistry for All I FuseSchool


https://youtu.be/ckbsHM2igT0

1-64 Total Training Support Ltd


Issue 2 - September 2016 © Copyright 2016
Module 2.1 Matter
pH scale

Battery acid

Stomach acid

Lemon Juice

Vina gar

Acid rain

Saliva

Sea water

Milk of magnesia

Ammojh^M^S-SOSS

Soapy water

Bleach

Drain cleaner

A nickel cadmium (Ni-Cad)


battery contains potassium
hydroxide (an alkali)

Total Training Support Ltd 1-65


©Copyright 2016 Module 2.1 Matter Issue 2 - September 2016
Module 2.1 Matter

Intentionally Blank

1-66 Total Training Support Ltd


Issue 2 - September 2016
Module 2.1 Matter ©Copyright 2016
Licence Category A, B1, B2 and B3
j s
Physics
2.2 Mechanics
Module 2.2 Mechanics
Copyright notice
© Copyright. AH worldwide rights reserved. No part of this publication may be reproduced, stored in a retrieval
system or transmitted in any form by any other means whatsoever; i.e. photocopy, electronic, mechanical recording
or otherwise without the prior written permission of Total Training Support Ltd.

Knowledge levels - Category A, B1, B2, B3 and C Aircraft Maintenance Licence


Basic knowledge for categories A, B1, B2 and B3 are indicated by the allocation of knowledge levels indicators {1,
2 or 3) against each applicable subject. Category C applicants must meet either the category B1 or the category B2
basic knowledge levels.
The knowledge level indicators are defined as follows:

LEVEL 1
• A familiarization with the principal elements of the subject.

Objectives:
• The applicant should be familiar with the basic elements of the subject.
• The applicant should be able to give a simple description of the whole subject, using common words and
examples.
• The applicant should be able to use typical terms.

LEVEL 2
• A general knowledge of the theoretical and practical aspects of the subject.
• An ability to apply that knowledge.

Objectives:
• The applicant should be able to understand the theoretical fundamentals of the subject.
• The applicant should be able to give a general description of the subject using, as appropriate, typical
examples.
• The applicant should be able to use mathematical formulae in conjunction with physical laws describing the
subject.
• The applicant should be able to read and understand sketches, drawings and schematics describing the
subject.
• The applicant should be able to apply his knowledge in a practical manner using detailed procedures.

LEVEL 3
• A detailed knowledge of the theoretical and practical aspects of the subject.
• A capacity to combine and apply the separate elements of knowledge in a logical and comprehensive
manner.

Objectives:
• The applicant should know the theory of the subject and interrelationships with other subjects.
• The applicant should be able to give a detailed description of the subject using theoretical fundamentals
and specific examples.
• The applicant should understand and be able to use mathematical formulae related to the subject.
• The applicant should be able to read, understand and prepare sketches, simple drawings and schematics
describing the subject.
» The applicant should be able to apply his knowledge in a practical manner using manufacturer’s
instructions.
• The applicant should be able to interpret results from various sources and measurements and apply
corrective action where appropriate.

2-2 Total Training Support Ltd


Issue 2 ~ September 2016 ©Copyright 2016
Module 2.2 Mechanics
Certification statement
These Study Notes comply with the syllabus of EASA Regulation (EU) No. 1321/2014 Annex III
(Part-66) Appendix I, and the associated Knowledge Levels as specified below:

Knowledge
Part-66 Levels
Objective
Reference
A B1 B2 B3
Mechanics 2.2
Statics 2.2.1 1 2 1 1
Forces, moments and couples,
representation as vectors
Centre of gravity
Elements of theory of stress,
strain and elasticity: tension,
compression, shear and torsion
Nature and properties of solid,
fluid and gas
Pressure and buoyancy in liquids
(barometers)
Kinetics 2.2.2 1 2 1 1
Linear movement: uniform
motion in a straight line, motion
under constant acceleration
(motion under gravity);
Rotational movement: uniform
circular motion
(centrifugal/centripetal forces);
Periodic motion: pendula
movement;
Simple theory of vibration,
harmonics and resonance;
Velocity ratio, mechanical
advantage and efficiency
Dynamics 2.2.3
(a) 1 2 1 1
Mass
Force, inertia, work, power,
energy (potential, kinetic and
total energy), heat, efficiency
Module 2.2 Mechanics
(b) 1 2 2 1
Momentum, conservation of
momentum
impulse
Gyroscopic principles
Friction: nature and effects,
coefficient of friction (rolling
resistance)
Fluid dynamics 2.2.4
(a) 2 2 2 2
Specific gravity and density
(b) 1 2 1 1
Viscosity, fluid resistance,
effects of streamlining;
Effects of compressibility on
fluids
Static, dynamic and total
pressure: Bernoulli’s
Theorem, venturi

2-4 Total Training Support Ltd


Issue 2 - September 2016
Module 2.2 Mechanics © Copyright 2016
Module 2.2 Mechanics ®!Jf
Table of Content
2.2.1 Statics_______________________________________________________________ 7
Mass, force and weight_____________________________________________________7
Moments, couples and torque_______________________________________________ 15
Centre of gravity (centre of mass)____________________________________________32
Vectors_____ ___________________________________________________________ 52
Stress, strain and elasticit__________________________________________________99
Secondary stresses - shear, torsion and bending______________________________ 102
Structural definitions ____ 114
Materials behaviour______________________________________________________ 116
Shear force and bending moment diagrams___________________________________118
Nature and properties of solids, liquids and gas _______________________________ 130
Pressure ___ __________________________________________________________ 136
Barometers ____________________________________________________________150
Humidity __________________________________________________ 154
Pressure and force (hydrostatics)___________________________________________158
Fluid pressure and depth__________________________________________________ 182
Buoyancy in liquids.......... .................................................................................................. 188
2.2.2 Kinetics_ __________________________________________________________ 201
Linear movement________ _______________________________________________ 201
Rotational movement_____________________________________________________210
Centrifugal / centripetal acceleration_________________________________________222
Periodic motion __________ 228
Simple theory of vibration _________________________________________________234
Velocity ratio, mechanical advantage and efficiency____________________________ 247
Gear trains and gear ratios__ _____________________________________________266
2.2.3 Dynamics __ 277
Mass_______ 277
Force and inertia________________________________________________________ 277
Work, energy and power__________________________________________________295
Momentum and conservation of momentum__________________________________ 305
Impulse __ ____________________________________________________________ 312
Gyroscopic principles„____________________________________________________ 316
Friction_______ 524
2.2.4 Fluid dynamics_____________________________________________________ 333
Specific gravity and density________________________________________________333
Viscosity 340
Fluid resistance and streamlining ___________________________________________344
Effects of compressibility on fluids___________________________________________348
Static, dynamic and total pressure___________________________________________350
Bernoulli’s theorem and the venturi________ 352

Total Training Support Ltd 2-5


©Copyright 2016 Module 2.2 Mechanics Issue 2 - September 2016
IBjp Module 2.2 Mechanics

1 milligram (mg] 0.0154 grain


SSMSSISfiSIS
ISiiBlSB
1 kilogram [kg] 1,000 g 2.2046 lb
BBlfigggSi

1 tonne [t] 1,000 kg 0.9842 long ton (UK)

®HSaiMR

1 ounce [oz] 437.5 grain 28.35 g


1 pound pb] ' afisssiis 0.4536 kg ■
1 stone 14 lb 6.3503 kg
1 hundredweight [cwt] ■IlSsliSSI
1 slug 14.6 kg
1 long ton (UK) 20 cwT|fgf 1.0Wt.
1 short ton (US) 2,000 lb 0.9071

Mass conversions

-1000 -1000 -1000

mg q kg t
k x 1000 7k x1000 7 k x 1000 7

Mass and weight clarification j


Centripetal force and gravitation
Physics I Khan Academy
https:ZZyoutu.be/luBoeDihLUc

Mass = 100kg
Weight = 162.2N

Mass depends upon the amount of matter.


Weight depends upon the mass and the amount of gravity

2-6 Total Training Support Ltd


Issue 2 - September 2016
Module 2.2 Mechanics ©Copyright 2016
Modub 2.2 Mechanics
2.2.1 Statics
Mass, force and weight

Mass
Mass (symbolized ‘m’) is a dimensionless quantity representing the amount of matter in a
particle or object. The standard unit of mass in the International System (SI) is the kilogram (kg),
and the slug in the imperial system.

1 slug = 14.59 kilogram

In physics, mass is a property of a physical body. It is a measure of an object’s resistance to


acceleration (a change in its state of motion) when a force is applied. It also determines the
strength of its mutual gravitational attraction to other bodies.

Force
The physicist uses the word ‘force’ to describe any push or pull. A force is one kind of vector. A
vector is a quantity that has both size and direction.

A force has a certain magnitude or size. Also, a force is always in a certain direction. To
completely describe a force, it is necessary to specify both the size of the push or pull and its
direction.

The units in which force are measured are the pound (lb) in the imperial system and the newton
(N) in the metric system. The newton is named after Sir Isaac Newton, a famous British
physicist who lived in the 17th century.

The relationship between the metric and imperial units is given by the conversion factor:

1 lb = 4.448 N

Weight
A weight is one kind of force. It is defined as the gravitational pull of the earth on a given body.
The direction of this force is toward the geometrical centre of the earth.

Distinction between mass and weight


The physicist very carefully distinguishes between ‘mass’ and ‘weight’. As we have seen, mass
is the quantity of matter, determined by the number of protons and neutrons in the body, and
weight is a measure of the gravitational pull of the earth on this quantity of matter.

It may seem that this is an unimportant distinction. However, there is one important difference.

The mass of an object is the same wherever this object is in the universe. The mass of a stone
is the same if the stone is on the earth, on Mars, in a space ship, or some place in the Milky
Way Galaxy. If the stone is not on the earth but is in a space station orbiting the earth some
distance from the earth’s surface, the weight of this stone is different from its weight on the
earth’s surface. If the stone is on the planet Mars, we speak of its ‘weight on Mars’, the
gravitational pull of Mars on the stone.

Total Training Support Ltd 2-7


©Copyright 2016 Module 2.2 Mechanics Issue 2 - September 2016
Module 2.2 Mechanics
As you have probably figured out, the greater the mass of an object on the surface of the earth,
the greater is the weight of this object These two quantities are approximately proportional to
each other as long as the body remains on the surface of the earth. The word ’approximately’ in
the previous sentence refers to the fact that the pull of the earth on a body of a given mass
varies slightly with the position of the body on the earth’s surface. For example, a body that
weighs 57.3 lbs at the North Pole would weigh 57.0 lbs at a place on the equator. This occurs
because a body at either pole is slightly closer to the centre of the earth than it is at the equator.
Thus, the pull of the earth on the body is greater at the poles and slightly smaller at other places
on the earth. However, we usually neglect this slight difference.

Physicists and engineers measure masses of bodies in slugs or kilograms and weights in
pounds or Newtons. The equation relating mass and weight is:

w = mg

This is a form of Newton’s second law (F = ma).

In this equation, g has a definite numerical value. We will use the following relations:
lbs JN
g = 32 or g = 9.8
slug kg
There is a great source of confusion in British marketing practices. For example, we often see
on a packet of sugar the information regarding the contents:

1 kg or 2.2 lbs

We note that 2.2 lbs equals 1 kg. We have just learned that 2.2 lbs is the ‘weight’ of the sugar
and that 1 kg is the ‘mass’ of the sugar. In other words, British packaging practices list the
weight of the product if we deal with the imperial system and list the mass of the product if we
are in the metric system.

For example, suppose the weight of a piece of cheese is marked 32 oz. and we wish to know
the number of grams. First we convert the weight in ounces to 2 lbs. Then we convert from
pounds to Newtons.
4.448 N
W = 2 lbs x = 8.90 N
1 lb

Next, we use the relation:


W
w = mg or m=—
9
Therefore, we write:

m =. ............ = 0.908 kg = 908 grams


9.8 N/kg

Note that we can convert from pounds to Newtons since both are units of weight and we can
convert from kilograms to slugs since both are units of mass. However, if we want to find a
mass if we know a weight or a weight if we know a mass we must use the equation:

m ~ w/g or w - mg

2-8 Total Training Support Ltd


Issue 2 - September 2016 © Copyright 2016
Module 2.2 Mechanics
Module 2.2 Mechanics
in summary, let us note that mass is a measure of the quantity of matter - ultimately, a measure
of the number of protons and neutrons in the body and weight is the force with which the earth
pulls on a body. These are related but not identical concepts. The units of mass are slugs and
kilograms. The units of weight are pounds and Newtons. A mass can be changed from slugs to
kilograms and vice versa. A weight can be changed from Newtons to pounds or vice versa.
However, one cannot say that one pound equals 454 grams. The only correct statement is that
a body having a weight of one pound has a mass of 454 grams. The equation relating mass and
weight is:

w = mg or m=—
9
Mass and weight clarification | Centripetal force and gravitation I Physics I Khan Academy
https://youtu.be/luBoeDihLUc

Weighing scales indicating in stones and kg Weighing scales indicating in lb and kg

Total Training Support Ltd 2-9


©Copyright 2016 Module 2.2 Mechanics Issue 2 - September 2016
Module 2.2 Mechanics

Intentionally Blank

2-10 Total Training Support Ltd


Issue 2 - September 2016
Module 2.2 Mechanics ©Copyright 2016
Module 2.2 Mechanics
Worksheet

1. What is the mass of sugar having a weight of 45 N?

2. What is the weight of a dumbbell having a mass of 23 kg?

3. What is the mass of a man having a weight of 350 lbs?

4. What is the weight of an object having a mass of 23.6 slugs?

5. What is the weight (in lbs) of the corn flakes in a box where the mass is listed as 680 g?

6. What is the mass in grams of 2.5 lbs of pasta?

Totai Training Support Ltd 2-11


© Copyright 2016 Modute 2.2 Mechanics Issue 2 ~ September 2016
Module 2.2 Mechanics

intentionally Blank

2-12 Total Training Support Ltd


Issue 2 - September 2016
Module 2.2 Mechanics © Copyright 2016
Modute 2.2 Mechanics
Answers

All answers are to 3 significant figures

1. 4.59 kg

2. 225 N

3. 10.9 slugs

4. 755 lbs

5. 1.45 lbs

6. 1,140g

Total Training Support Ltd 2-13


© Copyright 2016 Module 2.2 Mechanics Issue 2 - September 2016
Module 2.2 Mechanics

Intentionally Blank

2-14 Total Training Support Ltd


Issue 2 - September 2016
Module 2.2 Mechanics © Copyright 2016
Module 2.2 Mechanics
Moments, couples and torque

Lever arms and moments


The distance of any object from the fulcrum is called the ‘lever arm’. The lever arm multiplied by
the weight (or mass) of the object is its turning effect about the fulcrum. This turning effect is
known as the ‘moment’.

The moment of a force about a point is the product of the force and the perpendicular distance
from the point to the line of action of the force.

Moment = Fd

Where;
F = the force in newtons (N) or pounds (lb)
d = perpendicular distance in metres (m) or feet (ft)

Moments are expressed in lbs.ft or Newton metres and the direction must be specified,
clockwise (+) or anticlockwise (-), positive or negative.

Total Training Support Ltd 2-15


© Copyright 2016 Module 2.2 Mechanics Issue 2 - September 2016
Module 2.2 Mechanics
Example 1

A 10 N force acts on a spanner, at a perpendicular distance of 100 mm from the centre of the
nut and bolt. What is the moment M1 applied to the nut and bolt?

Moment = Fd

= 10x100

= 1,000 Nm

What would the moment (M3) be if the spanner was 3 times longer?

Moment = Fd

= 10x300

= 3,000 Nm

The principle of moments


When an object is in equilibrium the sum of the anticlockwise moments about a turning point
must be equal to the sum of the clockwise moments.

sum of anticlockwise moments = sum clockwise moments

The simple see-saw is a good example, A small weight at a large distance can be balanced by
a large weight at a small distance.

Example 2

Person B of mass 500 N sits on a see-saw, 2 m from the fulcrum. How far from the fulcrum
would person A, of mass 1,000 N need to sit, in order for the see-saw to be balanced?

sum of anticlockwise moments = sum clockwise moments

500 N x 2m = 1,000 NxX

X = 500 Nx2m/1,000 N

X=1m

2-16 Total Training Support Ltd


Issue 2 - September 2016
Module 2.2 Mechanics ©Copyright 2016
Module 2.2 Mechanics

Nut & Bolt

Moment = Force x distance

M = Fd

<------ 100 m m-------- <-------------- 500mm---------- '---- >


Example 1

Moment = Force x distanc

Person 8 Person A
SOON 1,OOGN

Example 2

Sum of anticlockwise moments = Sum of clockwise

Total Training Support Ltd 2-17


©Copyright 2016 Module 2.2 Mechanics Issue 2 - September 2016
Module 2.2 Mechanics
Example 3

In the figure shown opposite, we can write an equilibrium formula as follows:

sum of anticlockwise moments := sum of clockwise moments

Wi x di := W2 X d2

3 lb x 10" == 6 lbs x 5"

30 lb ins == 30 lb ins

Example 4

In the figure shown opposite, we can write an equilibrium formula, and determine the necessary
size of the weight required (X) to be placed at 7.5" in order to balance the system, as follows:

sum of anticlockwise moments ■= sum of clockwise moments

(Wixdi) == (W2 x d2) + (W3 x ds)

30 lb ins ■= 15 lb ins + (X x 7.5")

30-15 := 7.5X

= 15/7.5

X == 2 lbs

2-18 Total Training Support Ltd


Issue 2 - September 2016 © Copyright 2016
Module 2.2 Mechanics
Module 2.2 Mechanics
Weighti x Distance = Weights x Distances

Anticlockwise moment Clockwise moment

Sum of anticlockwise moments = Sum of clockwise moments

Wi x di = W2 x d2

Example 3

Sum of anticlockwise moments = Sum of clockwise moments

W1 x di = W2 x d2

Example 4

Sum of anticlockwise moments = Sum of clockwise moments

(W1 x di) = (W2 x d2) + (Ws x ds)

Total Training Support Ltd 2-19


©Copyright 2016 Module 2.2 Mechanics Issue 2 - September 2016
Module 2.2 Mechanics
Couples
A ‘couple’ is a pair of forces of magnitude F that are equal and opposite but applied at points
separated by a distance d perpendicular to the forces. The combined moment of the forces
produces a torque Fd on the object on which they act.

An example is the cutting of an internal thread with a tap and tap wrench. The force applied at
one end of the wrench handle, multiplied by the distance to the centre of rotation is just half of
the torque felt at the tap itself, since there is an equal torque applied at the other wrench handle.

The moment (or torque) of a couple is calculated by multiplying the size of one of the force (F)
by the perpendicular distance between the two forces (d).

E.g. a steering wheel in a car;

Moment of Couple = Fd

Moments I Moments, torque, and angular momentum I Physics I Khan Academy


https://youtu.be/ESusD8HRLBI

Moments (part 2) I Moments, torque, and angular momentum I Physics I Khan Academy
https://youtu.be/peXbz0frV74

Introduction to torque I Moments, torque, and angular momentum I Physics I Khan Academy
https://youtu.be/QhuJn8YBtmg

2-20 Total Training Support Ltd


Issue 2 - September 2016 ©Copyright 2016
Module 2.2 Mechanics
Module 2.2 Mechanics

A couple’ is a pair of forces of magnitude F


that are equal and opposite but applied at
points separated by a distance d
perpendicular to the forces. The combined
moment of the forces produces a torque Fd
on the object on which they act.

Couple ~ tangential force applied by just one hand


multiplied by the distance between the hands

Totat Training Support Ltd 2-21


© Copyright 2016 Module 2.2 Mechanics Issue 2 - September 2016
Module 2.2 Mechanics

Intentionally Blank

2-22 Total Training Support Ltd


Issue 2 - September 2016
Module 2.2 Mechanics © Copyright 2016
Module 2.2 Mechanics
Worksheet

The diagrams below show a series of beams balancing on a pivot.

Determine the size of the mass or the distance as required to maintain the beam in balance.

Total Training Support Ltd 2-23


© Copyright 2016 Moduie 2.2 Mechanics Issue 2 - September 2016
Module 2.2 Mechanics

15. A ship’s wheel has a couple applied to it by the captain of 60 Nm. The diameter of the
wheel is 0.8 m. What is the force applied on just one side of the wheel?

2-24 Total Training Support Ltd


Issue 2 - September 2016
Module 2.2 Mechanics © Copyright 2016
Module 2.2 Mechanics
Answers

1. 1 N

2- 4 N

3. 2m

4. 8N

5. 5N

6. 4m

7. 10 N

8, 5m

9. 9N

10. 4N

11. 4 N

12. 1 N

13. 3 m

14. 3 m

15. 75 N

Total Training Support Ltd 2~25


© Copyright 2016 Module 2.2 Mechanics Issue 2 - September 2016
Module 2.2 Mechanics
Torque wrenches
Consider the diagrams in the figure shown opposite. We define torque as the force (F) applied
to a body that is pivoted at a point (0) multiplied by the distance from the pivot point to the place
where the force is applied and multiplied by the sin of the angle between r and F. We will use
the Greek letter tau (t) for torque. The distance mentioned in the preceding sentence is called
the lever arm and symbolized by the letter d.

The defining equation is:

x = Fd sin 9

In the diagram, we note that 9 = 90°. This is by far the most common case. Since sin 90° = 1,
this common case reduces to the simpler equation:

x = Fd

However, it must be remembered that in those cases where 9 is not 90°, the full equation must
be used. Note also that the unit for torque is the lb ft, lb in or the Nm.

Extensions
The figure opposite shows a typical beam type torque wrench which has an extension spanner
attached. If this combination is used to torque load a fastener, then the following formula should
be used to calculate the wrench scale reading which corresponds to the specified torque value:

Scale reading = specified torque x ~~~

Where; L = distance between the driving tang and the centre of the handle

X = length of extension spanner between centres

A simple way of calculating the scale reading required without using the formula is set out in the
following example, for which the specified torque loading is 300 lb in and the lengths of the
wrench and spanner are 10 and 5 inches respectively.

a) Force required on wrench handle to produce a torque of 300 lb in is 300 lb in divided by


the distance between nut and wrench handle,
300 Ib.in
10 in + 5 in
b) Scale reading when force on handle is 20 lb is, 20 lb x 10 in 200 lb in.

Force must therefore be applied to the wrench handle until a reading of 200 lb in is shown on
the wrench scale, and this will represent a 300 lb in torque load applied to the nut. With the
‘break’ type wrench, the adjustment must be pre-set at 200 lb in.

When using an extension spanner with a torque wrench, the spanner and wrench should be as
nearly as possible in line. If it is necessary to diverge by more than 15° from a straight line (due,
for example, to intervening structure), then the direct distance (D) between the nut and wrench
handle must be substituted for ‘L + X’ in the formula for calculating wrench scale reading. This is
shown in the figure opposite, and the scale reading in this instance will be equal to specified
torque x.

2-26 Totai Training Support Ltd


Issue 2 - September 2016 © Copyright 2016
Module 2.2 Mechanics
Siodute 2.2 Mechanics
How to use an crowfoot or extension
adapter on a torque wrench
https://youtu.be/tvKHH55qdQ

r 90°
p

A torque wrench fitted with an extension spanner positioned out-of-line with the wrench

Total Training Support Ltd 2-27


©Copyright 2016 Module 2.2 Mechanics Issue 2 - September 2016
Module 2.2 Mechanics
Whenever a torque wrench is used, it must be confirmed that the specified torque and the
wrench scale are in the same units; if not, then the specified torque should be converted, by
calculation, to the units shown on the wrench scale, and any measurements taken in
appropriate units.

When applying torque, the wrench handle should be lightly gripped and force applied smoothly
at 90° to the axis of the wrench.

2-28 Total Training Support Ltd


Issue 2 - September 2016 ©Copyright 2016
Module 2.2 Mechanics
Module 2.2 Mechanics
Worksheet

1. Calculate the torque applied to a nut and bolt by a 12-inch spanner when a force of 12 lb is
applied perpendicular at the end of the spanner.

2. How much force is required to torque a nut and bolt to 50 Nm with a wrench 0.5 m long?

3. A nut is to be torqued to 50 in lb. A torque wrench of 17-inches is used with an extension of


3 inches. What setting should the torque wrench be adjusted to?

Total Training Support Ltd 2-29


©Copyright 2016 Module 2.2 Mechanics Issue 2 - September 2016
Module 2.2 Mechanics

Intentionally Blank

2-30 Total Training Support Ltd


Issue 2 - September 2016
Module 2.2 Mechanics ©Copyright 2016
Module 2.2 Mechanics
Answers

1. 144 in lb

2. 100 N

3. 42.5 in lb

Total Training Support Ltd 2-31


© Copyright 2016 Module 2.2 Mechanics Issue 2 - September 2016
Module 2.2 Mechanics
Centre of gravity (centre of mass)

Centre of mass of regular and irregular shapes (centroids)


In many applications it is important that objects are designed with stability in mind. This requires
an understanding of the centre of mass, as well as an ability to find out where it is. By
incorporating a low centre of mass and wide base into an object, we can reduce the chance of it
toppling over.

Mass is the amount of matter an object has. Every part of an object forms part of its overall
mass. But when we try to balance an object on a point, there will only be one place where it will
balance.

You can therefore think of the mass of an object being concentrated at this point, known as the
centre of mass, or ‘centroid’.

Finding the centre of mass for symmetrical objects


The centre of mass for a symmetrical object can be found easily. The axes of symmetry are
marked on the object. The centre of mass is where the axes of symmetry cross.

Finding the centre of mass by suspending objects


The centre of mass for an irregular shaped, non-symmetrical object is found in a different way.

1. Drill a small hole in the object and hang it up so that it is free to swing without obstruction.

2. Hang a plumb line (a piece of string with a weight hanging from it) from the same
suspension point. This lets you mark the vertical line directly below the suspension point.

3. Drill another hole at a different location within the object,

4. Again hang a plumb line to determine the vertical and mark it on.

5. The point at which the two marked lines cross is the centre of mass.

Center of mass I Impacts and linear momentum { Physics I Khan Academy


https://youtu.beA/rflZifKluw

Centre Of Gravity
https://youtu.be/R8wKV0UQtlo

2-32 Total Training Support Ltd


Issue 2 - September 2016 © Copyright 2016
Module 2.2 Mechanics
itoduJe 2.2 Mechanics

Finding the centroid of regular shapes

Centre of mass
(centroid) is where
the lines cross

Finding the centroid of irregular shapes


Total Training Support Ltd 2-33
©Copyright 2016 Module 2.2 Mechanics Issue 2 - September 2016
Module 2.2 Mechanics
Centre of gravity (centre of mass) by calculation
When a beam is pivoted at some point (the ‘fulcrum’), and has several masses positioned at
different points along the beam, such that the beam is in equilibrium (balanced), then the
position of the fulcrum can be found by standard principle of moments and calculation.

Arm
The arm is the horizontal distance that an item is located from the datum. If the particular item is
located forward, or to the left of the datum, it is shown with a negative (-) sign. If the item is
located aft, or to the right of the datum, it is shown with a positive (+) sign.

The location of the datum is arbitrary, but for convenience, it is usually considered to be at one
end of the beam.

The conventional signs which are applied to arms and moments in relation to their direction
from the C of G datum are as follows:

• Horizontal (-) forward and (+) aft of the datum


• Vertical (-) below and (+) above the datum
• Transverse (-) right and (+) left of the datum

Example

A uniform beam weighs 40 lbs; it is 15 ft long and has a load of 10 lbs, 25 lbs and 45 lbs at
distances of 2 ft, 6 ft and 14 ft from the left hand end respectively. Find the C of G position
relative to the centre of the beam.

2-34 Total Training Support Ltd


Issue 2 - September 2016 ©Copyright 2016
Module 2.2 Mechanics
Module 2.2 Mechanics
45 lbs

Take moments about X

Clockwise turning moments - 45 x 6.5 = 292.5 lbs ft


Anticlockwise turning moments - 25x1.5 + 10x5.5 = 92.5 lbs ft
Resultant moment = greater - lesser
= 292.5-92.5 = 200 lbs ft

Thus, as the resultant moment is clockwise, the C of G must be to the right of X.

C of G relative to X = resultant moment


total weight

Total Weight = 10 + 25 + 40 + 45 = 120 lbs

Thus, C of G position = 200 = 1 ft 8 in


120

i.e., the C of G is 1 foot 8 inches to the right of X.

Note: The 45 lbs force acts 6.5 ft from ‘X’ (14 - 15/2)

Total Training Support Ltd 2-35


©Copyright 2016 Module 2.2 Mechanics Issue 2 - September 2016
Module 2.2 Mechanics
The tabular method
The tabular method is useful when the moment system is complex.

The method works by listing the masses (including the mass of the beam itself, acting at the
centre of the beam, assuming that the beam mass is uniformly distributed). In the next column
the arm of each respective mass is listed. The datum point, for convenience, is taken to be one
end of the beam.

The masses and the arms are then totalled at the bottom of their respective columns.

The position of the CG (measured from the

Example

A uniform beam 60 cm long and having a mass of 8 kg, has masses of 2 kg, 10 kg, 20 kg and
30 kg at distances of 6 cm, 14 cm, 23 cm and 36 cm respectively, from the left hand end. Find
C of G of beam.

2-36 Total Training Support Ltd


Issue 2 - September 2016 © Copyright 2016
Module 2,2 Mechanics
Module 2.2 Mechanics

Item Mass (kg) Arm (cm) Moment (kg cm)


Beam 8 30
1st load 2 6
2nd load 10 14
3rd load 20 23
4th load 30 36
Totals

C of G of the whole system resultant moment


total mass

Total Training Support Ltd 2-37


©Copyright 2016 Module 2.2 Mechanics Issue 2 - September 2016
Module 2.2 Mechanics
Aircraft centre of gravity
An aircraft is balanced if it remains level when suspended from an imaginary point (see diagram
opposite). This point is the location of its ideal CG.

The CG of an aeroplane or helicopter is the location where ail the mass of the aircraft is
considered to act.

Balancing
Aircraft balance refers to the location of the centre of gravity along the longitudinal axis of the
aircraft. The Centre of Gravity (CG) is the point about which an aircraft would balance if it were
possible to suspend it from that point. On weight and balance drawings the location of the CG is
usually denoted the symbol shown opposite.

Obtaining this balance is simply a matter of placing loads so that the average arm of the loaded
aircraft falls within the C of G range. The exact location of the range is specified for each type of
aircraft in the flight manual.

Centre of gravity limits


Centre of gravity (CG) limits are specified longitudinal (forward and aft) and/or lateral (left and
right) limits within which the aircraft’s centre of gravity must be located during flight. The CG
limits are indicated in the aeroplane flight manual. The area between the limits is called the CG
range of the aircraft.

If, due to incorrect loading of an aircraft, the aircraft’s CG is located outside of the designated
limits, the aircraft will be difficult to control, or suffer increased fuel consumption, or both.

The datum
The ‘datum’ is an imaginary vertical line from which all horizontal measurements are taken or
indicated, with the aircraft in level flight attitude, for balance purposes,

The datum may be located at any convenient location by the manufacturer of the aircraft.

There is no fixed rule for the location of this datum. In most cases it is located on the nose of the
aircraft or some point on the aircraft structure itself.

There are some examples in the diagram opposite.

How an Aeroplane Flies - Balance and Stability


https://youtu.be/fLup4nORfmQ

Moments of force
https://youtu.be/zhWDnWal5TE

2-38 Total Training Support Ltd


Issue 2 - September 2016 © Copyright 2016
Module 2.2 Mechanics
Module 2.2 Mechanics

if an aircraft is
suspended from its
centre of gravity
position, it will balance

Center of gravity

Graphical symbol for


centre of gravity
position
Total weight

Centre of Gravity (CG) is the point at which all the


mass of the aircraft can be considered to act

The datum is 66.25 inches The datum is at the front


ahead of the wing’s leading edge face of the firewall

The datum is 1097 inches ahead of


the centre of the main gear

1097 in
... .......... mi...............

7h Examples of centre of
gravity position datum
. locations
....

Total Training Support Ltd 2-39


©Copyright 2016 Module 2.2 Mechanics Issue 2 - September 2016
Module 2.2 Mechanics
Aircraft weighing methods
Aircraft weighing equipment consist of weighbridge scales, hydrostatic weighing units or
electrical/electronic strain gauge type weighing equipment. The capacity of the equipment must
be compatible with the load, so that accurate measurements may be obtained.

• Weighbridge scales

These consist of a separate weighing platform for each wheel or bogey, the mass at
each reaction point being indicated directly on the balance arm or on a dial indicator.
Large aircraft may be weighed in a hangar, using portable weighbridge scales, or on
weighbridges set permanently into the floor.

• Hydrostatic weighing units

These are based upon the principle that fluid pressure in a cylinder, in which a piston is
working, depends on the area of the piston and the load applied to it. The units are
placed between the lifting jacks and the aircraft jacking points and the weight at each
position recorded on a gauge.

The gauge may be calibrated directly into weight units or a conversion may be required
to obtain the correct units. It is important that the jacks used with these units are vertical
and the units correctly positioned, otherwise side loads may be imposed on the units and
inaccurate readings obtained.

2-40 Total Training Support Ltd


Issue 2 - September 2016
Module 2.2 Mechanics ©Copyright 2016
Module 2.2 Mechanics

Weighing using weighbridge scales

Hydrostatic weighing units


Total Training Support Ltd 2-41
© Copyright 2016 Module 2.2 Mechanics Issue 2 - September 2016
Module 2.2 Mechanics
• Electrical or electronic weighing equipment

These type incorporate three or more weighing cells, using metallic resistance elements
or strain gauges, the resistance of which varies with change in length, due to elastic
strain.

These strain gauges are either incorporated into cells between the aircraft and the jacks,
or they are used in portable weighbridge platforms placed beneath the aircraft wheels.
The output may be measured with a galvanometer, or sent to an instrumentation unit,
which adds all of the platform values and digitally displays the aircraft load.

Weighing on aircraft jacks


Jacking should be done in accordance with the maintenance manual procedures and suitable
jacking adapters should be placed at the jacking points. Weighing units of sufficient capacity
should be attached to the jacks and the jacks positioned at each jacking point.

Zero indication of each weighing unit should be verified, before the aircraft is raised evenly, until
clear of the ground when the aircraft should be levelled. Readings should be made at each
weighing point, and to ensure representative readings are obtained, a second reading should be
made.

The mass of the aircraft may be deduced by adding all of the readings from each weighing
point.

With the aircraft weight correctly established, it remains only to calculate the CG.

2-42 Total Training Support Ltd


Issue 2 - September 2016 © Copyright 2016
Module 2.2 Mechanics
Modute 2.2 Mechanics

Total Training Support Ltd 2-43


©Copyright 2016 Moduie 2.2 Mechanics Issue 2 - September 2016
Module 2.2 Mechanics
Calculation of aircraft’s CG
While the CG of a nose-wheeled aircraft must, obviously, be somewhere near the main wheels,
its location may be found, using the previously described methods.

Example 1

A nose-wheeled aircraft, which has been weighed and its dry operating mass has been
calculated to be 1,915 kg. The CG is forward of the main undercarriage and, using the main
wheel centre-line position as a reference datum, the CG can be found by means of the formula:

A x B 4- C

Where; A = Mass at the nose wheel

B = Distance between the front and rear reactions

C = Total Mass (the sum of all the reactions)

Thus: AxB =100x6 = 600 kg m

and, as C = 1,915 kg

then, the CG = 600 (kg m) -5-1,915 (kg) = 0.3133 m or 313.3 mm.

The CG is, therefore, calculated (to 3 significant figures) to be 313 mm forward of the main
wheel centre-line.

2-44 Total Training Support Ltd


Issue 2 - September 2016 ©Copyright 2016
Module 2,2 Mechanics
Module 2,2 Mechanics

Calculation aircraft’s CG (main


wheel centre-line as reference
datum)

Total Training Support Ltd 2-45


© Copyright 2016 Module 2.2 Mechanics Issue 2 - September 2016
Module 2.2 Mechanics
Example 2

The main wheel centre-line is not, however, always taken as the reference datum and, as
another example, the reference datum could be taken to be somewhere between the nose
wheel and the main wheel positions.

In this instance it is necessary to use the formula:

Total
CG = moment
Total mass

It is also necessary to remember that moments, which are aft of a reference datum are
considered to be positive or additive (+ve), while moments, which are forward of the datum are
considered to be negative or subtractive (-ve).

Using the data, displayed in the table below, enables the aircraft’s CG to be calculated.

Position Mass Arm (m) Moment (kg m)


(kg)
Left main wheel 905 +4 3,620
Right main 910 +4 3,640
wheel
Nose wheel 100 -2 -200
Totals 1,915 +7,060
Calculation o aircraft’s CG

Thus total moment (+7,060 kg m) +• total mass (1,915 kg)


= +3.687 m (to three decimal places).

Therefore, the CG is located 3.687 metres aft of the reference datum (which, by observation,
remains at 313 millimetres forward of the main wheel centre-line, as previously calculated).

Weight and Balance


https://youtu.be/HgOcA9eES78

2~46 Total Training Support Ltd


Issue 2 - September 2016
Module 2.2 Mechanics © Copyright 2016
Module 2.2 Mechanics

Example 2

Calculation aircraft’s CG
(reference datum aft of
nose wheel)

Total Training Support Ltd 2-47


©Copyright 2016 Module 2.2 Mechanics Issue 2 - September 2016
Module 2.2 Mechanics

Intentionally Blank

2-48 Tota! Training Support Ltd


Issue 2 - September 2016
Module 2.2 Mechanics ©Copyright 2016
Module 2.2 Mechanics 1I!SP
Worksheet

1. An aeroplane stands with its nose wheel and main wheels on weighing scales. The
longitudinal distance of the nose wheel from the datum is 1.2 m. The longitudinal distance
of the mainwheels is 3.5 m.

An electronic display shows the mass indicated on the nose gear is 1,100 kg, and 2,300 kg
on each nose leg.

Determine the CG position from the datum, of the aeroplane.

2. The datum of an aeroplane is 100 inches forward of the leading edge of the wing spar, or
128 inches forward of the main-wheel weighing points. The weight on the nose wheel is
340 pounds, and the distance between main wheels and nose wheel is 78 inches. The total
weight of the aeroplane is 2,006 pounds.

Determine the CG position from the datum, of the aeroplane.

3. The distance between the datum of an aeroplane and the main-wheel weighing points is 75
inches, the weight of the nose wheel is 340 pounds, and the distance between main wheels
and nose wheel is 78 inches. The total net weight of the aeroplane is 2,006 pounds.

Determine the CG position from the datum, of the aeroplane.

Total Training Support Ltd 2-49


© Copyright 2016 Module 2.2 Mechanics Issue 2 - September 2016
Module 2.2 Mechanics

Intentionally Blank

2-50 Total Training Support Ltd


Issue 2 - September 2016
Module 2.2 Mechanics ©Copyright 2016
Module 2.2 Mechanics
Answers

1. 3.06 m

2. 114.8 inches

3. 88.2 inches

Total Training Support Ltd 2-51


©Copyright 2016 Module 2.2 Mechanics Issue 2 - September 2016
Module 2.2 Mechanics
Vectors
A study of physics will involve the introduction of a variety of quantities that are used to describe
the physical world. Examples of such quantities include distance, displacement, speed, velocity,
acceleration, force, mass, momentum, energy, work, power, etc. All these quantities can by
divided into two categories ~ vectors and scalars.

A vector quantity is a quantity that is fully described by both magnitude and direction. On the
other hand, a scalar quantity is a quantity that is fully described by its magnitude alone.

Examples of vector quantities include displacement, velocity, acceleration, and force. Each of
these quantities are unique in that a full description of the quantity demands that both a
magnitude and a direction are quoted. Vector quantities are not fully described unless both
magnitude and direction are listed.

Representing vectors
Vector quantities are often represented by scaled vector diagrams. Vector diagrams depict a
vector by use of an arrow drawn to scale in a specific direction. An example of a scaled vector
diagram is shown in the diagram opposite. The vector diagram depicts a displacement vector.
Observe that there are several characteristics of this diagram that make it an appropriately
drawn vector diagram.

• a scale is clearly provided


• a vector arrow (with arrowhead) is drawn in a specified direction. The vector arrow has a
head and a tail.
• the magnitude and direction of the vector is clearly labelled. In this case, the diagram
shows the magnitude is 20 m and the direction is 30 degrees west of north.

Conventions for describing directions of vectors


Vectors can be directed due east, due west, due south, and due north. But some vectors are
directed northeast (at a 45-degree angle); and some vectors are even directed northeast, yet
more north than east. Thus, there is a clear need for some form of a convention for identifying
the direction of a vector that is not due east, due west, due south, or due north. There are a
variety of conventions for describing the direction of any vector. The two conventions that will be
discussed and used in this unit are described below:

1. The direction of a vector is often expressed as an angle of rotation of the vector about its
“tail” from east, west, north, or south. For example, a vector can be said to have a direction
of 40 degrees north of west (meaning a vector pointing west has been rotated 40 degrees
towards the northerly direction) of 65 degrees east of south (meaning a vector pointing
south has been rotated 65 degrees towards the easterly direction).

2. The direction of a vector is often expressed as a counter-clockwise angle of rotation of the


vector about its “tail” from due east. Using this convention, a vector with a direction of
30 degrees is a vector that has been rotated 30 degrees in a counter-clockwise direction
relative to due east. A vector with a direction of 160 degrees is a vector that has been
rotated 160 degrees in a counter-clockwise direction relative to due east. A vector with a
direction of 270 degrees is a vector that has been rotated 270 degrees in a
counter-clockwise direction relative to due east. This is one of the most common
conventions for the direction of a vector and will be utilized throughout this unit.

Two illustrations of the second convention (discussed above) for identifying the direction of a
vector are shown opposite.

2-52 Total Training Support Ltd


Issue 2 - September 2016 ©Copyright 2016
Module 2.2 Mechanics
Module 2,2 Mechanics

Scale: 1 cm = 4 m

Conventions for
describing directions of
vectors in quadrants

40° counter-clockwise 40° counter-clockwise


rotation from east rotation from east

Total Training Support Ltd 2-53


©Copyright 2016 Module 2.2 Mechanics Issue 2 - September 2016
Module 2.2 Mechanics
Observe in the first example that the vector is said to have a direction of 40 degrees. You can
think of this direction as follows: suppose a vector pointing east had its tail pinned down and
then the vector was rotated an angle of 40 degrees in the counter-clockwise direction.

Observe in the second example that the vector is said to have a direction of 240 degrees. This
means that the tail of the vector was pinned down and the vector was rotated an angle of
240 degrees in the counter-clockwise direction beginning from due east. A rotation of
240 degrees is equivalent to rotating the vector through two quadrants (180 degrees) and then
an additional 60 degrees into the third quadrant.

Representing the magnitude of a vector


The magnitude of a vector in a scaled vector diagram is depicted by the length of the arrow. The
arrow is drawn a precise length in accordance with a chosen scale. For example, the diagram
opposite shows a vector with a magnitude of 20 miles. Since the scale used for constructing the
diagram is 1 cm = 5 miles, the vector arrow is drawn with a length of 4 cm. That is,
4 cm x (5 miles/1 cm) = 20 miles.

Using the same scale (1 cm = 5 miles), a displacement vector that is 15 miles will be
represented by a vector arrow that is 3 cm in length. Similarly, a 25-mile displacement vector is
represented by a 5-cm long vector arrow. And finally, an 18-mile displacement vector is
represented by a 3.6-cm long arrow. See the examples shown opposite.

In conclusion, vectors can be represented by use of a scaled vector diagram. On such a


diagram, a vector arrow is drawn to represent the vector. The arrow has an obvious tail and
arrowhead.

The magnitude of a vector is represented by the length of the arrow. A scale is indicated (such
as, 1 cm = 5 miles) and the arrow is drawn the proper length according to the chosen scale. The
arrow points in the precise direction. Directions are described by the use of some convention.
The most common convention is that the direction of a vector is the counter-clockwise angle of
rotation which that vector makes with respect to due east.

Vector addition
A variety of mathematical operations can be performed with and upon vectors. One such
operation is the addition of vectors. Two vectors can be added together to determine the result
(or resultant). Observe the summations opposite, of two force vectors.

The rules for summing vectors are applied to free-body diagrams in order to determine the net
force (i.e., the vector sum of all the individual forces).

Intro to vectors & scalars I One-dimensional motion I Physics I Khan Academy


https://youtu.be/ihNZIp7iUHE

2-54 Total Training Support Ltd


Issue 2 - September 2016
Module 2.2 Mechanics ©Copyright 2016
Modiste 2.2 Mechanics

A vector drawn to Three vectors drawn to scale


scale

Fnet is 400 N, up Fnei is 200 N, down Fnet is 20 N, left

Ftons* 1,200 N
Fnorm = 600 N
Fair = 600 N
Ffriet - 20 N

Fgrav ~ 800 N Fgrav = 800 N F grav “ 800 N

Free body diagrams

10

-5
= o

Free body diagrams - adding and subtracting vectors

Total Training Support Ltd 2-55


©Copyright 2016 Module 2.2 Mechanics Issue 2 - September 2016
Module 2.2 Mechanics
The task of summing vectors will be extended to more complicated cases in which the vectors
are directed in directions other than purely vertical and horizontal directions. For example, a
vector directed up and to the right will be added to a vector directed up and to the left. The
vector sum will be determined for the more complicated cases shown in the diagrams opposite.

There are a variety of methods for determining the magnitude and direction of the result of
adding two or more vectors. The two methods that will be discussed:

• the Pythagorean theorem and trigonometric methods


• the head-to-tail method using a scaled vector diagram

The Pythagorean theorem is a useful method for determining the result of adding two (and only
two) vectors that make a right angle to each other. The method is not applicable for adding
more than two vectors or for adding vectors that are not at 90-degrees to each other. The
Pythagorean theorem is a mathematical equation that relates the length of the sides of a right
triangle to the length of the hypotenuse of a right triangle.

To see how the method works, consider the following.

Example:

John leaves the base camp and hikes 11 km, north and then hikes 11 km east. Determine
John’s resulting displacement.

This problem is to determine the result of adding two displacement vectors that are at right
angles to each other. The result (or resultant) of walking 11 km north and 11 km east is a vector
directed northeast as shown in the diagram opposite.

Since the northward displacement and the eastward displacement are at right angles to each
other, the Pythagorean theorem can be used to determine the resultant (i.e., the hypotenuse of
the right triangle).

The result of adding 11 km, north plus 11 km, east is a vector with a magnitude of 15.6 km.
Later, the method of determining the direction of the vector will be discussed.

Examples:

In each examples A and B opposite, use the Pythagorean theorem to determine the magnitude
of the vector sum.

Example A:

R2 = (5)2 + (10)2
R2 = 125
R = <(T55)
R = 11.2 km

Example B:

R2 = (30)2 + (40)2
R2 = 2,500
R = V (2,500)
R = 50 km

2-56 Total Training Support Ltd


Issue 2 - September 2016
Module 2.2 Mechanics ©Copyright 2016
ilodute 2.2 Mechanics
Visualizing vectors in 2 dimensions | Two-dimensional motion 1 Physics i Khan Academy
https://youtu.be/xp6ibul8UuQ

???
Pythagorean Theorem

???

???
a
Complicated vector additions

Example A Examnle B

10 km, north 30 km, west


+
5 km, west 40 km, south

242 = R2
15.6 = R

Using Pythagorean theorem

Total Training Support Ltd 2-57


© Copyright 2016 Module 2.2 Mechanics Issue 2 - September 2016
HR1 Module 2.2 Mechanics
Using trigonometry to determine a vector’s direction
The direction of a resultant vector can often be determined by use of trigonometric functions.
Most students recall the meaning of the useful mnemonic SOH CAH TOA from module 1.
SOH CAH TOA is a mnemonic that helps one remember the meaning of the three common
trigonometric functions - sine, cosine, and tangent functions.

These three functions relate an acute angle in a right triangle to the ratio of the lengths of two of
the sides of the right triangle. The sine function relates the measure of an acute angle to the
ratio of the length of the side opposite the angle to the length of the hypotenuse. The cosine
function relates the measure of an acute angle to the ratio of the length of the side adjacent the
angle to the length of the hypotenuse.

The tangent function relates the measure of an angle to the ratio of the length of the side
opposite the angle to the length of the side adjacent to the angle. The three equations opposite
summarize these three functions in equation form.

These three trigonometric functions can be applied to the hiker problem in order to determine
the direction of the hiker’s overall displacement.

The process begins by the selection of one of the two angles (other than the right angle) of the
triangle. Once the angle is selected, any of the three functions can be used to find the measure
of the angle. Write the function and proceed with the proper algebraic steps to solve for the
measure of the angle. The work is shown opposite.

Once the measure of the angle is determined, the direction of the vector can be found. In this
case the vector makes an angle of 4 degrees with due east. Thus, the direction of this vector is
written as 4 degrees. (The direction of a vector is the counter-clockwise angle of rotation that
the vector makes with due east.)

The calculated angle is not always the direction


The measure of an angle as determined through use of SOH CAH TOA is not always the
direction of the vector. The following vector addition diagram is an example of such a situation.
Observe that the angle within the triangle is determined to be 26. degrees using
SOH CAH TOA. This angle is the southward angle of rotation that the vector R makes with
respect to west. Yet the direction of the vector as expressed with the CCW (counter-clockwise
from east) convention is 206.6 degrees.

2-58 Total Training Support Ltd


Issue 2 - September 2016 © Copyright 2016
Module 2.2 Mechanics
Jiodufe 2.2 SVteehanscs

a = side adjacent to angle 0


b = side opposite to angle 0
a c = hypotenuse of triangle

xai co
sin 0 = “I cos 0 = c tan 0

Trigonometrical ratios

sinO=TLLp =0.7051
15.6 km
0 = sin1(7051) = 45°

Using trigonometrical ratios

10 km, west * 5 km, south

10
tan 0 = 5 km _ q $
10 km
0 = tan-1(0.5) = 26.6°

Direction of R is 180° + 26.6°

The calculated angle is not always the direction

Total Training Support Ltd 2-59


© Copyright 2016 Module 2.2 Mechanics Issue 2 - September 2016
Module 2.2 Mechanics
Examples:

In examples A and B below, use SOH CAH TOA to determine the direction of the resultant.

Example A:

tan(Theta) = (5/10) = 0.5


Theta - tan’1 (0.5)
Theta = 26.6 degrees
Direction of R = 90 degrees + 26.6 degrees
Direction of R = 116.6 degrees

Example B:

tan(Theta) = (40/30) = 1.333


Theta = tarn1 (1.333)
Theta = 53.1 degrees
Direction of R = 180 degrees + 53.1 degrees
Direction of R = 233.1 degrees

2-60 Total Training Support Ltd


Issue 2 - September 2016 © Copyright 2016
Module 2.2 Mechanics
Module 2.2 Mechanics

Example A txampie B

10 km, north 30 km, west


+
5 km, west 40 km, south

Totai Training Support Ltd 2-61


©Copyright 2016 Module 2.2 Mechanics issue 2 - September 2016
Module 2.2 Mechanics
Head-to-tail method
The magnitude and direction of the sum of two or more vectors can also be determined by use
of an accurately drawn scaled vector diagram. Using a scaled diagram, the head-to-tail
method is employed to determine the vector sum or resultant.

Either using centimetre-sized displacements upon a map or metre-sized displacements in a


large open area, a student makes several consecutive displacements beginning from a
designated starting position.

Suppose that you were given a map of your local area and a set of 18 directions to follow.
Starting at home base, these 18 displacement vectors could be added together in consecutive
fashion to determine the result of adding the set of 18 directions. Perhaps the first vector is
measured 5 cm, east. Where this measurement ended, the next measurement would begin.
The process would be repeated for all 18 directions. Each time one measurement ended, the
next measurement would begin. In essence, you would be using the head-to-tail method of
vector addition.

The head-to-tail method involves drawing a vector to scale on a sheet of paper beginning at a
designated starting position. Where the head of this first vector ends, the tail of the second
vector begins (thus, head-to-tail method). The process is repeated for all vectors that are being
added. Once all the vectors have been added head-to-tail, the resultant is then drawn from the
tail of the first vector to the head of the last vector; i.e., from start to finish. Once the resultant is
drawn, its length can be measured and converted to real units using the given scale. The
direction of the resultant can be determined by using a protractor and measuring its
counter-clockwise angle of rotation from due east.

A step-by-step method for applying the head-to-tail method to determine the sum of two or more
vectors is given below.

1. Choose a scale and indicate it on a sheet of paper. The best choice of scale is one that will
result in a diagram that is as large as possible, yet fits on the sheet of paper.

2. Pick a starting location and draw the first vector to scale in the indicated direction. Label the
magnitude and direction of the scale on the diagram (e.g., scale: 1 cm = 20 m).

3. Starting from where the head of the first vector ends, draw the second vector to scale in the
indicated direction. Label the magnitude and direction of this vector on the diagram.

4. Repeat steps 2 and 3 for all vectors that are to be added.

5. Draw the resultant from the tail of the first vector to the head of the last vector. Label this
vector as resultant or simply R.

6. Using a ruler, measure the length of the resultant and determine its magnitude by
converting to real units using the scale (4.4 cm x 20 m/1 cm = 88 m).

7. Measure the direction of the resultant using the counter-clockwise convention discussed
earlier in this lesson.

An example of the use of the head-to-tail method is illustrated opposite. The problem involves
the addition of three vectors:

The head-to-tail method is employed as described above and the resultant is determined
(drawn in red). Its magnitude and direction is labelled on the diagram.
2-62 Total Training Support Ltd
Issue 2 - September 2016 © Copyright 2016
Module 2.2 Mechanics
Module 2.2 Mechanics

Home base
§
Scale -..
8
j 3 cm
9 „
j 5cm
! 8 cm
Simple ‘head-to-tail’
method example

•••

300°

Three vectors

210°

Three vectors arranged ‘head-to-tail’ to find the resultant

Total Training Support Ltd 2-63


©Copyright 2016 Moduie 2.2 Mechanics Issue 2 - September 2016
Kip/ Module 2.2 Mechanics
interestingly enough, the order in which three vectors are added has no effect upon either the
magnitude or the direction of the resultant. The resultant will still have the same magnitude and
direction.

For example, consider the addition of the same three vectors in a different order.

15 m, 210 degrees + 25 m, 300 degrees + 20 m, 45 degrees

When added together in this different order, these same three vectors still produce a resultant
with the same magnitude and direction as before (20.3 m, 312 degrees). The order in which
vectors are added using the head-to-tail method is insignificant.

Resultants
The resultant is the vector sum of two or more vectors. It is the result of adding two or more
vectors together. If displacement vectors A, B, and C are added together, the result will be
vector R. As shown in the diagram, vector R can be determined by the use of an accurately
drawn, scaled, vector addition diagram.

To say that vector R is the resultant displacement of displacement vectors A, B, and C is to say
that a person who walked with displacements A, then B, and then C would be displaced by the
same amount as a person who walked with displacement R. Displacement vector R gives the
same result as displacement vectors A + B + C. That is why it can be said that

A+B+C~ R

The above discussion pertains to the result of adding displacement vectors. When displacement
vectors are added, the result is a resultant displacement. But any two vectors can be added as
long as they are the same vector quantity. If two or more velocity vectors are added, then the
result is a resultant velocity.

If two or more force vectors are added, then the result is a resultant force, if two or more
momentum vectors are added, then the result is resultant momentum.

In all such cases, the resultant vector (whether a displacement vector, force vector, velocity
vector, etc.) is the result of adding the individual vectors.

It is the same thing as adding A + B + C + ... .

“To do A + B + C is the same as to do R.”

2-64 Total Training Support Ltd


Issue 2 - September 2016
Module 2.2 Mechanics © Copyright 2016
Modute 2.2 Mechanics

Three vectors
15 m, 210 deg. + 25 m, 300 deg. + 20 m, 45 deg.

Three vectors arranged


‘head-to-tail’ to find the resultant

Three vectors arranged


‘head-to-tail’ to find the resultant

Total Training Support Ltd 2-65


© Copyright 2016 Module 2.2 Mechanics Issue 2 - September 2016
»*' ■
Module 2.2 Mechanics
Example:

Consider a rugby player who gets hit simultaneously by three players on the opposing team
(players A, B, and C). The rugby player experiences three different applied forces. Each applied
force contributes to a total or resulting force. If the three forces are added together using
methods of vector addition (discussed earlier), then the resultant vector R can be determined. In
this case, to experience the three forces A, B and C is the same as experiencing force R.

To be hit by players A, B, and C would result in the same force as being hit by one player
applying force R. “To do A T B + C is the same as to do R.”

Vector R is the same result as vectors A + B + C.

in summary, the resultant is the vector sum off all the individual vectors. The resultant is
the result of combining the individual vectors together. The resultant can be determined
by adding the individual forces together using vector addition methods.

2-66 Total Training Support Ltd


Issue 2 - September 2016 © Copyright 2016
Module 2.2 Mechanics
SVSodufe 2.2 Mechanics

Three force vectors acting


on a rugby player

The three vectors arranged


‘head-to-tail’ to find the
resultant force and direction

Total Training Support Ltd 2-67


© Copyright 2016 Module 2.2 Mechanics Issue 2 - September 2016
Module 2.2 Mechanics
Vector components
A vector is a quantity that has both magnitude and direction. Displacement, velocity,
acceleration, and force are examples of the vector quantities

In situations in which vectors are directed at angles to the customary coordinate axes, a useful
mathematical trick will be employed to transform the vector into two parts with each part being
directed along the coordinate axes.

For example, a vector that is directed northwest can be thought of as having two parts - a
northward part and a westward part. A vector that is directed upward and rightward can be
thought of as having two parts - an upward part and a rightward part.

Any vector directed in two dimensions can be thought of as having an influence in two different
directions. That is, it can be thought of as having two parts. Each part of a two-dimensional
vector is known as a component. The components of a vector depict the influence of that vector
in a given direction. The combined influence of the two components is equivalent to the
influence of the single two-dimensional vector. The single two-dimensional vector could be
replaced by the two components.

Angled vectors have two components


If a dog’s chain is stretched upward and rightward and pulled tight by his master, then the
tension force in the chain has two components - an upward component and a rightward
component. To the dog, the influence of the chain on his body is equivalent to the influence of
two chains on his body - one pulling upward and the other pulling rightward. If the single chain
were replaced by two chains, with each chain having the magnitude and direction of the
components, then the dog would not know the difference. This is not because the dog is dumb,
but rather because the combined influence of the two components is equivalent to the influence
of the single two-dimensional vector.

2-68 Total Training Support Ltd


Issue 2 - September 2016 © Copyright 2016
Module 2.2 Mechanics
Module 2.2 Mechanics

+ ... ... . . .mm

Northwest vectors have a northward and a westward part.

An upward and rightward vector has an upward and rightrward part.

The upward and rightward force of the chain is equivalent


to an upward force and a rightward force by two chains

Total Training Support Ltd 2-69


© Copyright 2016 Module 2.2 Mechanics Issue 2 - September 2016
Module 2.2 Mechanics
Consider an aeroplane that is flying from London Heathrow Airport to Glasgow. Suppose that
the aeroplane is flying in such a manner that its resulting displacement vector is northwest. If
this is the case, then the displacement of the aeroplane has two components - a component in
the northward direction and a component in the westward direction.

This is to say that the aeroplane would have the same displacement if it were to take the trip to
Glasgow in two segments - one directed due north and the other directed due west. If the single
displacement vector were replaced by these two individual displacement vectors, then the
passengers in the aeroplane would end up in the same final position. The combined influence of
the two components is equivalent to the influence of the single two-dimensional displacement.

Any vector directed in two dimensions can be thought of as having two different components.
The component of a single vector describes the influence of that vector in a given direction. This
is called vector resolution.

Vector resolution
Any vector directed at an angle to the horizontal (or the vertical) can be thought of as having
two parts. That is, any vector directed in two dimensions can be thought of as having two
components.

For example, if a chain pulls upward at an angle on the collar of a dog, then there is a tension
force directed in two dimensions. This tension force has two components: an upward
component and a rightward component. As another example, consider an aeroplane that is
displaced northwest from Heathrow Airport (in London) to a Glasgow. The displacement vector
of the aeroplane is in two dimensions (northwest). Thus, this displacement vector has two
components: a northward component and a westward component.

There are two basic methods for determining the magnitudes of the components of a vector
directed in two dimensions. The process of determining the magnitude of a vector is known as
vector resolution. The two methods of vector resolution that we will examine are

♦ the parallelogram method


• the trigonometric method

2-70 Total Training Support Ltd


issue 2 - September 2016 ©Copyright 2016
Module 2.2 Mechanics
Module 2.2 Mechanics

An aeroplane journey from London Heathrow Airport to Glasgow can


be resolved into a northward component and a westward component

Total Training Support Ltd 2-71


©Copyright 2016 Module 2.2 Mechanics Issue 2 - September 2016
Module 2.2 Mechanics
Parallelogram method of vector resolution
The parallelogram method of vector resolution involves using an accurately drawn, scaled
vector diagram to determine the components of the vector. Briefly put, the method involves
drawing the vector to scale in the indicated direction, sketching a parallelogram around the
vector such that the vector is the diagonal of the parallelogram, and determining the magnitude
of the components (the sides of the parallelogram) using the scale. If one desires to determine
the components as directed along the traditional x- and y-coordinate axes, then the
parallelogram is a rectangle with sides that stretch vertically and horizontally.

A step-by-step procedure for using the parallelogram method of vector resolution is:

1. Select a scale and accurately draw the vector to scale in the indicated direction.

2. Sketch a parallelogram around the vector: beginning at the tail of the vector, sketch vertical
and horizontal lines; then sketch horizontal and vertical lines at the head of the vector; the
sketched lines will meet to form a rectangle (a special case of a parallelogram).

3. Draw the components of the vector. The components are the sides of the parallelogram.
The tail of the components start at the tail of the vector and stretches along the axes to the
nearest corner of the parallelogram. Be sure to place arrowheads on these components to
indicate their direction (up, down, left, right).

4. Meaningfully label the components of the vectors with symbols to indicate which
component represents which side. A northward force component might be labelled Fnorth. A
rightward velocity component might be labelled Vx; etc.

5. Measure the length of the sides of the parallelogram and use the scale to determine the
magnitude of the components in real units. Label the magnitude on the diagram.

The step-by-step procedure above is illustrated in the diagram opposite to show how a velocity
vector with a magnitude of 50 m/s and a direction of 60 degrees above the horizontal may be
resolved into two components.

The diagram shows that the vector is first drawn to scale in the indicated direction; a
parallelogram is sketched about the vector; the components are labelled on the diagram; and
the result of measuring the length of the vector components and converting to m/s using the
scale.

2-72 Total Training Support Ltd


Issue 2 - September 2016 ©Copyright 2016
Module 2.2 Mechanics
Module 2.2 Mechanics

Scale; 1cm = 10 m/s

Parallelogram method of vector resolution

Total Training Support Ltd 2-73


©Copyright 2016 Module 2.2 Mechanics Issue 2 ~ September 2016
Module 2.2 Mechanics
Trigonometric method of vector resolution
The trigonometric method of vector resolution involves using trigonometric functions to
determine the components of the vector. Earlier, the use of trigonometric functions to determine
the direction of a vector was described. Now, trigonometric functions will be used to determine
the components of a single vector. Recall from the earlier discussion that trigonometric functions
relate the ratio of the lengths of the sides of a right triangle to the measure of an acute angle
within the right triangle. As such, trigonometric functions can be used to determine the length of
the sides of a right triangle if an angle measure and the length of one side are known.

The method of employing trigonometric functions to determine the components of a vector are
as follows:

1. Construct a rough sketch (no scale needed) of the vector in the indicated direction. Label
its magnitude and the angle that it makes with the horizontal.

2. Draw a rectangle about the vector such that the vector is the diagonal of the rectangle.
Beginning at the tail of the vector, sketch vertical and horizontal lines. Then sketch
horizontal and vertical lines at the head of the vector. The sketched lines will meet to form a
rectangle.

3. Draw the components of the vector. The components are the sides of the rectangle. The
tail of each component begins at the tail of the vector and stretches along the axes to the
nearest corner of the rectangle. Be sure to place arrowheads on these components to
indicate their direction (up, down, left, right).

4. Meaningfully label the components of the vectors with symbols to indicate which
component represents which side. A northward force component might be labelled Fnorth. A
rightward force velocity component might be labelled vx; etc.

5. To determine the length of the side opposite the indicated angle, use the sine function.
Substitute the magnitude of the vector for the length of the hypotenuse. Use some algebra
to solve the equation for the length of the side opposite the indicated angle.

6. Repeat the above step using the cosine function to determine the length of the side
adjacent to the indicated angle.

The above method is illustrated opposite for determining the components of the force acting
upon the dog. As the 60-Newton tension force acts upward and rightward on the dog at an
angle of 40 degrees, the components of this force can be determined using trigonometric
functions.

In conclusion, a vector directed in two dimensions has two components - that is, an influence in
two separate directions. The amount of influence in a given direction can be determined using
methods of vector resolution. Two methods of vector resolution have been described here ~ a
graphical method (parallelogram method) and a trigonometric method.

2-74 Total Training Support Ltd


Issue 2 - September 2016 ©Copyright 2016
Module 2.2 Mechanics
Module 2.2 Mechanics

Fhoriz
sin 40° _" 60N
vert COS 40°
SON
Fvert = 60 N x sin 40° Fhoriz ~ 60 N x cos 40°

Fvert = 38.6 N Fhoriz = 45.9 N

Trigonometric method of vector resolution

Total Training Support Ltd 2-75


©Copyright 2016 Module 2.2 Mechanics Issue 2 - September 2016
Module 2.2 Mechanics
Component method of vector addition
Earlier, you learned that vectors oriented at right angles to one another can be added together
using the Pythagorean theorem. For instance, two displacement vectors with magnitude and
direction of 11 km, north and 11 km, east can be added together to produce a resultant vector
that is directed both north and east. When the two vectors are added head-to-tail as shown
opposite, the resultant is the hypotenuse of a right triangle. The sides of the right triangle have
lengths of 11 km and 11 km. The resultant can be determined using the Pythagorean theorem;
it has a magnitude of 15.6 km. The solution is shown below the diagram.

This Pythagorean approach is a useful approach for adding any two vectors that are directed at
right angles to one another. A right triangle has two sides plus a hypotenuse; so the
Pythagorean theorem is perfect for adding two right angle vectors. But there are limits to the
usefulness of the Pythagorean theorem in solving vector addition problems.

For instance, the addition of three or four vectors does not lead to the formation of a right
triangle with two sides and a hypotenuse. So at first glance it may seem that it is impossible to
use the Pythagorean theorem to determine the resultant for the addition of three or four vectors.
Furthermore, the Pythagorean theorem works when the two added vectors are at right angles to
one another - such as for adding a north vector and an east vector. But what can one do if the
two vectors that are being added are not at right angles to one another? Is there a means of
using mathematics to reliably determine the resultant for such vector addition situations? Or is
the student of physics left to determining such resultants using a scaled vector diagram? Here,
we will learn how to approach more complex vector addition situations by combining the
concept of vector components and the principles of vector resolution with the use of the
Pythagorean theorem.

2-76 Total Training Support Ltd


Issue 2 - September 2016 © Copyright 2016
Module 2.2 Mechanics
Modute 2.2 Mechanics

ik

11 km, E
11 km, N + -------- -—

-| 2 4. -j -p — p2 Vectors oriented at right


242 = R2 angles to one another can
15.6 ~ R be added together using
the Pythagorean theorem

Total Training Support Ltd 2-77


© Copyright 2016 Module 2.2 Mechanics Issue 2 - September 2016
Module 2.2 Mechanics
Addition of three or more right-angle vectors
As our first example, consider the following vector addition problem:

Example 1:

A student drives his car 6.0 km, north before making a right hand turn and driving 6.0 km to the
east. Finally, the student makes a left hand turn and travels another 2.0 km to the north. What is
the magnitude of the overall displacement of the student?

Like any problem in physics, a successful solution begins with the development of a mental
picture of the situation. The construction of a diagram like that shown opposite often proves
useful in the visualization process.

When these three vectors are added together in head-to-tail fashion, the resultant is a vector
that extends from the tail of the first vector (6.0 km, north, shown in red) to the arrowhead of the
third vector (2.0 km, north, shown in green). The head-to-tail vector addition diagram is shown
opposite.

As can be seen in the diagram, the resultant vector (drawn in black) is not the hypotenuse of
any right triangle - at least not of any immediately obvious right triangle. But would it be possible
to force this resultant vector to be the hypotenuse of a right triangle? The answer is yes! To do
so, the order in which the three vectors are added must be changed. The vectors above were
drawn in the order in which they were driven.

The student drove north, then east, and then north again.

But if the three vectors are added in the order 6.0 km, N + 2.0 km, N + 6.0 km, E, then the
diagram will look like that shown opposite.

After rearranging the order in which the three vectors are added, the resultant vector is now the
hypotenuse of a right triangle. The lengths of the perpendicular sides of the right triangle are
8.0 m, north (6.0 km + 2.0 km) and 6.0 km, east. The magnitude of the resultant vector (R) can
be determined using the Pythagorean theorem.

R2 = (8.0 km)2 + (6.0 km)2


R2 = 64.0 + 36.0
R2= 100.0_
R = a/(100.0)
R= 10.0 km

In the first vector addition diagram above, the three vectors were added in the order in which
they are driven. In the second vector addition diagram above, the order in which the vectors
were added was switched around. The size of the resultant was not affected by this change in
order.

This illustrates an important point about adding vectors: the resultant is independent by the
order in which they are added. Adding vectors A + B + C gives the same resultant as adding
vectors B + A + C or even C + B + A. As long as all three vectors are included with their
specified magnitude and direction, the resultant will be the same. This property of vectors is the
key to the strategy used in the determination of the answer to the above example problem.

2-78 Total Training Support Ltd


Issue 2 - September 2016 ©Copyright 2016
Module 2.2 Mechanics
Module 2.2 Mechanics

6.0 km

Total Training Support Ltd 2-79


©Copyright 2016 Module 2.2 Mechanics Issue 2 - September 2016
Module 2.2 Mechanics
Example 2:

A man walks around a room. Starting at the door he walks 2.0 metres, south. He makes a right
hand turn and walk 16.0 metres, west. He turns right again and walks 24.0 metres, north. He
then turns left and walk 36.0 metres, west. What is the magnitude of his overall displacement?

A graphical representation of the given problem will help visualize what is happening. The
diagram opposite depicts such a representation.

When these four vectors are added together in head-to-tail fashion, the resultant is a vector that
extends from the tail of the first vector (2.0 m, south, shown in red) to the arrowhead of the
fourth vector (36.0 m, west, shown in green). The head-to-tail vector addition diagram is shown
opposite.

The resultant vector (drawn in black and labelled R) in the vector addition diagram above is not
the hypotenuse of any immediately obvious right triangle. But by changing the order of addition
of these four vectors, one can force this resultant vector to be the hypotenuse of a right triangle.
For instance, by adding the vectors in the order of 2.0 m, S + 24.0 m, N + 16.0 m, W + 36.0 m.
W, the resultant becomes the hypotenuse of a right triangle. This is shown in the vector addition
diagram opposite.

With the vectors rearranged, the resultant is now the hypotenuse of a right triangle that has two
perpendicular sides with lengths of 22.0 m, north and 52.0 m, west. The 22.0 m, north side is
the result of 2.0 m, south and 24.0 m, north added together. The 52.0 m, west side is the result
of 16.0 m, west and 36.0 m, west added together. The magnitude of the resultant vector (R) can
be determined using the Pythagorean theorem.

R2 = (22.0 m)2 + (52.0 m)2


R2 = 484 o + 2,704.0
R2 = 3,188.0
R = a/(3,188.0)
R = 56.5 m

As can be seen in these two examples, the resultant of the addition of three or more right angle
vectors can be easily determined using the Pythagorean theorem. Doing so involves the adding
of the vectors in a different order.

2-80 Total Training Support Ltd


Issue 2 - September 2016 ©Copyright 2016
Module 2.2 Mechanics
Module 2.2 Mechanics

t
♦ + + 229

Example 2

Total Training Support Ltd 2-81


© Copyright 2016 Module 2.2 Mechanics Issue 2 - September 2016
Module 2.2 Mechanics
SOH CAH TOA and the direction of vectors
The above discussion explains the method for determining the magnitude of the resultant for
three or more perpendicular vectors. The topic of direction has been ignored in the discussion.
Now we will turn our attention to the method for determining the direction of the resultant vector.
As a quick review, recall that earlier in this lesson a convention for expressing the direction of a
vector was introduced. The convention is known as the counter-clockwise from east convention,
often abbreviated as the CCW convention. Using this convention, the direction of a vector is
often expressed as a counter-clockwise angle of rotation of the vector about its tail from due
east.

To begin our discussion, let us return to Example 1 above where we made an effort to add
three vectors:

6.0 km, N + 6.0 km, E + 2.0 km, N.

In the solution, the order of addition of the three vectors was rearranged so that a right triangle
was formed with the resultant being the hypotenuse of the triangle.

The triangle is redrawn opposite. Observe that the angle in the lower left of the triangle has
been labelled as theta (6). Theta (0) represents the angle that the vector makes with the north
axis. Theta (0) can be calculated using one of the three trigonometric functions introduced
earlier in this lesson - sine, cosine or tangent. The mnemonic SOH CAH TOA is a helpful way of
remembering which function to use. In this problem, we wish to determine the angle measure of
theta (0) and we know the length of the side opposite theta (0) - 6.0 km - and the length of the
side adjacent the angle theta (0) - 8.0 km.

The TOA of SOH CAH TOA indicates that the tangent of any angle is the ratio of the lengths of
the side opposite to the side adjacent that angle. Thus, the tangent function will be used to
calculate the angle measure of theta (0).

The work is shown below.

Tangent(0) = Opposite/Adjacent
Tangent(O) = 6.0/8.0
Tangent(0) = 0.75
0 = tan’1 (0.75)
0 = 36.869...
0 =37°

The problem is not over once the value of theta (0) has been calculated. This angle measure
must now be used to state the direction. One means of doing so is to simply state that the
direction of the resultant is 37° east of north. Alternatively, the counter-clockwise convention
could be used. Since the angle that the resultant makes with east is the complement of the
angle that it makes with north, we could express the direction as 53° CCW.

We will now consider Example 2 as a second example of how to use SOH CAH TOA to
determine the direction of a resultant. In Example 2, we were trying to determine the magnitude
of 2.0 m. S + 16.0 m, W + 24.0 m, N + 36.0 m, W. The solution involved re-ordering the addition
so that the resultant was the hypotenuse of a right triangle with known sides. The right triangle is
shown opposite. The resultant is drawn in black. Once more, observe that the angle in the lower
right of the triangle has been labelled as theta (0). Theta (0) represents the angle that the vector
makes with the north axis.

2-82 Total Training Support Ltd


Issue 2 - September 2016
Module 2.2 Mechanics ©Copyright 2016
Module 2.2 Mechanics
Theta (0) can be calculated using the tangent function. In this problem, we wish to determine
the angle measure of theta (0) and we know the length of the side opposite theta (0) - 52.0 m -
and the length of the side adjacent the angle theta (0) - 22.0 m. The TOA of SOH CAH TOA
indicates that the tangent of any angle is the ratio of the lengths of the side opposite to the side
adjacent that angle. Thus, the tangent function will be used to calculate the angle measure of
theta (0). The work is shown below.

Example 1

24.0 km, E

2.0 km, N

Total Training Support Ltd 2-83


©Copyright 2016 Module 2.2 Mechanics Issue 2 - September 2016
Module 2.2 Mechanics
Tangent(0) = Opposite/Adjacent
Tangent(0) = 52,0/22.0
Tangent(O) = 2.3636 ...
e=tarr1 (2.3636 ...)
9 = 67.067 ...
0 = 67.1°

The problem is not over once the value of theta (8) has been calculated. This angle measure
must now be used to state the direction.

One means of doing so is to simply state that the direction of the resultant is 67.1 ° west of north.
Alternatively, the counter-clockwise convention could be used. The north axis is rotated 90°
counter-clockwise from east and this vector is an additional 67,1° counter-clockwise past north.

Thus the CCW direction is 157.1° CCW.

In summary, the direction of a vector can be determined in the same way that it is always
determined - by finding the angle of rotation counter-clockwise from due east. Since the
resultant is the hypotenuse of a right triangle, this can be accomplished by first finding an angle
that the resultant makes with one of the nearby axes of the triangle.

Once done, a little thinking is required in order to relate the angle to a direction.

Addition of non-perpendicular vectors


Now we will consider situations in which the two (or more) vectors that are being added are not
at right angles to each other. The Pythagorean theorem is not applicable to such situations
since it applies only to right triangles. Two non-perpendicular vectors will not form a right
triangle. Yet it is possible to force two (or more) non-perpendicular vectors to be transformed
into other vectors that do form a right triangle. The trick involves the concept of a vector
component and the process of vector resolution.

A vector component describes the effect of a vector in a given direction. Any angled vector has
two components; one is directed horizontally and the other is directed vertically. For instance, a
northwest vector has a northward component and a westward component. Together, the effect
these two components are equal to the overall effect of the angled vector.

As an example, consider an aeroplane that flies northwest from London Heathrow airport
towards Glasgow. The northwest displacement vector of the aeroplane has two components - a
northward component and a westward component. When added together, these two
components are equal to the overall northwest displacement. This is shown in the diagram
opposite.

The northwest vector has north and west components that are represented as Ax and Ay. It can
be said that

A = Ax + Ay

So whenever we think of a northwest vector, we can think instead of two vectors - a north and a
west vector. The two components Ax + Ay can be substituted in for the single vector A in the
problem.

2-84 Total Training Support Ltd


Issue 2 - September 2016 ©Copyright 2016
Module 2.2 Mechanics
Module 2.2 Mechanics

An aeroplane journey from London Heathrow Airport to Glasgow can


be resolved into a northward component and a westward component

Total Training Support Ltd 2-85


© Copyright 2016 Module 2.2 Mechanics issue 2 - September 2016
Module 2.2 Mechanics
Now suppose that your task involves adding two non-perpendicular vectors together. We will
call the vectors A and B. Vector A is a tricky angled vector that is neither horizontal nor vertical.
And vector B is a easy vector directed horizontally. The situation is shown below.

Of course vector A has two components - Ax and Ay. These two components together are equal
to vector A.

That is, A = Ax + Ay .

And since this is true, it makes sense to say that

A 4- B = Ax + Ay + B.

And so the problem of A + B has been transformed into a problem in which all vectors are at
right angles to each other. With all vectors being at right angles to one another, their addition
leads to a resultant that is at the hypotenuse of a right triangle. The Pythagorean theorem can
then be used to determine the magnitude of the resultant.

To see how this process works with an actual vector addition problem, consider example 3
shown below.

Example 3

During a football match, a player made the following movements after the ball was kicked into
play. First, he ran in the southern direction for 2.6 metres. He then shuffled to his left (west) for a
distance of 2.2 metres. Finally, he made a half-turn and ran downfield a distance of 4.8 metres
in a direction of 240° counter-clockwise from east (30° W of S) before finally contacting the ball.
Determine the magnitude and direction of the player’s overall displacement.

As is the usual case, the solution begins with a diagram of the vectors being added.

To assist in the discussion, the three vectors have been labelled as vectors A, B, and C. The
resultant is the vector sum of these three vectors; a head-to-tail vector addition diagram reveals
that the resultant is directed southwest. Of the three vectors being added, vector C is clearly the
tricky vector. Its direction is neither due south nor due west. The solution involves resolving this
vector into its components.

The process of vector resolution was discussed earlier. The process involves using the
magnitude and the sine and cosine functions to determine the x- and y-components of the
vector. Vector C makes a 30° angle with the southern direction. By sketching a right triangle
with horizontal and vertical legs and C as the hypotenuse, it becomes possible to determine the
components of vector C. This is shown in the diagram opposite.

The side adjacent this 30° angle in the triangle is the vertical side; the vertical side represents
the vertical (southward) component of C - Cy. So to determine Cy, the cosine function is used.
The side opposite the 30° angle is the horizontal side; the horizontal side represents the
horizontal (westward) component of C - Cx. The values of Cx and Cy can be determined by
using SOH CAM TOA. The cosine function is used to determine the southward component since
the southward component is adjacent to the 30° angle. The sine function is used to determine
the westward component since the westward component is the side opposite to the 30° angle.
The work is shown opposite.

Now our vector addition problem has been transformed from the addition of two easy vectors
and one tricky vector into the addition of four easy vectors.
2-86 Total Training Support Ltd
Issue 2 - September 2016 © Copyright 2016
Module 2.2 Mechanics
Module 2.2 Mechanics
B
... . .. a ??? Using Pythagorean theorem to
determine the resultant of 3

Ax
mmiwiiiiiifi^ "t* Ay

R2 = (A<B)2 + Ay2

Any vector which is not 90 degrees (A) or parallel, can be


resolved into two components (Ax and Ay)

Cx = C*sin(30°)
Cx = 4.8*sin(30°) = 2.400... m

Gy = O cos (30°)
Cy = 4.8*cos(30°) = 4.156... m

Totai Training Support Ltd 2-87


© Copyright 2016 Module 2.2 Mechanics Issue 2 - September 2016
Module 2.2 Mechanics
With all vectors oriented along are customary north-south and east-west axes, they can be
added head-to-tail in any order to produce a right triangle whose hypotenuse is the resultant.
Such a diagram is shown opposite.

The triangle’s perpendicular sides have lengths of 4.6 metres and 6.756 metres. The length of
the horizontal side (4,6 m) was determined by adding the values of B (2.2 m) and Cx (2.4 m).
The length of the vertical side (6.756... m) was determined by adding the values of A (2.6 m)
and Cy (4.156... m). The resultant’s magnitude (R) can now be determined using the
Pythagorean theorem.

R2 = (6.756... m)2 + (4.6 m)2


R2 = 45.655 T 21.16
R2 = 66.815
R = <766:815)
R = 8.174 m
R = ~8.2 m

The direction of the resultant can be determined by finding the angle that the resultant makes
with either the north-south or the east-west vector. The diagram opposite shows the angle theta
(G) marked inside the vector addition triangle. This angle theta is the angle that the resultant
makes with west. Its value can be determined using the tangent function. The tangent function
(as in TOA) relates the angle value to the ratio of the lengths of the opposite side to the
adjacent side. That is,

tangent(0) = (6.756... m)/(4.6 m) = 1.46889...

Using the inverse tangent function, the angle theta (0) can be determined.

0 = tan'1.(1.46889...) = 55.7536... 0
0 = -56°

This 56° angle is the angle between the resultant vector (drawn in black above) and the
westward direction. This makes the direction 56° south of west. The direction of the resultant
based on the counter-clockwise from east convention (CCW) can be determined by adding 180°
to the 56°.

So the CCW direction is 236°.

2-88 Total Training Support Ltd


Issue 2 - September 2016 ©Copyright 2016
Module 2.2 Mechanics
Module 2.2 Mechanics

4,6 m

Example 3

Total Training Support Ltd 2-89


© Copyright 2016 Module 2.2 Mechanics Issue 2 - September 2016
Module 2.2 Mechanics
Relative velocity
On occasion objects move within a medium that is moving with respect to an observer. For
example, an aeroplane usuaily encounters a wind - air that is moving with respect to an
observer on the ground below.

As another example, a motorboat in a river is moving amidst a river current - water that is
moving with respect to an observer on dry land. In such instances as this, the magnitude of the
velocity of the moving object (whether it be an aeroplane or a motorboat) with respect to the
observer on land will not be the same as the speedometer reading of the vehicle. That is to say,
the speedometer on the motorboat might read 20 mph; yet the motorboat might be moving
relative to the observer on shore at a speed of 25 mph. Motion is relative to the observer. The
observer on land, often named (or misnamed) the “stationary observer” would measure the
speed to be different than that of the person on the boat. The observed speed of the boat must
always be described relative to who the observer is.

Tailwinds, headwinds, and side winds


To illustrate this principle, consider an aeroplane flying amidst a tailwind. A tailwind is merely a
wind that approaches the aeroplane from behind, thus increasing its resulting velocity. If the
aeroplane is traveling at a velocity of 100 km/hr with respect to the air, and if the wind velocity is
25 km/hr, then what is the velocity of the aeroplane relative to an observer on the ground
below? The resultant velocity of the aeroplane (that is, the result of the wind velocity
contributing to the velocity due to the aeroplane’s motor) is the vector sum of the velocity of the
aeroplane and the velocity of the wind. This resultant velocity is quite easily determined if the
wind approaches the aeroplane directly from behind. As shown in the diagram opposite, the
aeroplane travels with a resulting velocity of 125 km/hr relative to the ground.

If the aeroplane encounters a headwind, the resulting velocity will be less than 100 km/hr. Since
a headwind is a wind that approaches the aeroplane from the front, such a wind would decrease
the aeroplane’s resulting velocity.

Suppose an aeroplane traveling with a velocity of 100 km/hr with respect to the air meets a
headwind with a velocity of 25 km/hr. in this case, the resultant velocity would be 75 km/hr; this
is the velocity of the aeroplane relative to an observer on the ground. This is depicted in the
diagram opposite.

Now consider an aeroplane traveling with a velocity of 100 km/hr, south that encounters a side
wind of 25 km/hr, west. Now what would the resulting velocity of the aeroplane be? This
question can be answered in the same manner as the previous questions. The resulting velocity
of the aeroplane is the vector sum of the two individual velocities. To determine the resultant
velocity, the aeroplane velocity (relative to the air) must be added to the wind velocity. This is
the same procedure that was used above for the headwind and the tailwind situations; only
now, the resultant is not as easily computed. Since the two vectors to be added - the southward
aeroplane velocity and the westward wind velocity - are at right angles to each other, the
Pythagorean theorem can be used. This is illustrated in the diagram opposite.

In this situation of a side wind, the southward vector can be added to the westward vector using
the usual methods of vector addition. The magnitude of the resultant velocity is determined
using Pythagorean theorem. The algebraic steps are as follows:

(100 km/hr)2 + (25 km/hr)2 = R2


10,000 + 625 - R2
10,625 = R2
V (10,625) = R R = 103.1 km/hr
2-90 Total Training Support Ltd
Issue 2 - September 2016 © Copyright 2016
Module 2.2 Mechanics
Module 2.2 Mechanics
Plane + Wind = Resultant
velocity velocity velocity

Aeroplane with a tailwind


100 km/hr 25 km/hr 125 km/hr

The plane travels with a velocity relative to the


ground which is the vector sum of the plane’s
velocity (relative to the air) and the wind velocity

Plane + Wind ™ Resultant


velocity velocity velocity

4 I

Aeroplane with a headwind

100 km/hr 25 km/hr 75 km/hr

Plane + Wind Resultant


velocity velocity velocity

Aeroplane with a side wind

100 km/hr 25 km/hr 103.1 km/hr

Total Training Support Ltd 2-91


© Copyright 2016 Module 2.2 Mechanics Issue 2 - September 2016
Module 2.2 Mechanics
The direction of the resulting velocity can be determined using a trigonometric function. Since
the aeroplane velocity and the wind velocity form a right triangle when added together in head-
to-tail fashion, the angle between the resultant vector and the southward vector can be
determined using the sine, cosine, or tangent functions.

The tangent function can be used; this is shown below:

tan (theta) = (opposite/adjacent)


tan (theta) = (25/100)
theta = tan'1 (25/100)
theta = 14.0 degrees

If the resultant velocity of the aeroplane makes a 14.0-degree angle with the southward
direction (theta in the above diagram), then the direction of the resultant is 256 degrees.

Like any vector, the resultant’s direction is measured as a counter-clockwise angle of rotation
from due east.

2-92 Total Training Support Ltd


Issue 2 - September 2016 ©Copyright 2016
Module 2.2 Mechanics
Module 2.2 Mechanics

Using the tan function to find


the aeroplane angle

Total Training Support Ltd 2-93


©Copyright 2016 Module 2.2 Mechanics Issue 2 - September 2016
Module 2.2 Mechanics

Intentionally Blank

2-94 Total Training Support Ltd


Issue 2 - September 2016
Module 2.2 Mechanics ©Copyright 2016
Module 2.2 Mechanics igjjjp
Worksheet

1. Add the following vectors and determine the resultant.

3.0 m/s, 45 degrees and 5.0 m/s, 135 degrees

2. Add the following vectors and determine the resultant,

5.0 m/s, 45 degrees and 2.0 m/s, 180 degrees

3. Add the following vectors and determine the resultant.

2.0 m/s, 150 degrees and 4.0 m/s, 225 degrees

4. A force vector of magnitude 25 N acts in a direction of 30 degrees (counter-clockwise).


Determine the vertical and horizontal components of this vector.

5. An aeroplane flies a heading of 030 degrees at 50 mph. Determine the northerly and
easterly components of the aeroplane speed

6. An aeroplane can travel with a speed of 80 mph with respect to the air. Determine the
resultant velocity of the aeroplane (magnitude only) if it encounters a

a) 10 mph headwind.

b) 10 mph tailwind.

c) 10 mph crosswind.

d) 60 mph crosswind.

7. A motorboat traveling 5 m/s, east encounters a current traveling 2.5 m/s, north. What is the
resultant velocity of the motor boat?

Total Training Support Ltd 2-95


© Copyright 2016 Module 2.2 Mechanics Issue 2 - September 2016
Module 2.2 Mechanics

Intentionally Blank

2-96 Total Training Support Ltd


Issue 2 - September 2016
Module 2.2 Mechanics © Copyright 2016
Modute 2.2 Mechanics
Answers

1. 5.83 m/s, 104 deg. Draw vectors to scale:

2. 3.85m/s, 66.5 deg. Draw vectors to scale:

Total Training Support Ltd 2-97


©Copyright 2016 Module 2.2 Mechanics Issue 2 - September 2016
Module 2.2 Mechanics
4. Horizontal component = 21.7 N,
Vertical component = 12.5 N

5. Northerly component = 25 mph


Easterly component = 43.3 mph

6. a) A headwind would decrease the resultant velocity of the aeroplane to 70 mph.

b) A tailwind would increase the resultant velocity of the aeroplane to 90 mph.

c) A 10 mph crosswind would increase the resultant velocity of the aeroplane to 80.6 mph.

Using the Pythagorean theorem:


[ (80 mph)2 + (10 mph)2 ]

d) A 60 mph crosswind would increase the resultant velocity of the aeroplane to 100 mph.
Using the Pythagorean theorem:
a/ [ (80 mph)2 + (60 mph)2 ]

7. 5.59 m/s

2-98 Total Training Support Ltd


Issue 2 ~ September 2016
Module 2.2 Mechanics © Copyright 2016
Module 2.2 Mechanics
Stress, strain and elasticity
The structure (or ‘airframe’) of an aircraft or helicopter is designed to carry the loads imposed by
the forces of lift, thrust, drag and weight in an efficiently aerodynamic way which still permits a
commercially viable payload.

Any structure can be divided into individual elements which, when loaded, will behave in a
certain way in a known direction. Generally, a simple structure is made up of three types of
member, i.e. beams, struts and ties. A member subject to bending is known as a beam, one
being subject to compression as a strut and one being subject to tension as a tie.

The load applied to a member is the force in pounds, kilograms or newton’s, but the most
important factor from the point of view of the strength of a member is not the load it has to carry
but the relationship between the load and the cross-sectional area of the member. This is called
the ‘stress’ in the member and is measured in pounds per square inch, newtons per square
metre or some other suitable units.

When a load is applied to a member in a framework it will, to some extent, move under the load.
For example, a tie will stretch under load. The vertical movement of the wing tips of a large
aircraft is evidence of the extension and compression of the upper and lower surfaces. This
movement under load is called ‘strain’.

Most parts of an aircraft cannot be simply classified as a strut, a tie or a beam because the
loads applied to them can change direction depending on whether the aircraft is in flight or on
the ground. A typical example is the lift strut of a high-wing light aircraft. It is in fact a strut only
when the aircraft is on the ground and it is supporting the weight of the wing. During a heavy
landing, the compressive loads on the strut are increased considerably. In flight, it is no longer a
strut but acts as a tie since the lift developed by the wing has reversed the loads.

The spar of a helicopter rotor blade will be in tension at the upper surface and in compression at
the lower surface with the aircraft on the ground and the rotor stationary. In flight, the situation is
reversed with the upper surface experiencing the compressive stress. Each part of the aircraft
can be treated in the same way for the different conditions of operation and an appreciation
gained as to the direction of the loads applied. Such knowledge is useful when inspecting for
possible damages. In addition to the loads imposed by lift and weight there are also loads
imposed by thrust and drag and, on the ground, manoeuvres such as braking and turning.

Prior to designing an effective repair to a damaged sheet metal aircraft, the stresses that act on
the structure must have been identified. There are only five types of stress, among which two
are primary ones.

The other three types can, for practical purposes, be expressed in terms of the first two.

Total Training Support Ltd 2-99


©Copyright 2016 Module 2.2 Mechanics Issue 2 - September 2016
Module 2.2 Mechanics
Primary stresses - tension, compression and shear

The two basic structural stresses to which aircraft are subject are:

• tension
• compression.

Tension - tension is a primary stress that tries to pull a body apart. When a weight is supported
by a chain, the chain is subjected to tension or, as it is normally expressed, to a tensile stress.
The weight is attempting to pull the chain apart. The result is a stretching effect on the airframe.
Bracing wires in an aircraft are usually in tension.

Compression - the other primary stress, tries to squeeze the part together. A weight supported
on a post exerts a force that tries to squeeze the ends of the post together or to collapse it. This
is called ‘compressive stress’. Tension and compression are the two basic stresses and the
other three, bending, torsion and shear, are really just different arrangements of tension and
compression working on a body at the same time.

2-100 Total Training Support Ltd


Issue 2 ~ September 2016 © Copyright 2016
Module 2.2 Mechanics
Module 2.2 Mechanics

Total Training Support Ltd 2-101


© Copyright 2016 Moduie 2.2 Mechanics Issue 2 - September 2016
Module 2.2 Mechanics
Secondary stresses - shear, torsion and bending

Shear - shear forces try to slide a body apart and if a rivet or bolt is examined that has failed
because of shear forces, it will be recognized that the shank has actually been pulled apart, not
along its length, but across its shank. Rivets hold pieces of the aircraft’s skin together and in a
properly designed riveted joint, the rivets support shear loads only. Such a joint should never be
required to support tensile loads. The airframe is subjected to variable torsional stresses during
turns and other manoeuvres.

Torsion - torsion is a twisting force. When a structural member is twisted, a tensile stress acts
diagonally across the member and a compressive stress acts at right angles to the tension. The
crankshaft of an aircraft engine is under a torsional load when the engine spins the propeller.

2-102 Total Training Support Ltd


Issue 2 - September 2016
Module 2.2 Mechanics ©Copyright 2016
Module 2.2 Mechanics

Propeller

Compression

A torsional stress consists of tension and


compression both acting diagonally across
the body and perpendicular to each other.

H
I

A shear stress on a rivet attempts to slide the shank apart.

Total Training Support Ltd 2-103


©Copyright 2016 Module 2.2 Mechanics Issue 2 ~ September 2016
Module 2.2 Mechanics
Bending - a bending force tries to pull one side of a body apart while at the same time
squeezing the other side together When a weight is put on a wooden board fixed at one end to
a wall, the top of the board is under a tensile stress while the bottom feels compression.

Wing spars of cantilever wings or the section of a wing spar outboard of the struts is subjected
to bending stresses. In flight, the top of the spar is being compressed and the bottom is under
tension, but on the ground, the top is in tension and the bottom is in compression.

The wing struts are under tension in flight but under compression on the ground. There are
many other areas of the airframe structure that experience bending forces during normal flight.

An aircraft structure in flight is subjected to many and varying stresses due to the varying loads
that may be imposed. The designer’s problem is trying to anticipate the possible stresses that
the structure will have to endure, and to build it sufficiently strong to withstand these. The
problem is complicated by the fact that an aeroplane structure must be light as well as strong.

Internal Forces-Tension, Shear Force, Bending Moment


https://youtu.be/Z-jQyeYApU8

2-104 Total Training Support Ltd


Issue 2 - September 2016 © Copyright 2016
Module 2.2 Mechanics
Total Training Support Ltd 2-105
© Copyright 2016 Moduie 2.2 Mechanics Issue 2 - September 2016
Module 2.2 Mechanics
Stress and strain
What is known as axial (or normal) stress, is defined as the force perpendicular to the cross
sectional area of the member divided by the cross sectional area. Or:

Stress “ - (units Ifo/in2 or N/m2)


Area
In the figure opposite, a solid rod of length L, is under simple tension due to force F, as shown.
If we divide that axial force, F, by the cross sectional area of the rod (A), this would be the axial
stress in the member. Axial stress is the equivalent of pressure in a gas or liquid. As you
remember, pressure is the force/unit area. So axial stress is really the ‘pressure’ in a solid
member. Now the question becomes, how much ‘pressure’ can a material bear before it fails.

In fact, if we look at a metal rod in simple tension, we see that there will be an elongation (or
deformation) due to the tension. If we then graph the tension (force) versus the deformation, we
obtain a result as shown in the figure opposite.

We see that, if our metal rod is tested by increasing the tension in the rod, the deformation
increases. In the first region the deformation increases in proportion to the force. That is, if the
amount of force is doubled, the amount of deformation is doubled.

This is Hooke’s law and could be written this way: F oo k, where k is a constant depending on
the material (and is sometimes called the spring constant). After enough force has been applied
the material enters the plastic region - where the force and the deformation are not
proportional, but rather a small amount of increase in force produces a large amount of
deformation. In this region, the rod often begins to ‘neck down’, that is, the diameter becomes
smaller as the rod is about to fail. Finally, the rod actually breaks.

The point at which the elastic region ends is called the elastic limit, or the proportional limit. In
actuality, these two points are not quite the same.

The elastic limit is the point at which permanent deformation occurs, that is, after the elastic
limit, if the force is taken off the sample, it will not return to its original size and shape,
permanent deformation has occurred.

The proportional limit is the point at which the deformation is no longer directly proportional to
the applied force (Hooke’s law no longer holds). Although these two points are slightly different,
we will treat them as the same in this text.

Intro to springs and Hooke’s law I Work and energy 1 Physics I Khan Academy
https://youtu.be/ZzwuHS9ldbY

Stress, Strain, and Elasticity (Part 1)


https://youtu.be/ikBWIESmrxE

Stress, Strain, and Elasticity (Part 2)


https://youtu.be/sH82s01Tt80

Stress, Strain, and Elasticity (Part 3)


https ://youtu.be/1 sYgK7x5bTE

2-106 Total Training Support Ltd


Issue 2 - September 2016
Module 2.2 Mechanics © Copyright 2016
Module 2.2 Mechanics
Axial stress (c) = FA

SRB
Aluminium 1.9 xW 2.1 x 104

Copper 2.3 x104 4.9 x104


IBKSBS!
;-• .r Tzr.‘ 7"Steei
Annealed ' •'Z! ‘ ‘?IA- 3.6 x 10yr; 4 -’ iT p. .?• O-'T.7.1 :■! i 4 2 ■' —V-
'-- - xIO
iOWSSOsTsOOBfiSS
Elastic limit and ultimate stress of some common materials

Force-extension diagram

Total Training Support Ltd 2-107


© Copyright 2016 Module 2.2 Mechanics Issue 2 - September 2016
Module 2.2 Mechanics
Next, rather than examining the applied force and resulting deformation, we will instead graph
the axial stress verses the axial strain. We have defined the axial stress earlier.

The axial strain is defined as the fractional change in length or:


(deformation of member)
Strain ~
divided by the (original length of member)

AL
Strain = —
Lo
where Lo is the original length of the member.

We may write:

Strain
Original Length

Strain has no units - since it is length divided by length, however it is sometimes expressed as
In/in (or inches per inch)’ in some texts.

As we see from the figure opposite, the stress verses strain graph has the same shape and
regions as the force verses deformation graph. In the elastic (linear) region, since stress is
directly proportional to strain, the ratio of stress/strain will be a constant (and actually equal to
the slope of the linear portion of the graph).

This constant is known as Young’s Modulus, and is usually symbolized by an E or Y. We will


use E for Young’s modulus. We may now write:
Young’s Modulus (E) = $*ress,
' ' Strain
(This is another form of Hooke’s law.)

The value of Young’s modulus - which is a measure of the amount of force needed to produce
a unit deformation - depends on the material.

Young’s Modulus for steel is 30 x 106 lb/in2, for aluminium E = 10 x 106 lb/in2, and for
brass E = 15 x 106 lb/in2.

To summarize our stress/strain/Hooke’s law relationships up to this point, we have:


F"orce
Stress ™-------- (units lb/in2 or N/m2)
Area
Strain ~ Deformation
Original Length
w
Young»’s m . =—
Modulus Stress
------
Strain
Mechanical Properties Definitions
https://youtu.be/1 UbO7UxvPBc

2-108 Total Training Support Ltd


Issue 2 - September 2016 ©Copyright 2016
Module 2.2 Mechanics
Module 2.2 Mechanics

Axial force in a member of length Lo causing deformation

....... .7* T-LT'” "i. -y-.v ISfHH1 ?J??s.rr.. :;j.< : 1 ■■:.


a;Xlia&0C"?T
■■■■■.'■'.' :' -■. ■'■ '■ ■;•.:■'<■<■■:■:.■; fe.:■=;' ■ ■ Aluminium io x io6 ioxio6
jftasKaSSgSSSSSSBHB8HBIB8
Aluminium 1.9 x104 2.1 x 104 Copper 16x10® 17x10®
5.5 x 104 .S! 6.6 x 10“ iSSR
Copper 2.3 x 104 4.9 x 104 Iron 13 x 10s 1.45 x10s
ISIKHiSSS#WHSi fflWsSIfiliSillll g^||oO®SWB!S’
Annealed Steel 3.6 x104 7.1 x 104 Ethyl Alcohol „. v. ,v . p.... . . . ., , ..._........0.16x10®
. , c,.,....,...... ,. .
6.0 x 104 TMItiliSH iKITBWTOMQBfisi
Elastic limit and ultimate stress of Water 0.31 x 10s
'i’i-'“T ■ ' ;g;';7?; :r=i<3
some common materials Mercury
Young’s modulus and bulk modulus of

Stress-strain graph

Total Training Support Ltd 2-109


©Copyright 2016 Module 2.2 Mechanics Issue 2 - September 2016
Module 2.2 Mechanics
Example

The elastic limit for copper is 2.3 x 104 lb/in2 and the ultimate strength is 4.9 x 104 lb/in2.
Suppose that a copper rod has a cross-sectional area of 0.5 in2.

A force of 11,500 lbs applied longitudinally to this rod would just be within the elastic limit.

A force of 12,000 lbs would deform the rod in such a way that it would not return to its original
size after the force is removed.

A force of 24,500 lbs would cause the rod to rupture.

2-110 Total Training Support Ltd


Issue 2 - September 2016 ©Copyright 2016
Module 2.2 Mechanics
Moduie 2.2 Mechanics
Worksheet

1. A steei boit with a cross- sectional area of 0.1 in2 and a length of 6.0 in is subjected to a
force of 580 lbs. What is the increase in length of the bolt?

(Hint: Find the stress. Then use Young’s Modulus of steel to find the strain. From the strain
find the extension).

2. An iron body of volume 145 in3 is subjected to a pressure of 500 lb/in2. What is the
decrease in volume of this body?

3. A copper rod has a cross-sectional area of 0.04 in2 and a length of 24 in. What longitudinal
force must be applied to cause this rod to stretch by 0.0024 inches?

4. An aluminium brace inside a wing of an aeroplane has a cross-sectional area of 0.2 in2.
What is the greatest longitudinal force that can be applied to the brace without causing the
brace to be permanently deformed?

Totai Training Support Ltd 2-111


© Copyright 2016 Module 2.2 Mechanics Issue 2 - September 2016
Module 2.2 Mechanics

Intentionally Blank

2-112 Totai Training Support Ltd


Issue 2 ~ September 2016
Module 2.2 Mechanics ©Copyright 2016
Module 2.2 Mechanics
Answers

1. 0.0012 in.

2. 0.05 in3

3. 64 lb

4. 3,800 lb

Total Training Support Ltd 2-113


©Copyright 2016 Module 2.2 Mechanics Issue 2 - September 2016
Module 2.2 Mechanics
Structural definitions

Strut - A strut is a structural component designed to resist longitudinal compression. Struts


provide outwards-facing support in their lengthwise direction, which can be used to keep two
other components separate.

Bracing struts and wires of many kinds were extensively used in early aircraft to stiffen and
strengthen, and sometimes even to form, the main functional airframe. Throughout the 1920s
and 1930s they fell out of use in favour of the low-drag cantilever construction. Most aircraft
bracing struts are principally loaded in compression, with wires taking the tension loads. Lift
struts came into increasing use during the changeover period and remain in use on smaller
aircraft today where ultimate performance is not an issue. Typically, they are applied to a high-
wing monoplane and act in tension during flight.

Struts have also been widely used for purely structural reasons to attach engines, landing gear
and other loads. The oil-sprung legs of retractable landing gear are still called Oleo struts

Tie - Also called a strap or tie rod, are linear structural components designed to resist tension.

Beam - A beam is a structural element that is capable of withstanding load primarily by resisting
against bending. The bending force induced into the material of the beam as a result of the
external loads, own weight, span and external reactions to these loads is called a bending
moment

Cantilever - A type of beam under bending stress (which is greatest at the root end) and shear
stress (which is constant along the beam). It is supported at one end only.

Bulk Modulus - The bulk modulus gives the change in volume of a solid substance as the
pressure on it is changed. The formula for bulk modulus is very similar to that for Young’s
Modulus:

x.
Pressure
Bulk Modulus (B) = — = Pressure x
Volumetric Strain AV AV
Vo
Some examples of Bulk Modulus for different materials are given here.

Poisson’s Ratio - As a member is stressed in tension, its length increases (axial strain) and its
width decreases (transverse strain). The opposite occurs when the member is in compression.

Poisson’s Ratio is the ratio of transverse strain (sy) to the axial strain (ex) in a stressed member.

Poisson’s £y
ratio =

2-114 Total Training Support Ltd


Issue 2 - September 2016 © Copyright 2016
Module 2.2 Mechanics
Slodute 2.2 Mechanics

External struts and ties on a biplane

Bending stress

Volume Vo

Volume
Vo«AV

Bulk modulus = Pressure x


AV

Total Training Support Ltd 2-115


©Copyright 2016 Module 2.2 Mechanics Issue 2 - September 2016
Module 2.2 Mechanics
Materials behaviour
The following are the properties of a material under stress which are important in engineering
terms;

Tensile Strength
Tensile strength or tenacity describes the ability of a material to resist breaking under a tensile
(stretching) force. Ultimate tensile strength (UTS) is a measure of the maximum pulling stress
which a material can endure before breaking. UTS is rated as a given load across a unit area.

Compressive Strength
This is a measure of a materials ability to withstand compressive or squeezing loads without
splitting or cracking.

Shear Strength
Shear Strength is the ability to resist off-set (scissor action) loads.

Toughness
Toughness is the ability of a material to resist fracture under impact load. It is the opposite of
brittleness and should not be confused with strength.

Elasticity
Elasticity is the ability of a material to return to its original shape after deformation. The
extension or compression of a spring is a good example.

Plasticity
Plasticity is the opposite to elasticity and is the ability of a material to retain any shape imposed
by a force when that force is removed. A good example of this is the stamping of images on
medals and coins.

Ductility
Ductility is the capacity of a material for plastic deformation under tensile loading without
fracture. A ductile material may be worked into shape without loss of strength and may be
drawn into wire form. If subjected to a shock load, the material will yield and become deformed.

Malleability
Malleability is the capacity of a material for plastic deformation and forming under compressive
load without cracking. It should not be confused with ductility.

Hardness
The ability of a material to withstand scratching, abrasion, indentation or permanent distortion
and wear. Hardness may be increased by cold working the metals and, in the case of many
alloys, by heat treatment.

Brittleness
Brittleness is the tendency of a material to fracture when subjected to a shock loading or blow.
There is no permanent deformation before fracture to act as a warning of failure.

Rigidity
Rigidity or stiffness is the ability of a material to withstand a load without deflection. It should not
be confused with strength and rigid materials are often brittle.

Properties of Materials
https://youtu.be/LdxSW6pNj5s

2-116 Total Training Support Ltd


Issue 2 - September 2016 ©Copyright 2016
Module 2.2 Mechanics
Module 2.2 Mechanics
GT Grade Carbon Ultegra

Shear
Toughness

Plasticity

Ductility

Brittleness
Rigidity

Total Training Support Ltd 2-117


©Copyright 2016 Module 2.2 Mechanics Issue 2 - September 2016
Module 2.2 Mechanics
Shear force and bending moment diagrams
From a design point of view, it is often necessary to know the maximum and minimum internal
loads in a structure and where they are located.

If the shear force and bending moment are calculated and graphed, then the maximum and
minimum of each are easily identified and located.

Shear force diagram (SFD) of a simple beam


We will look at calculating the shear force diagram of a simple beam. A shearing force occurs
when a perpendicular force is applied to static material (in this case a beam).

Think of a knife cutting through a carrot. Imagine the beam is the carrot and a point load is the
knife. As the knife applies a downward force, it cuts (or shears) the carrot. These forces occur
along numerous points of a beam, and it is important to determine where these shears are at
the greatest points as this may be where a beam fails.

To calculate the shear forces of a beam, follow the following simple steps:

1. Start with the far left side of the beam

If there is an upward force (i.e. a support) then the SFD will start at this force above the x-
axis. If there is a downward point load and no support, then the shear force diagram will
start as a negative at the value of the point load.

2. Move across the beam

As you come across loads, you simply add (or subtract) these loads from the value you
already have, keeping a cumulative total.

2-118 Totai Training Support Ltd


issue 2 - September 2016
Module 2.2 Mechanics © Copyright 2016
Module 2.2 Mechanics
20 kN

A
2m 2m

20 kN

10 kN
Shear force diagram Shear force diagram
10 kN

10 kN 10 kN
Shear force diagram
10 kN

-10 kN

Total Training Support Ltd 2-119


© Copyright 2016 Moduie 2.2 Mechanics Issue 2 - September 2016
Module 2.2 Mechanics
Bending moment diagram (BMD) of a simple beam
Opposite are simple instructions on how to calculate the bending moment diagram of a simple
supported beam. Study this method as it is very versatile (and can be adapted to many different
types of problem). The ability to calculate the bending moment of a beam is very common
practice for structural engineers and often comes up in college and high school exams.

A moment is rotational force that occurs when a force is applied perpendicularly to a point at a
given distance away from that point. It is calculated as the perpendicular force multiplied by the
distance from the point. A “bending moment” is simply the bend that occurs in a beam due to a
moment.

it is important to remember two things when calculating bending moments;

• the standard units are Nm (or lb ft) and


• clockwise bending is taken as negative.

Let’s look at the steps to calculate a bending moment diagram:

1. Calculate reactions at supports and draw a free body diagram.

Once you have the reactions, draw your free body diagram and shear force diagram
underneath the beam:

2. From left to right, make “cuts” before and after each reaction/load.

To calculate the bending moment of a beam, we must work in the same way we did for the
shear force diagram. Starting at x = 0 we will move across the beam and calculate the
bending moment at each point.

2-120 Total Training Support Ltd


Issue 2 - September 2016 © Copyright 2016
Module 2.2 Mechanics
Module 2.2 Mechanics

20 kN

20 kN

supports

Total Training Support Ltd 2-121


©Copyright 2016 Moduie 2.2 Mechanics Issue 2 - September 2016
Module 2.2 Mechanics
Cut 1
Make a “cut” just after the first reaction of the beam. In our simple example.

So, when we cut the beam, we only consider the forces that are applied to the left of our cut. In
this case we have a 10 kN force in the upward direction. Now as you recall, a bending moment
is simply the force x distance. So as we move further from the force, the magnitude of the
bending moment will increase. We can see this in our BMD. The equation for this part of our
bending moment diagram is:

-M(x) = 10(-x)

M(x) = 10x

2-122 Total Training Support Ltd


Issue 2 - September 2016 ©Copyright 2016
Module 2.2 Mechanics
Module 2.2 Mechanics

Cut 1

We make a cut just


after the first reaction
(or toad)

There is only one force


applied to the beam

The bending at this point


increases as you move
across the beam

Total Training Support Ltd 2-123


©Copyright 2016 Module 2.2 Mechanics issue 2 - September 2016
Module 2.2 Mechanics
Cut 2
This cut is made just before the second force along the beam. Since there are no other loads
applied between the first and second cut, the bending moment equation will remain the same.
This means we can calculate the maximum bending moment (in this case at the midpoint,
or x = 5) by simply substituting x = 5 into the above equation:

2-124 Total Training Support Ltd


Issue 2 “ September 2016
Module 2.2 Mechanics © Copyright 2016
Module 2.2 Mechanics

Cut 2

There is still only one


reaction tro consider

Bending moment
continues along the
same line as there are
no other forces applied
to the beam

Total Training Support Ltd 2-125


©Copyright 2016 Module 2.2 Mechanics Issue 2 - September 2016
Module 2.2 Mechanics
Cut 3
This cut is made just after the second force along the beam. Now we have two forces that act to
the left of our cut: a 10 kN support reaction and a -20 kN downward acting load. So now we
must consider both these forces as we progress along our beam. For every metre we move
across the beam, there will a +10 kNm moment added from the first force and -20 kNm from the
second. So after the point x = 5, our bending moment equation becomes:

M(x) = 50 +10(x-5) - 20(x-5)

M(x) = 50 -10(x-5) for 5 < x < 10

Note: The reason we write (x-5) is because we want to know the distance from the point x = 5
only. Anything before this point uses a previous equation.

2-126 Total Training Support Ltd


issue 2 - September 2016
Module 2.2 Mechanics © Copyright 2016
Module 2.2 Mechanics

Cut 3

Cutting just to the


right of the
downward force
means we now have
to consider two
forces

Again, consider ail


forces to the left of
your cut

Since a new force has


been applied, the
equation will now
change. In this case, the
downward force is much
greater, therefore, the
movement in the x
direction will equate to a
10x-20x = -10x
change in the bending
moment

Total Training Support Ltd 2-127


©Copyright 2016 Module 2.2 Mechanics Issue 2 - September 2016
Module 2.2 Mechanics
Cut 4
Now move across to the right of our beam again and make a cut just before our next force. In
this case, our next cut will occur just before the reaction from right support. Since there are no
other forces between the support and our previous cut, the equation will remain the same:

M(x) = 50 - 10(x-5) for 5 < x < 10

And let’s substitute x = 10 into this to find the find bending moment at the end of the beam:

M(x) = 50 ~~ 10(10-5) = 0 kNm

This makes perfect sense. Since our beam is static (and not rotation) it makes sense that our
beam should have zero moment at this point when we consider all our forces. It also satisfies
one of our initial conditions, that the sum of moments at a support is equal to zero.

Note: If your calculations lead you to any other number other than 0, you have made a mistake!

Shear, Bending Moment, Bending Stress Diagrams (Nazeer A. Khan)


https://youtu. be/PtOxi ILphqQ

2-128 Totai Training Support Ltd


Issue 2 ~ September 2016
Module 2.2 Mechanics © Copyright 2016
Module 2.2 Mechanics

Cut 4

Like cut 3, we still


have two forces
acting to the left of
our cut, so these will
be the only forces
we will consider

if we continue using the


formula we found in our
previous cut, we should
find that the bending
moment diagram will
head towards zero

Total Training Support Ltd 2-129


© Copyright 2016 Moduie 2.2 Mechanics Issue 2 - September 2016
Module 2.2 Mechanics
Nature and properties of solids, liquids and gas
All matter exists in one of three states - solid, liquid or gas. Liquids and gases are both
considered to be ‘fluids’ in their behaviour.

The following notes characterize the three states:

Solid
The greatest forces of attraction are between the particles in a solid and they pack together in a
neat and ordered arrangement.

The particles are too strongly held together to allow movement from place to place but the
particles vibrate about their position in the structure.

With increase in temperature, the particles vibrate faster and more strongly as they gain kinetic
energy.

The properties of a solid

• Solids have the greatest density (‘heaviest’) because the particles are closest together.
• Solids cannot flow freely like gases or liquids because the particles are strongly held in fixed
positions.
• Solids have a fixed surface and volume (at a particular temperature) because of the strong
particle attraction.
• Solids are extremely difficult to compress because there is no real ‘empty’ space between
the particles.

Solids will expand a little on heating but nothing like as much as liquids because of the greater
particle attraction restricting the expansion (contract on cooling). The expansion is caused by
the increased strength of particle vibration.

Liquid
Much greater forces of attraction between the particles in a liquid compared to gases, but not
quite as much as in solids.

Particles quite close together but still arranged at random throughout the container, there is a
little close range order as you can get clumps of particles clinging together temporarily.

Particles moving rapidly in all directions but more frequently colliding with each other than in
gases.

With increase in temperature, the particles move faster as they gain kinetic energy.

Properties of a Siquid

• Liquids have a much greater density than gases (‘heavier’) because the particles are much
closer together.
• Liquids flow freely despite the forces of attraction between the particles but liquids are not as
‘fluid’ as gases.
• Liquids have a surface, and a fixed volume (at a particular temperature) because of the
increased particle attraction, but the shape is not fixed and is merely that of the container
itself.
• Liquids are not readily compressed because of the lack of ‘empty’ space between the
particles.

2-130 Total Training Support Ltd


Issue 2 - September 2016 ©Copyright 2016
Module 2.2 Mechanics
Module 2.2 Mechanics
• Liquids will expand on heating (contract on cooling) but nothing like as much as gases
because of the greater particle attraction restricting the expansion. When heated, the liquid
particles gain kinetic energy and hit the sides of the container more frequently, and more
significantly, they hit with a greater force, so in a sealed container the pressure produced
can be considerable.

Atom arrangement in a solid

Atom arrangement in a liquid

Solid
• holds shape • shape of container • shape of container
• fixed volume • free surface • volume of container
« fixed volume

Solid, liquid and gas comparison

Total Training Support Ltd 2-131


©Copyright 2016 Module 2.2 Mechanics Issue 2 - September 2016
Module 2.2 Mechanics
Gas
Almost no forces of attraction between the particles which are completely free of each other.

Particles widely spaced and scattered at random throughout the container so there is no order
in the system.

Particles moving rapidly in all directions, frequently colliding with each other and the side of the
container.

With increase in temperature, the particles move faster as they gain kinetic energy.

Properties of a gas

• Gases have a low density (light’) because the particles are so spaced out in the container
(density = mass 4- volume).
• Gases flow freely because there are no effective forces of attraction between the particles.
• Gases have no surface, and no fixed shape or volume, and because of lack of particle
attraction, they spread out and fill any container.
• Gases are readily compressed because of the ’empty’ space between the particles.
• If the ‘container’ volume can change, gases readily expand on heating because of the lack of
particle attraction, and readily contract on cooling. On heating, gas particles gain kinetic
energy and hit the sides of the container more frequently, and more significantly, they hit
with a greater force. Depending on the container situation, either or both of the pressure or
volume will increase (reverse on cooling).
• The natural rapid and random movement of the particles means that gases readily ‘spread’
or diffuse. Diffusion is fastest in gases where there is more space for them to move and the
rate of diffusion increases with increase in temperature.

States of Matter (solids, liquids and gases) I The Chemistry Journey [ The Fuse School
https://youtu.be/bMbmGzV-Ezs

2-132 Total Training Support Ltd


issue 2 - September 2016 © Copyright 2016
Module 2.2 Mechanics
Module 2.2 Mechanics

Solid Liquid Gas


* holds shape • shape of container * shape of container
• fixed volume • free surface • volume of container
® fixed volume

Solid, liquid and gas comparison

Total Training Support Ltd 2-133


© Copyright 2016 Module 2.2 Mechanics Issue 2 - September 2016
Module 2.2 Mechanics
Changes of state
We can use the diagrams shown opposite, to explain changes of state and the energy changes
involved.

Evaporation (liquid to gas)


in evaporation and boiling the highest kinetic energy molecules can ‘escape’ from the attractive
forces of the other liquid particles. The particles lose any order and become completely free.
Energy is needed to overcome the attractive forces in the liquid and is taken in from the
surroundings.

This means heat is taken in (endothermic). Boiling is rapid evaporation at a fixed temperature
called the boiling point and requires continuous addition of heat. Evaporation takes place more
slowly at any temperature between the melting point and boiling point and results in the liquid
becoming cooler.

Condensation (gas to liquid)


On cooling, gas particles lose kinetic energy and eventually become attracted together to form a
liquid. There is an increase in order as the particles are much closer together and can form
clumps of molecules. The process requires heat to be lost to the surroundings i.e. heat given
out, so condensation is exothermic.

Melting (solid to liquid)


When a solid is heated the particles vibrate more strongly and the particle attractive forces are
weakened. Eventually, at the melting point, the attractive forces are too weak to hold the
structure together and the solid melts. The particles become free to move around and lose their
ordered arrangement. Energy is needed to overcome the attractive forces, so heat is taken in
from the surroundings and melting is an endothermic process.

Freezing (liquid to solid)


On cooling, liquid particles lose kinetic energy and become more strongly attracted to each
other. Eventually at the freezing point the forces of attraction are sufficient to remove any
remaining freedom and the particles come together to form the ordered solid arrangement.
Since heat must be removed to the surroundings freezing is an exothermic process.

Sublimation and deposition


Sometimes solids can become gases without ever becoming liquids. This is called sublimation.

When a gas becomes a solid without going through the liquid phase, it is called deposition.

Particulate Nature of Matter and Changes of State


https://youtu.be/ndw9XYA4iF0

2-134 Total Training Support Ltd


Issue 2 - September 2016 © Copyright 2016
Module 2.2 Mechanics
Module 2.2 Mechanics
Oas tiguid
(exothermic)
condense

Ml,.evaporate
''(endotherniife^
Gas to liquid transformation

Liquid to solid transformation

Sublimation

Deposition

Total Training Support Ltd 2-135


© Copyright 2016 Module 2.2 Mechanics Issue 2 - September 2016
Module 2.2 Mechanics
Pressure

Definition
Pressure is the force exerted on a unit area, i.e.:
„ Force Mass x Acceleration
Pressure =--------- =------------------------------
Area Area

It can be applied to any fluid (both gases and liquids).

In the atmosphere, pressure is caused by the mass air acting under the force of gravity on a
given area. As all molecules act under gravity then the pressure can also be considered to be
the weight of a column of air on a unit area.

A fluid contains a certain number of molecules. These molecules are in rapid and ceaseless
motion, and bombard any surface that is immersed in the fluid.

During the collision between a molecule and the surface, the molecule exerts a force on the
surface. When added the millions upon millions of collisions result in an evenly distributed force
on the surface.

Pressure has a magnitude but, unlike a vector, it has no direction. In many old books one will
often find the definition that pressure acts equally in all directions, it does not. Pressure does
not act in any direction; it is the force created as a result of pressure acting on a surface that
has a direction. The direction depends upon the orientation of the surface.

The forces always act at right angles to the surface that the pressure is exposed to. You may
perhaps think that this explanation is a little pedantic, but the pressure definition is important to
understand when dealing with aerodynamic forces.

If the volume (holding the same number of molecules) is reduced, the molecules act on a
smaller area. Thus, the force exerted per unit area increases, hence the volume has higher
pressure. The opposite will occur if the volume is increased.

If the temperature in the enclosed fluid is increased, then the molecules start to move faster.
The force exerted by the collision is increased and so is the pressure. If the temperature is
lowered the opposite takes place and at the theoretical temperature absolute zero (0), the
molecules are in rest and the pressure is zero (0).

2-136 Total Training Support Ltd


Issue 2 - September 2016 ©Copyright 2016
Module 2.2 Mechanics
Module 2,2 Mechanics
Force
(weight)
[N]

Pa =H,= pascal PSI=M,

Volume
of gas [m3]

Bouncing
f°rce

Air mass (molecules)

A pressure creates a force


upon any surface with which it
is in contact.

The Force = Pressure x Area

Increased gas temperature

Pressure is caused by collisions of the molecules of the fluid against the


surfaces with which it is contact.

For any given number of molecules, a smaller volume creates a higher


number of collisions thus a higher pressure.

A higher temperature gives the molecules more kinetic energy so they


strike the surfaces with greater force, thus a greater pressure

Total Training Support Ltd 2-137


©Copyright 2016 Module 2.2 Mechanics Issue 2 - September 2016
Module 2.2 Mechanics
Another important property of gases is that they always tend to move towards regions having a
lower pressure than where it comes from (i.e. high pressure always flows towards low pressure,
until the pressures are equalised).

Gas Pressure: The Basics


https ://youtu. be/zvh 9uv2 Hxx4

Air pressure
https://youtu.be/oWiOEMaMy3c

2-138 Total Training Support Ltd


issue 2 - September 2016 ©Copyright 2016
Module 2.2 Mechanics
Module 2.2 Mechanics

Cold air is heavier (more dense) so it sinks,


increases pressure, and a wind blows
towards areas of warm air (low pressure)

lilliiai
Force
WS:
pressure

A fluid flows from a high pressure


towards a lower pressure

Total Training Support Ltd 2-139


©Copyright 2016 Module 2.2 Mechanics Issue 2 - September 2016
Module 2.2 Mechanics
Unit
The SI unit for pressure is newton per square meter (N/m2). If an area of 1 m2 is exposed to a
force of 1 N the pressure is 1 N/m2 also known as 1 pascal (Pa). A pressure of 1 Pa is an
extremely low pressure (1/100,000 Bar).

Although largely obsolete, the imperial system of units is still encountered, and pressure is
expressed in pounds per square inch.

In meteorology, the unit of pressure is the millibar (mb).

Before the introduction of the millibar, meteorological pressure was measured in terms of the
length of a column of mercury in a barometer that the weight of the atmosphere could support.

Measuring pressure
A mass of air has weight, i.e. it is a force. The weight is the mass of the air (kg) multiplied by the
acceleration due to gravity (9.81 m/s2) The weight of air exerted to a surface produces a
pressure.

Atmospheric pressure is measured by means of a barometer. The principle of measuring air


pressure is illustrated opposite where a mercury barometer is shown (known as Torricelli’s
mercury barometer). Mercury is denoted ‘Hg’, or ‘hg’. The weight of a certain air mass exerts a
force on the mercury surface. Depending of the magnitude of the outside pressure a quantum of
mercury is either raised or lowered inside a glass tube, which is sealed at the top (it is a perfect
vacuum). The column of mercury above the surface, either measured in mms of mercury (mm
Hg) or inches of mercury (in Hg), thus represents the value of the local atmospheric pressure.

The standard pressure (or normal pressure) measured at mean sea level (MSL) is 1 ATM,
equivalent to 760 mm hg or 29.92 in hg (29.92” hg). This way of measuring is somewhat
misleading since it is a length and not a pressure. But, like it or not, it is common practice to do
so!

A pressure of 1 ATM is equal to 101,326 Pa or 1,013.25 hPa (hectopascals) or 101.325 kPa


(kilopascal). The unit hectopascal (hPa) is the standard unit now applied in all meteorological
aspects, and has replaced the older millibar (mb) unit. However, there is no numerical difference
between mb and hPa as 1 mb ~ 1 hPa.

When measuring the pressure in hydraulic systems and pneumatic systems etc., the unit “bar”
is frequently used in European countries, because it is a large unit. 1 bar = 100,000 Pa.

Most industries in the USA and in the commonwealth countries still prefer the unit “pound per
square inches’ (psi) or pound per square feet (PSF).

No wonder that people often get confused with all these units in frequent use. If you have to
make calculations based on SI units and pressure is included, the pressure must be converted
to pascal.

2-140 Total Training Support Ltd


Issue 2 - September 2016 ©Copyright 2016
Module 2.2 Mechanics
Module 2.2 Mechanics

“Vacuum

SjefdW

Atmospheric E
pressure P
A
mriM^

Principle of the mercury barometer - The force due to the atmospheric pressure holds the
column of mercury in the tube. The weight of the column of mercury is equal to the force due
to atmospheric pressure over the same cross sectional area

Total Training Support Ltd 2-141


© Copyright 2016 Module 2.2 Mechanics Issue 2 - September 2016
Module 2.2 Mechanics
Atmospheric pressure
The atmosphere is the entire mass of air that surrounds the earth. While it extends upward for
about 500 miles, the section of primary interest is the portion that rests on the earth's surface
and extends upward for about 7% miles. This layer is called the troposphere.

If a column of air 1-inch square extending all the way to the “top" of the atmosphere could be
weighed, this column of air would weigh approximately 14.7 pounds at sea level. Thus,
atmospheric pressure at sea level is approximately 14.7 psi.

As one ascends, the atmospheric pressure decreases by approximately 1,0 psi for every
2,343 feet. However, below sea level, in excavations and depressions, atmospheric pressure
increases. Pressures under water differ from those under air only because the weight of the
water must be added to the pressure of the air.

The atmospheric pressure does not vary uniformly with altitude. It changes more rapidly at lower
altitudes because of the compressibility of the air, which causes the air layers close to the
earth’s surface to be compressed by the air masses above them. This effect, however, is
partially counteracted by the contraction of the upper layers due to cooling. The cooling tends to
increase the density of the air.

Atmospheric pressures are quite large, but in most instances practically the same pressure is
present on all sides of objects so that no single surface is subjected to a great load.

Atmospheric pressure acting on the surface of a liquid, figure opposite, view (A), is transmitted
equally throughout the liquid to the walls of the container, but is balanced by the same
atmospheric pressure acting on the outer walls of the container.

In figure opposite view (B), atmospheric pressure acting on the surface of one piston is
balanced by the same pressure acting on the surface of the other piston. The different areas of
the two surfaces make no difference, since for a unit of area, pressures are balanced.

At sea level, pressure generally varies between 950 and 1,050 mb.

With increasing altitude, the mass of overlying air decreases and so the pressure falls. Pressure
values in the international standard atmosphere are given below:

ow 187.6 2/72 141


300.9 4.36 8.89 226
wOfWife
465.6 6.75 13.75 349
696.8 10.11 20.58 523
14.7 760

From the table it should be noted that at about 18,000 ft the pressure is half the sea level value.

Also, it should now be apparent that the rate of pressure decrease with height is not constant. In
the first 10,000 ft the pressure falls at a rate of approximately 1 mb per 30 ft but between
30,000 ft and 40,000 ft the pressure decrease is closer to 1 mb per 88 ft.

Measuring Gas Pressure and Atmospheric Pressure


https://youtu.be/GjWs_MzoLbA

2-142 Total Training Support Ltd


issue 2 - September 2016 © Copyright 2016
Module 2,2 Mechanics
Module 2.2 Mechanics
Atmospheric Pressure
https ://youtu. be/xJ H Js A7 b YGc

' \ \ Column of air

\\W

The mass of a column of air produces a force at its


base, spread over a unit area

03)
14.7 lbs per square inch

Forces acting on liquids Forces acting on liquids

7?^-.^'7 T Aimpspheriephessure^units- ./?.7B77A7 ■77'• gggi


ggghpaggg ■S7j - ih’WG X<»;atm .:z:< p..:,;psi wnri^X:
1 IO'2 2.95 x 104 7.5 x 103 9.87 x 10~6 | 1.45 x 10'4
102 1 2.95X10-2 0.75 9.87 x 10'4 1.45 x 10“2
3.39 x 103 33.9 1 25.4 3.34x10-2 0.49
1.33 x 102 1.33 3.94X10-2 1 1.32 x 103 1.93 x IO'2
^gT'7^b-glg Standard
gSOiRS 147
' oiP.
sea
6:B'g"x'y0'3--" 2.04.... 51.Z........ 1
level

Total Training Support Ltd 2-143


© Copyright 2016 Module 2.2 Mechanics Issue 2 - September 2016
Module 2.2 Mechanics
Isobars
Isobars are lines on a weather map joining together places of equal atmospheric pressure. On
the map, the isobar marked 1,028 for example, represents an area of relatively high pressure,
while the isobar marked 976, represents an area of relatively low pressure.

Isobars are used on weather forecast maps as shown opposite. The numbers measure the
atmospheric pressure in millibars.

Usually isobars are drawn at intervals of two or four millibars. The closer the isobars are
together the windier it is.

2-144 Total Training Support Ltd


Issue 2 - September 2016
Module 2.2 Mechanics © Copyright 2016
Module 2.2 Mechanics

Isobars (lines of equal pressure) are used on a weather map (figures are mb or hPa).

Wind blows from high pressure to low pressure. The closer the isobars are to each other,
the stronger the wind.

Total Training Support Ltd 2-145


© Copyright 2016 Module 2.2 Mechanics Issue 2 - September 2016
Module 2.2 Mechanics
Cabin altitude
Cabin altitude is a term used to express cabin pressure in terms of equivalent altitude above
sea level. For example, a cabin altitude of 6,000 feet means that the pressure inside the aircraft
cabin is the same as the atmospheric pressure at an altitude of 6,000 feet. Looking at the
standard atmosphere table, the pressure is found to be 1,696 lb/ft2 which upon division by 144
gives the pressure in lb/in2 to be 11.78 lb/in2

At a cabin altitude of 8,000 feet, the passengers and crew can ride in relative comfort without
any special oxygen supply. Planes which fly at much higher altitudes than 8,000 feet must be
furnished with a special atmosphere control system. It is highly advantageous to fly at high
altitudes both for economy of fuel consumption, and the smooth air high above the level of
turbulent weather systems. At these high altitudes, the pressure outside the aeroplane can be
significantly lower than the cabin pressure.

At 8,000 ft, the standard atmosphere table tells us that the air pressure is 1,572 lb/ft2 or
10.92 lb/in2 This is the pressure that is normally maintained in the cabin even though the
aeroplane is flying at a higher altitude.

Suppose the aeroplane is flying at an altitude of 40,000 ft. At this altitude the pressure is
393 lb/ft2 or 2.73 lb/in2 This means that for a cabin altitude of 8,000 ft for an aeroplane flying at
40,000 ft, there is a net outward pressure of 8.19 lb/in2 This number was obtained by
subtracting 2.73 lb/in2 from 10.92. For a Learjet with a pressurized area of 45,000 in2, we are
dealing with a bursting force of over 368,000 lbs (8.19 x 45 thousand). In addition to being able
to withstand this much force, a safety factor of 1.33 is generally used by design engineers.
Therefore, the pressurized portion of the fuselage must be constructed to have an ultimate
strength of over 460 thousand pounds or about 230 tons. The challenge of finding lightweight
materials which can withstand these large forces is great.

In the description of an aircraft’s air conditioning and pressurization system, a differential


pressure is given. The differential pressure is the maximum difference between cabin pressure
and atmospheric pressure which the pressurization system can sustain. For example, the air-
cycle air-conditioning system of a Boeing 747 can maintain a pressure differential of 8.9 lb/in2.
This means that the system can maintain a cabin pressure 8.9 lb/in2 greater than the
atmospheric pressure surrounding the aeroplane. This also means that there is an upper limit
imposed by the pressurization system on the altitude at which the aeroplane can fly.

2-146 Total Training Support Ltd


Issue 2 - September 2016
Module 2.2 Mechanics ©Copyright 2016
A cabin altitude and differential pressure indicator

Flight altitude

Flight altitude .«■««— cabin altitude


Even though the aeroplane is flying at an actual altitude of 41,000 ft., the cabin
is pressurised such that its equivalent altitude is only 7,000 ft.

Total Training Support Ltd 2-147


©Copyright 2016 Module 2.2 Mechanics Issue 2 - September 2016
Module 2.2 Mechanics
Absolute pressure and gauge (relative) pressure
The barometer pressure readout, for a given pressure, depends on how the barometer is
designed. The atmospheric pressure is also called the ambient pressure (meaning
“surrounding” pressure). The barometer measured pressure is an absolute pressure reading.
The atmospheric pressure is perhaps 1 ATM and may be higher or lower than that, but can
never be lower than 0 ATM. Hence an absolute pressure reading can never be less than zero.

If the pressure inside an air-tank is measured, with the ambient pressure as the zero reference,
it is called a Gauge (or relative) measurement. The pressure inside the tank can either be larger
or lower than the ambient reference pressure. A higher pressure is positive whereas a lower
pressure - sometimes called a partial vacuum - is negative.

A gauge on a pressurized air cylinder measures the pressure above the ambient atmospheric
pressure. This relative pressure is called a gauge pressure. If the cylinder is empty the gauge
pressure is zero; not because the pressure inside the tank is zero, but because the pressure
differential (Pd) across the tank is zero. This means that the ambient pressure and the tank
pressure is the same.

A gauge pressure of 10 ATM tells us that the pressure inside the cylinder is 10 ATM above the
ambient pressure. The actual pressure that the gas is exerting on the walls of the cylinder is the
absolute pressure. The relationship between the gauge pressure (Pg), the atmospheric
pressure (Patm) and the absolute pressure (Pabs) is;
1*“%
r abs “ r atm + rg

Thus, the absolute pressure inside the cylinder is 11 ATM (assuming that the atmospheric
pressure is 1 ATM).

We could also say that the differential pressure is 10 ATM. If the ambient pressure is lowered
the differential pressure increases. If the gauge reads -0.5 ATM at the equal ambient conditions,
the absolute pressure inside the cylinder is 0.5 ATM - a partial vacuum relative to the ambient.

This can be appreciated when trying to measure the pressure inside a fiat tyre on a car using a
standard tyre pressure gauge. Of course it will indicate 0 psi, because the standard tyre
pressure gauge indicates gauge pressure. The actual pressure in the tyre is atmospheric
pressure (approximately 14.7 psi).

Differential pressure
Differential pressure is often of importance to the aircraft designer. For example, the strength of
a pressurized fuselage is dependent upon the pressure difference between the cabin (internal)
pressure and the ambient (external) pressure. This pressure difference is called differential
pressure.

Units
In some units, the pressure being measured is denoted by a letter “A”, or “G” or “D” after the unit
thus:

psiG = psi - Gauge


psiA = psi - Absolute
psiD = psi - Differential

Fluid Dynamics - Gauge and Absolute Rressure


https://youtu.be/bzaCFkptlll

2-148 Total Training Support Ltd


Issue 2 - September 2016
Module 2.2 Mechanics ©Copyright 2016
Module 2.2 Mechanics
ATM A
Gauge Absolute
pressure pressure
+ A A
Positive S
gauge j
Atmospheric pressure I
pressure 0
(reference
pressure) Negative
gauge
pressure
(partial
vacuum)

0
o
Gauge pressure and absolute
pressure use different zero data

A pressure gauge indicates


pressure above atmospheric
(gauge pressure)

Measuring a flat tyre’s pressure with a gauge


will show zero pressure. But the pressure is in
fact atmospheric pressure (147 psi approx.)

Total Training Support Ltd 2-149


©Copyright 2016 Module 2.2 Mechanics Issue 2 - September 2016
Module 2.2 Mechanics
Barometers
A barometer is an instrument used to measure atmospheric pressure. It can measure the
pressure exerted by the atmosphere by using water, air, or mercury.

Mercury barometers
A standard mercury barometer has a glass tube of about 30 inches (about 76 cm) in height,
closed at one end, with an open mercury-filled reservoir at the base. Mercury (chemical symbol
‘hg’) in the tube adjusts until the weight of the mercury column balances the atmospheric force
exerted on the reservoir. High atmospheric pressure places more force on the reservoir, forcing
mercury higher in the column. Low pressure allows the mercury to drop to a lower level in the
column by lowering the force placed on the reservoir.

Since higher temperature at the instrument will reduce the density of the mercury the scale for
reading the height of the mercury is adjusted to compensate for this effect.

The standard temperature for reading a mercury barometer is 0°C (32°F). A correction factor is
read from a graph and applied to the reading for temperatures above 0°C. The barometer over­
reads at higher temperatures.

The mercury barometer’s design gives rise to the expression of atmospheric pressure in inches
or millimetres: the pressure is quoted as the level of the mercury’s height in the vertical column.
1 atmosphere is equivalent to about 29.9 inches, or 760 millimetres, of mercury. Barometers of
this type normally measure atmospheric pressures between 28 and 31 inches of mercury.

The reading from a barometer (in mm Hg or in Hg) can be converted into Pascals or psi by
using the formula:

Pressure = p g h (if using metric units)

Or

Pressure = p h (if using imperial units)

Where:

Pressure = the converted pressure (Pa or psi)


p = density of the mercury (13,600 kg/m3 or 62.4 Ibf/ft3)
g = acceleration due to gravity (9.81 m/s2)
h = height of mercury (m or ft)

Barometer - Device to calculate air pressure


https://youtu.be/JdbWOAfXceA

2-150 Total Training Support Ltd


Issue 2 - September 2016 ©Copyright 2016
Module 2.2 Mechanics
Module 2.2 Mechanics

Vacuum

Mercury

p
Atmospheric
A
pressure
Principle of the mercury
mm.Hg barometer
(mm of mercury)
The force due to the atmospheric
pressure holds the column of
Tonie.' mercury in the tube. The weight of
the column of mercury is equal to
the force due to atmospheric
pressure over the same cross
sectional area.

Temperature correction chart for a mercury barometer

Total Training Support Ltd 2-151


© Copyright 2016 Module 2.2 Mechanics Issue 2 - September 2016
Module 2.2 Mechanics
Aneroid barometers
An aneroid barometer uses a small, flexible metal vacuum chamber called an aneroid ceil. This
aneroid capsule (cell) is made from an alloy of beryllium and copper. The evacuated capsule (or
usually more capsules) is prevented from collapsing by a strong spring. Small changes in
external air pressure cause the cell to expand or contract. This expansion and contraction
drives mechanical levers such that the tiny movements of the capsule are amplified and
displayed on the face of the aneroid barometer. Many models include a manually set needle
which is used to mark the current measurement so a change can be seen. In addition, the
mechanism is made deliberately ‘stiff’ so that tapping the barometer reveals whether the
pressure is rising or falling as the pointer moves. They are used for measuring atmospheric
pressure.

Aneroid barometer mechanism


https ://youtu. be/O55h h ntG h 1A

2-152 Total Training Support Ltd


Issue 2 “ September 2016
Module 2.2 Mechanics ©Copyright 2016
Module 2.2 Mechanics

Total Training Support Ltd 2-153


©Copyright 2016 Module 2.2 Mechanics Issue 2 - September 2016
Module 2.2 Mechanics
Humidity
Some water in the form of invisible vapour, is intermixed with the air throughout the atmosphere,
it is the condensation of this vapour which gives rise to most weather phenomena: clouds, rain,
snow, dew, frost and fog. There is a limit to how much water vapour the air can hold and this
limit varies with temperature. When the air contains the maximum amount of vapour possible for
a particular temperature, the air is said to be saturated. Warm air can hold more water vapour
than cold air. In general, the air is rarely saturated, containing only a fraction of the possible
water vapour.

Absolute humidity
The amount of vapour in the air can be measured in a number of ways. The humidity of a parcel
of air is usually denoted by the mass of vapour contained within it, or the pressure that the water
vapour exerts. This is the absolute humidity of air.

Thus absolute humidity is the mass of water vapour in a given volume of air (i.e., density of
water vapour in a given parcel), usually expressed in grams per cubic metre, or as a percentage
of the total mass of the air.

Relative humidity
Relative humidity is measured by comparing the actual mass of vapour in the air to the mass
of vapour in saturated air at the same temperature. For example, air at 10°C contains 9.4 g/m3
(grams per cubic metre) of water vapour when saturated. If air at this temperature contains only
4.7 g/m3 of water vapour, then the relative humidity is 50%.

Thus, relative humidity is he amount of water vapour actually in the air divided by the amount of
water vapour the air can hold at that same temperature.

Dew point
When unsaturated air is cooled, relative humidity increases. Eventually it reaches a temperature
at which it is saturated. Relative humidity is 100%. Further cooling leads to condensation of the
excess water vapour. The temperature at which condensation sets in is called the dew point.

Relative Humidity vs Dewpoint


https://youtu.be/OiejHVHrdOo

2-154 Total Training Support Ltd


Issue 2 - September 2016
Module 2.2 Mechanics © Copyright 2016
Module 2.2 Mechanics

Low temperature Medium temperature High temperature


low moisture content medium moisture content high moisture content

Warm air holds more moisture (moisture droplets shown enlarged for indication)

Dew is condensed moisture from air.

The dew point is the temperature to which the


air must be cooled to make the relative
humidity 100% (saturated).

Temperatures below the dew point


temperature causes precipitation of the
moisture from the air.

Maximum possible water vapor Actual water vapor

Relative humidity and dew point

Total Training Support Ltd 2-155


©Copyright 2016 Module 2.2 Mechanics Issue 2 - September 2016
Module 2.2 Mechanics
Measuring humidity
A hygrometer is an instrument used for measuring the moisture content in the atmosphere.

The dew point, and other measures of humidity can be calculated from readings taken by a
hygrometer. A hygrometer has two thermometers, one dry bulb or standard air temperature
thermometer, and one wet bulb thermometer. The wet bulb thermometer is an ordinary
thermometer which has the bulb covered with a muslin bag, kept moist via an absorbent wick
dipped into water. Evaporation of water from the muslin lowers the temperature of the
thermometer. The difference between wet and dry bulb temperatures is used to calculate the
various measures of humidity.

Wet bulb temperature: A psychrometer, or wet-and-dry-bulb thermometer, is a special type of


hygrometer. It consists of two thermometers, one that is dry and one that is kept moist with
distilled water on a sock or wick. The two thermometers are thus called the dry-bulb and the
wet-bulb. At temperatures above the freezing point of water, evaporation of water from the wick
lowers the temperature, so that the wet-bulb thermometer usually shows a lower temperature
than that of the dry-bulb thermometer.

Relative humidity is computed from the ambient temperature as shown by the dry-bulb
thermometer and the difference in temperatures as shown by the wet-bulb and dry-bulb
thermometers. Relative humidity can also be determined by locating the intersection of the wet
and dry-bulb temperatures on a psychrometric chart.

The two thermometers coincide when the air is fully saturated, and the greater the difference
the drier the air. Psychrometers are commonly used in meteorology.

2-156 Total Training Support Ltd


Issue 2 - September 2016 © Copyright 2016
Module 2.2 Mechanics
Module 2.2 Mechanics

A hygrometer indicates relative


humidity in percentage. The green
range indicates the zone of
humidity which humans find most
comfortable.

Relative humidity lower than the


green range will result in a dry
throat and mouth.

Relative humidity greater than the


green range will result in a
propensity to perspire profusely.

A typical Psychrometer.

Wet bulb and dry bulb


temperatures are used
to calculate humidity

Total Training Support Ltd 2-157


©Copyright 2016 Module 2.2 Mechanics Issue 2 - September 2016
Module 2.2 Mechanics
Pressure and force (hydrostatics)

The terms force and pressure are used extensively in the study of fluids. It is essential that we
distinguish between the terms. Force means a total push or pull. It is the push or pull exerted
against the total area of a particular surface and is expressed in pounds or grams. Pressure
means the amount of push or pull (force) applied to each unit area of the surface and is
expressed in pounds per square inch (lb/in2) or Newtons per square metre (N/m2). Pressure
maybe exerted in one direction, in several directions, or in ail directions.

Computing force, pressure, and area


A formula is used in computing force, pressure, and area in fluid power systems. In this formula,
P refers to pressure, F indicates force, and A represents area. Force equals pressure times
area. Thus, the formula is written:

F = PxA

Pressure equals force divided by area. By rearranging the formula, this statement may be
condensed into:

Since area equals force divided by pressure, the formula is written:

The figure opposite illustrates a memory aid for recalling the different variations of this formula.
Any letter in the triangle may be expressed as the product or quotient of the other two,
depending on its position within the triangle.

For example, to find area, consider the letter A as being set off to itself, followed by an equal
sign. Now look at the other two letters. The letter F is above the letter P; therefore,

Note: Sometimes the area may not be expressed in square units. If the surface is rectangular,
you can determine its area by multiplying its length (say, in inches) by its width (also in
inches). The majority of areas you will consider in these calculations are circular in
shape. Either the radius or the diameter may be given, but you must know the radius in
inches to find the area. The radius is one-half the diameter. To determine the area, use
the formula for finding the area of a circle. This is written A = nr2, where A is the area, p is
3.1416 (3.14 or 22/7 for most calculations), and r2 indicates the radius squared.

2-158 Total Training Support Ltd


Issue 2 - September 2016 © Copyright 2016
Module 2.2 Mechanics
Module 2.2 Mechanics

The pressure exerted on the piston extends uniformiy


throughout the liquid, causing it to push outward with equal
force per unit area on the walls and the bottom of the cylinder.

Total Training Support Ltd 2-159


© Copyright 2016 Module 2.2 Mechanics Issue 2 - September 2016
Module 2.2 Mechanics
Pressure and forces in Biquids
When the end of a solid bar is struck, the main force of the blow is carried straight through the
bar to the other end. This happens because the bar is rigid. The direction of the blow almost
entirely determines the direction of the transmitted force.

When a force is applied to the end of a column of confined liquid, it is transmitted straight
through to the other end and also equally and undiminished in every direction throughout the
column - forward, backward, and sideways - so that the containing vessel is literally filled with
pressure.

An example of this distribution of force is illustrated in the diagram of the water hose. The
outward push of the water is equal in every direction.

So far we have explained the effects of atmospheric pressure on liquids and how external
forces are distributed through liquids. Let us now focus our attention on forces generated by the
weight of liquids themselves. To do this, we must first discuss Pascal’s law.

2-160 Total Training Support Ltd


Issue 2 - September 2016 ©Copyright 2016
Module 2.2 Mechanics
Module 2.2 Mechanics

Forces acting on solids and liquids

Flat and water filled hoses

Total Training Support Ltd 2-161


© Copyright 2016 Module 2.2 Mechanics Issue 2 - September 2016
Module 2.2 Mechanics
Pascal’s law
The foundation of modern hydraulics was established when Pascal discovered that pressure in
a fluid acts equally in all directions. This pressure acts at right angles to the containing surfaces.
If some type of pressure gauge, with an exposed face, is placed beneath the surface of a liquid
(figure opposite) at a specific depth and pointed in different directions, the pressure will read the
same. Thus, we can say that pressure in a liquid is independent of direction.

Pressure due to the weight of a liquid, at any level, depends on the depth of the fluid from the
surface. If the exposed face of the pressure gauges, are moved closer to the surface of the
liquid, the indicated pressure will be less. When the depth is doubled, the indicated pressure is
doubled. Thus the pressure in a liquid is directly proportional to the depth.

Consider a container with vertical sides (figure opposite, bottom) that is


1 Toot-long and 1 foot wide. Let it be filled with water 1-foot-deep, providing 1 cubic foot of
water. 1 cubic foot of water weighs 62.4 pounds. Using this information and the equation,
P = F/A, we can calculate the pressure on the bottom of the container.
FT
P=L
A

„ 62.4 lb
1 ft2

= 62.4 Ib/ft2
Since there are 144 square inches in 1 square foot,

p = ^1 = 0.433 lb/in2 (psi)


'144
This can be stated as follows: the weight of a column of water
1-foot-high, having a cross-sectional area of 1 square inch, is 0.433 pound. If the depth of the
column is tripled, the weight of the column will be 3 x 0.433, or 1.299 pounds, and the pressure
at the bottom will be 1.299 lb/in2 (psi), since pressure equals the force divided by the area.
Thus, the pressure at any depth in a liquid is equal to the weight of the column of liquid at that
depth divided by the cross-sectional area of the column at that depth. The volume of a liquid
that produces the pressure is referred to as the fluid head of the liquid. The pressure of a liquid
due to its fluid head is also dependent on the density of the liquid.

If we let A equal any cross-sectional area of a liquid column and h equal the depth of the
column, the volume becomes Ah. Using the equation for density, D = weight (W)Zvolume (V),
the weight of the liquid above area A is equal to AhD.

V Ah
W ~ AhD

Pressure and Pascal’s principle (part 1) I Fluids I Physics I Khan Academy


https://youtu.be/Pn5YEMwQb4Y

2-162 Total Training Support Ltd


Issue 2 - September 2016 © Copyright 2016
Module 2.2 Mechanics
Module 2.2 Mechanics

Pascal’s law

Totai Training Support Ltd 2-163


© Copyright 2016 Moduie 2.2 Mechanics Issue 2 - September 2016
Module 2.2 Mechanics
Since pressure is equal to the force per unit area, set A equal to 1. Then the formula pressure
becomes:

P-hD

It is essential that h and D be expressed in similar units. That is, if D is expressed in pounds per
cubic foot, the value of h must be expressed in feet. If the desired pressure is to be expressed
in pounds per square inch, the pressure formula, becomes:

Pressure and force in fluid power systems


Pascal was also the first to prove by experiment that the shape and volume of a container in no
way alters pressure. Thus, in the figure opposite (top), if the pressure due to the weight of the
liquid at a point on horizontal line H is 8 psi, the pressure is 8 psi everywhere at level H in the
system. The equation P=F/A also shows that the pressure is independent of the shape and
volume of a container. If there is a resistance on the output piston and the input piston is
pushed downward, a pressure is created through the fluid, which acts equally at right angles to
surfaces in all parts of the container. If force 1 is 100 pounds and the area of the input piston is
10 square inches, then the pressure in the fluid is 10 psi.
100 lb
Win2
Recall that, according to Pascal’s law, any force applied to a confined fluid is transmitted in all
directions throughout the fluid.

Note: Fluid pressure cannot be created without resistance to flow. In this case, resistance is
provided by the equipment to which the output piston is attached. The force of resistance
acts against the top of the output piston. The pressure created in the system by the input
piston pushes on the underside of the output piston with a force of 10 pounds on each
square inch.

In this case (see figure opposite, centre), the fluid column has a uniform cross section, so the
area of the output piston is the same as the area of the input piston, or 10 square inches.
Therefore, the upward force on the output piston is 100 pounds (10 psi x 10 in2), the same as
the force applied to the input piston. Ail that was accomplished in this system was to transmit
the 100-pound force around the bend. However, this principle underlies practically all
mechanical applications of fluid power.

At this point you should note that since Pascal’s law is independent of the shape of the
container, it is not necessary that the tube connecting the two pistons have the same cross-
sectional area of the pistons, A connection of any size, shape, or length will do, as long as an
unobstructed passage is provided. Therefore, the system shown in the figure opposite (bottom),
with a relatively small, bent pipe connecting two cylinders, will act exactly the same as the
system shown in the figure above it.

2-164 Total Training Support Ltd


Issue 2 - September 2016
Module 2,2 Mechanics © Copyright 2016
Module 2.2 Mechanics

Hydrostatic pressure in different shaped containers

Forces, pistons and pressure

Force 1-100 Ibs Force 2-100 Ibs

Pressure due to forces on pistons

Total Training Support Ltd 2-165


© Copyright 2016 Module 2.2 Mechanics Issue 2 - September 2016
Module 2.2 Mechanics
The hydraulic ram principle
Consider the situation in the figure opposite, where the input piston is much smaller than the
output piston. Assume that the area of the input piston is 2 square inches. With a resistant force
on the output piston a downward force of 20 pounds acting on the input piston creates a
pressure 10 psi in the fluid.

Although this force is much smaller than the force applied in the figures above, the pressure is
the same. This is because the force is applied to a smaller area.

This pressure of 10 psi acts on all parts of the fluid container, including the bottom of the output
piston. The upward force on the output piston is 200 pounds (10 pounds of pressure on each
square inch). In this case, the original force has been multiplied tenfold while using the same
pressure in the fluid as before. In any system with these dimensions, the ratio of output force to
input force is always ten to one, regardless of the applied force. For example, if the applied
force of the input piston is 50 pounds, the pressure in the system will be 25 psi. This will support
a resistant force of 500 pounds on the output piston.

The system works the same in reverse. If we change the applied force and place a 200-pound
force on the output piston, making it the input piston, the output force on the input piston will be
one-tenth the input force, or 20 pounds (sometimes such results are desired). Therefore, if two
pistons are used in a fluid power system, the force acting on each piston is directly proportional
to its area, and the magnitude of each force is the product of the pressure and the area of each
piston.

Note the white arrows at the bottom of the figure opposite that indicate up and down movement
(*). The movement they represent will be explained later in the discussion of volume and
distance factors.

Pressure and Pascal’s principle (part 2) I Fluids I Physics I Khan Academy


https://youtu.be/IWDtFHDVqqk

2-166 Totai Training Support Ltd


Issue 2 - September 2016 ©Copyright 2016
Moduie 2.2 Mechanics
Module 2.2 Mechanics

Force 1 - 20 lbs Force 2 - 200 lbs

Input Output
piston piston
2 in2 20 in2

Pressure
10 lbs/in2

The hydraulic ram principle

Total Training Support Ltd 2-167


© Copyright 2016 Module 2.2 Mechanics Issue 2 - September 2016
Module 2.2 Mechanics
Differential areas
Consider the special situation shown in the figure opposite. Here, a single piston (1) in a
cylinder (2) has a piston rod (3) attached to one of its sides. The piston rod extends out of one
end of the cylinder. Fluid under pressure is admitted equally to both ends of the cylinder. The
opposed faces of the piston (1) behave like two pistons acting against each other. The area of
one face is the full cross-sectional area of the cylinder, say 6 square inches, while the area of
the other face is the area of the cylinder minus the area of the piston rod, which is 2 square
inches. This leaves an effective area of 4 square inches on the right face of the piston. The
pressure on both faces is the same, in this case, 20 psi. Applying the rule just stated, the force
pushing the piston to the right is its area times the pressure, or 120 pounds (20 x 6). Likewise,
the force pushing the piston to the left is its area times the pressure, or 80 pounds (20 x 4).
Therefore, there is a net unbalanced force of 40 pounds acting to the right, and the piston will
move in that direction. The net effect is the same as if the piston and the cylinder had the same
cross-sectional area as the piston rod.

Volume and distance factors


You have learned that if a force is applied to a system and the cross-sectional areas of the input
and output pistons are equal, as in the figures above, the force on the input piston will support
an equal resistant force on the output piston. The pressure of the liquid at this point is equal to
the force applied to the input piston divided by the piston’s area. Let us now look at what
happens when a force greater than the resistance is applied to the input piston.

In the system illustrated in the second figure opposite, assume that the resistance force on the
output piston is 100 psi. If a force slightly greater than 100 pounds is applied to the input piston,
the pressure in the system will be slightly greater than 10 psi. This increase in pressure will
overcome the resistance force on the output piston. Assume that the input piston is forced
downward 1 inch. The movement displaces 10 cubic inches of fluid. The fluid must go
somewhere. Since the system is closed and the fluid is practically incompressible, the fluid will
move to the right side of the system. Because the output piston also has a cross-sectional area
of 10 square inches, it will move 1 inch upward to accommodate the 10 cubic inches of fluid.
You may generalize this by saying that if two pistons in a closed system have equal cross-
sectional areas and one piston is pushed and moved, the other piston will move the same
distance, though in the opposite direction. This is because a decrease in volume in one part of
the system is balanced by one equal increase in volume in another part of the system.

Apply this reasoning to the system in the bottom figure opposite. If the input piston is pushed
down a distance of 1 inch, the volume of fluid in the left cylinder will decrease by 2 cubic inches.
At the same time, the volume in the right cylinder will increase by 2 cubic inches. Since the
diameter of the right cylinder cannot change, the piston must move upward to allow the volume
to increase. The piston will move a distance equal to the volume increase divided by the surface
area of the piston (equal to the surface area of the cylinder). In this example, the piston will
move one-tenth of an inch (2 in3 4- 20 in2). This leads to the second basic rule for a fluid power
system that contains two pistons: The distances the pistons move is inversely proportional to
the areas of the pistons. Or more simply, if one piston is smaller than the other, the smaller
piston must move a greater distance than the larger piston any time the pistons move.

2-168 Total Training Support Ltd


Issue 2 - September 2016
Module 2.2 Mechanics © Copyright 2016
Module 2.2 Mechanics

Pressures on
unequal areas

Area 6 in2 Area 2 in2

Total Training Support Ltd 2-169


©Copyright 2016 Module 2.2 Mechanics Issue 2 - September 2016
Module 2.2 Mechanics
Relationship between force, pressure, and head
In dealing with fluids, forces are usually considered in relation to the areas over which they are
applied. As previously discussed, a force acting over a unit area is a pressure, and pressure
can alternately be stated in pounds per square inch or in terms of head, which is the vertical
height of the column of fluid whose weight would produce that pressure.

In most of the applications of fluid power, applied forces greatly outweigh all other forces, and
the fluid is entirely confined. Under these circumstances it is customary to think of the forces
involved in terms of pressures. Since the term head is encountered frequently in the study of
fluid power, it is necessary to understand what it means and how it is related to pressure and
force.

At this point you need to review some terms in general use. ‘Gravity head’, when it is important
enough to be considered, is sometimes referred to as simply ‘head’. The effect of atmospheric
pressure is referred to as ‘atmospheric pressure’. (Atmospheric pressure is frequently and
improperly referred to as suction.) inertia effect, because it is always directly related to velocity,
is usually called ‘velocity head’; and friction, because it represents a loss of pressure or head, is
usually referred to as ‘friction head’.

Static and dynamic factors


Gravity, applied forces, and atmospheric pressure are static factors that apply equally to fluids
at rest or in motion, while inertia and friction are dynamic factors that apply only to fluids in
motion. The mathematical sum of gravity, applied force, and atmospheric pressure is the static
pressure obtained at any one point in a fluid at any given time. Static pressure exists in addition
to any dynamic factors that may also be present at the same time.

Remember, Pascal’s law states that a pressure set up in a fluid acts equally in all directions and
at right angles to the containing surfaces. This covers the situation only for fluids at rest or
practically at rest. It is true only for the factors making up static head. Obviously, when velocity
becomes a factor it must have a direction, and as previously explained, the force related to the
velocity must also have a direction, so that Pascal’s law alone does not apply to the dynamic
factors of fluid power.

The dynamic factors of inertia and friction are related to the static factors. Velocity head and
friction head are obtained at the expense of static head. However, a portion of the velocity head
can always be reconverted to static head. Force, which can be produced by pressure or head
when dealing with fluids, is necessary to start a body moving if it is at rest, and is present in
some form when the motion of the body is arrested; therefore, whenever a fluid is given velocity,
some part of its original static head is used to impart this velocity, which then exists as velocity
head.

Operation of hydraulic components


To transmit and control power through pressurized fluids, an arrangement of inter-connected
components is required. Such an arrangement is commonly referred to as a system. The
number and arrangement of the components vary from system to system, depending on the
particular application. In many applications, one main system supplies power to several
subsystems, which are sometimes referred to as circuits. The complete system may be a small
compact unit; more often, however, the components are located at widely separated points for
convenient control and operation of the system.

The basic components of a fluid power system are essentially the same, regardless of whether
the system uses a hydraulic or a pneumatic medium. There are five basic components used in a
system.
2-170 Total Training Support Ltd
Issue 2 - September 2016
Module 2.2 Mechanics © Copyright 2016
Module 2.2 Mechanics
These basic components are as follows:

• Reservoir or receiver
• Pump or compressor
• Lines (pipe, tubing, or flexible hose)
• Directional control valve
• Actuating device

Several applications of fluid power require only a simple system; that is, a system which uses
only a few components in addition to the five basic components. A few of these applications are
presented in the following paragraphs. We will explain the operation of these systems briefly at
this time so you will know the purpose of each component and can better understand how
hydraulics is used in the operation of these systems.

Total Training Support Ltd 2-171


© Copyright 2016 Module 2.2 Mechanics Issue 2 - September 2016
Mechanics
Hydraulic jack
The hydraulic jack is perhaps one of the simplest forms of a fluid power system. By moving the
handle of a small device, an individual can lift a load weighing several tons. A small initial force
exerted on the handle is transmitted by a fluid to a much larger area. To understand this better,
study figure opposite. The small input piston has an area of 5 square inches and is directly
connected to a large cylinder with an output piston having an area of 250 square inches. The
top of this piston forms a lift platform. If a force of 25 pounds is applied to the input piston, it
produces a pressure of 5 psi in the fluid, that is, of course, if a sufficient amount of resistant
force is acting against the top of the output piston. Disregarding friction loss, this pressure
acting on the 250 square inch area of the output piston will support a resistance force of
1,250 pounds. In other words, this pressure could overcome a force of slightly under
1,250 pounds. An input force of 25 pounds has been transformed into a working force of more
than half a ton; however, for this to be true, the distance travelled by the input piston must be
50 times greater than the distance travelled by the output piston. Thus, for every inch that the
input piston moves, the output piston will move only one-fiftieth of an inch.

This would be ideal if the output piston needed to move only a short distance. However, in most
instances, the output piston would have to be capable of moving a greater distance to serve a
practical application. The device shown in the figure opposite is not capable of moving the
output piston farther than that shown; therefore, some other means must be used to raise the
output piston to a greater height.

The output piston can be raised higher and maintained at this height if additional components
are installed as shown opposite. In this illustration the jack is designed so that it can be raised,
lowered, or held at a constant height. These results are attained by introducing a number of
valves and also a reserve supply of fluid to be used in the system.

Notice that this system contains the five basic components-the reservoir; cylinder 1, which
serves as a pump; valve 3, which serves as a directional control valve; cylinder 2, which serves
as the actuating device; and lines to transmit the fluid to and from the different components. In
addition, this system contains two valves, 1 and 2, whose functions are explained in the
following discussion.

As the input piston is raised, view (A), valve 1 is closed by the back pressure from the weight of
the output piston. At the same time, valve 2 is opened by the head of the fluid in the reservoir.
This forces fluid into cylinder 1. When the input piston is lowered, view (B), a pressure is
developed in cylinder 1. When this pressure exceeds the head in the reservoir, it closes valve 2.
When it exceeds the back pressure from the output piston, it opens valve 1, forcing fluid into the
pipeline. The pressure from cylinder 1 is thus transmitted into cylinder 2, where it acts to raise
the output piston with its attached lift platform. When the input piston is again raised, the
pressure in cylinder 1 drops below that in cylinder 2, causing valve 1 to close. This prevents the
return of fluid and holds the output piston with its attached lift platform at its new level. During
this stroke, valve 2 opens again allowing a new supply of fluid into cylinder 1 for the next power
(downward) stroke of the input piston. Thus, by repeated strokes of the input piston, the lift
platform can be progressively raised. To lower the lift platform, valve 3 is opened, and the fluid
from cylinder 2 is returned to the reservoir.

2-172 Total Training Support Ltd


Issue 2 ~ September 2016 © Copyright 2016
Module 2.2 Mechanics
Module 2.2 Mechanics

1,250 lbs

Output
piston
S| 250 in2
lliBSBIIS
. •••-?• -•=• 'J’: • •-'. ■■■??<■7• • '■: ■ '■•

SS&Bt
S|

5 Ibs/in2

A simple hydraulic jack

Input
S piston
<--- Output < 5 in2 22"
r'< ■ •

. . ... . piston [• | 2'T;i.

BW8
UJ
IIS
.. f ................. ;.

(A)

Input
piston
Output 25 lbs 5 in2
piston A
250 in2 V ....lUl®, ..
Cylinder (1)
SS2BS .J;?-:

Valve (1)
Valve (3)
5 Ibs/in2 Valve (2)
-'I' r’■!' -',. ,'; •-:•'• • - :• I, , 1 .' - ?'-' . I- j ? ■! a'.-

.. ................. /i.'r.-. —... .....

(B)

A hydraulic jack with valves

Total Training Support Ltd 2-173


©Copyright 2016 Module 2.2 Mechanics Issue 2 - September 2016
Module 2.2 Mechanics
Hydraulic brakes
The hydraulic brake system used in the car is a multiple piston system. A multiple piston system
allows forces to be transmitted to two or more pistons in the manner indicated in the figure
opposite. Note that the pressure set up by the force applied to the input piston (1) is transmitted
undiminished to both output pistons (2 and 3), and that the resultant force on each piston is
proportional to its area. The multiplication of forces from the input piston to each output piston is
the same as that explained earlier.

The hydraulic brake system from the master cylinders to the wheel cylinders on most
automobiles operates in a way similar to the system illustrated in the second figure opposite.

When the brake pedal is depressed, the pressure on the brake pedal moves the piston within
the master cylinder, forcing the brake fluid from the master cylinder through the tubing and
flexible hose to the wheel cylinders. The wheel cylinders contain two opposed output pistons,
each of which is attached to a brake shoe fitted inside the brake drum.

Each output piston pushes the attached brake shoe against the wall of the brake drum, thus
retarding the rotation of the wheel. When pressure on the pedal is released, the springs on the
brake shoes return the wheel cylinder pistons to their released positions. This action forces the
displaced brake fluid back through the flexible hose and tubing to the master cylinder.

The force applied to the brake pedal produces a proportional force on each of the output
pistons, which in turn apply the brake shoes frictionally to the turning wheels to retard rotation.

As previously mentioned, the hydraulic brake systems on most cars operate in a similar way. It
is beyond the scope of this manual to discuss the various brake systems.

2-174 Total Training Support Ltd


Issue 2 - September 2016 © Copyright 2016
Module 2.2 Mechanics
Module 2.2 Mechanics

Piston (3)
300 in2

Wheel cylinders

Hydraulic brake
system |

Adisasembled drum brake

Total Training Support Ltd 2-175


©Copyright 2016 Module 2.2 Mechanics Issue 2 - September 2016
Module 2.2 Mechanics
Accumulators
An accumulator is a pressure storage reservoir in which hydraulic fluid is stored under pressure
from an external source. The storage of fluid under pressure serves several purposes in
hydraulic systems.

In some aircraft hydraulic systems, it is necessary to maintain the system pressure within a
specific pressure range for long periods of time. It is very difficult to maintain a closed system
without some leakage, either external or internal. Even a small leak can cause a decrease in
pressure. By using an accumulator, leakage can be compensated for and the system pressure
can be maintained within an acceptable range for long periods of time.

Accumulators also damp out fluctuations in pressure due to the operation of services such as
control surfaces and landing gear. They can supply extra pressure when all the hydraulic
services are being operated at one time (flaps, control surfaces, landing gear etc.) and when the
hydraulic pump is unable to cope. They can also be used in an emergency when all other
hydraulic power pressure supplies (pumps etc.) have failed. Thus a large modern aircraft can be
controlled on accumulator power alone, for up to an hour.

Accumulators also compensate for thermal expansion and contraction of the liquid due to
variations in temperature.

The accumulator consists of a gas chamber, which is charged with air or nitrogen. This is called
the pre-charge pressure and is usually about 1,000 psi. This pressure is measured when there
is no hydraulic pressure. The air chamber is the under-side of the piston shown opposite. With
no hydraulic pressure, the air/nitrogen pressure will push the piston to the top of the
accumulator.

A pressure gauge may be attached to the accumulator to indicate the air/nitrogen pressure.
When the hydraulic pumps are switched on, the hydraulic pressure (acting on top of the piston,
in opposition to the air/nitrogen pressure) begins to rise. When the hydraulic pressure exceeds
the air/nitrogen pre-charge pressure (typically 1,000 psi), the piston will begin to move down
and further compress the air/nitrogen pressure.

At all times that the hydraulic pressure is above the air/nitrogen pre-charge pressure of
1,000 psi, the air/nitrogen and the hydraulic pressures are equal. Thus when the hydraulic
pressure has reached its working level of 3,000 psi, the air/nitrogen pressure is also 3,000 psi.

It is the additional pressure supplied to the air/nitrogen by the hydraulic pressure, which can be
used to feed back the pressure to the hydraulic fluid if the hydraulic fluid pressure falls below
that of the air/nitrogen. However, when the air/nitrogen gauge indicates 1,000 psi, the hydraulic
pressure is zero, since the air/nitrogen has expanded back to its original pre-charge pressure.

2-176 Total Training Support Ltd


Issue 2 - September 2016
Module 2.2 Mechanics ©Copyright 2016
Module 2.2 Mechanics

To hydraulic
system

Hydraulic accumulator

Total Training Support Ltd 2-177


©Copyright 2016 Module 2.2 Mechanics Issue 2 - September 2016
Module 2.2 Mechanics

intentionally Blank

2-178 Total Training Support Ltd


Issue 2 - September 2016
Module 2.2 Mechanics ©Copyright 2016
Module 2.2 Mechanics
Worksheet
1. Calculate the pressure on a gas when a force of 3,100 N is exerted on a piston of diameter
2 cm.

2. Calculate the force exerted when a pressure of 1 bar acts on a piston of diameter 8 cm
which has a piston rod of diameter 2 cm taking some of the piston area.

The piston face area in the hydraulic jack shown is 0.3 in2. The rod cross-sectional area is
0.1 in2. Calculate the force and direction the ram rod will move if a pressure of 12 psi enters
equally into both sides of the cylinder chamber.

4. A brake master cylinder has a piston diameter of 0.4 inches. It feeds pressure to 4 identical
wheel cylinders, each having just one piston of diameter 2 inches. What is the force on one
wheel-brake when the driver applies a force of 80 lbs to the master cylinder?

5. A hydraulic accumulator is charged with nitrogen to 600 psi. The hydraulic pump is then
switched on and it feeds 3,000 psi to the other side of the accumulator piston. What will be
the new pressure on the nitrogen side of the accumulator?

Totai Training Support Ltd 2-179


© Copyright 2016 Module 2.2 Mechanics Issue 2 - September 2016
Module 2.2 Mechanics

Intentionally Blank

2-180 Total Training Support Ltd


Issue 2 - September 2016
Module 2.2 Mechanics © Copyright 2016
Module 2.2 Mechanics
Answers

1. 10 MPa

2. 450 N

3. 1.2 lbs, right

4. 2,000 ibs

5. 3,000 psi

Total Training Support Ltd 2-181


© Copyright 2016 Module 2.2 Mechanics Issue 2 - September 2016
Module 2.2 Mechanics
Fluid pressure and depth
This section will explore pressure and depth. If a fluid is within a container, then the depth of an
object placed in that fluid can be measured. The deeper the object is placed in the fluid, the
more pressure it experiences. This is because is the weight of the fluid above it. The more
dense the fluid above it, the more pressure is exerted on the object that is submerged, due to
the weight of the fluid.

The formula that gives the P pressure on an object submerged in a fluid is:

P=pgh

Where:

p (rho) is the density of the fluid,


g is the acceleration due to gravity
h is the height of the fluid above the object

If the container is open to the atmosphere above, the added pressure must be included if one is
to find the total pressure on an object. The total pressure is the same as absolute pressure on
pressure readings, while the gauge pressure is the same as the fluid pressure alone, not
including atmospheric pressure.
Ptotal = Patmosphere + Pflurd

Ptotal = Patmosphere + p g h

Pressure at a depth in a fluid I Fluids I Physics I Khan Academy


https://youtu.be/5EWjlpc0S00

Finding height of fluid in a barometer I Fluids I Physics I Khan Academy


https://youtu.be/i6gz9VFyYks

Example:

Find the pressure on a scuba diver when she is 12 meters below the surface of the ocean.
Assume standard atmospheric conditions.

The density of sea water is 1.03 x 103 kg/m3 and the atmospheric pressure is 1.01 x 105 N/m2.

Ptiuid = p g h = (1.03 x103 kg/m3) (9.8 m/s2) (12 m)

= 1.21 x 105 N/m2 (Pa)

Ptotal Patmosphere + Pfluid = (1.01 X 103) + (1.21 X 103)

= 2.22 x 102 kPa

2-182 Total Training Support Ltd


Issue 2 - September 2016 © Copyright 2016
Module 2.2 Mechanics
Module 2.2 Mechanics

Due to weight of liquid above, the


pressure increases with depth
Pressure at depth h:
P~pgh

Due to weight of liquid above, the pressure increases with depth.


In water, at 10 m depth, the pressure experienced by a diver is
two times the atmospheric pressure at the surface

Total Training Support Ltd 2-183


© Copyright 2016 Module 2.2 Mechanics Issue 2 - September 2016
Module 2.2 Mechanics

Intentionally Blank

2-184 Total Training Support Ltd


Issue 2 - September 2016
Module 2,2 Mechanics ©Copyright 2016
Module 2.2 Mechanics WjB
Worksheet

1. What is the pressure experienced at a point on the bottom of a nine-meter-deep lake? The
density of water is 1,000 kg/m3.

Take atmospheric pressure = 101,000 Pa.

2. The interior of a submarine located at a depth of 45 m is maintained at normal atmospheric


conditions. Find the total force exerted on a 20 cm by 20 cm square window. Use the
density of sea water given above.

3. How many atmospheres is a depth of 100 m of ocean water?

Total Training Support Ltd 2-185


©Copyright 2016 Module 2.2 Mechanics Issue 2 - September 2016
Module 2.2 Mechanics

intentionally Blank

2-186 Total Training Support Ltd


Issue 2 - September 2016
Module 2.2 Mechanics © Copyright 2016
Module 2.2 Mechanics
Answers

1. 189 kP

2. 1.81 x 104 N

3. 11 atm

Total Training Support Ltd 2-187


© Copyright 2016 Module 2,2 Mechanics Issue 2 - September 2016
Module 2.2 Mechanics
Buoyancy In liquids
A fluid is a substance that flows easily. Gases and liquids are fluids, although sometimes the
dividing line between liquids and solids is not always clear. Because of their ability to flow, fluids
can exert buoyant forces, multiply forces in hydraulic systems, allow aircraft to fly and ships to
float.

Archimedes principle
Archimedes was a Greek philosopher and mathematician who lived about 250BC. There is a
story (maybe even true) about Archimedes that every physics student should hear. It goes as
follows:

The king who ruled Greece at that time asked his royal metalworkers to make him a
gold crown. When the crown was delivered it was indeed beautiful. However, the king
suspected that the crown was not pure gold. He did not want to destroy the crown but
he wanted to know if he had been cheated. What he needed was some type of non­
destructive evaluation (NDE dates back many years!). He asked Archimedes to solve
his problem. Archimedes pondered the question. The density (mass/volume) of gold
was well known. He knew of course how to determine the weight and mass of the crown
by simple weighing. However, since the crown did not have a regular shape it was
impossible to determine the volume by a mathematical calculation. The solution came
to Archimedes one day when his servant filled his bathtub too full. As Archimedes
stepped into his bath, he noticed that a volume of water equal to his volume overflowed!
With a flash of insight, he ran through Athens, stark naked, shouting “Eureka, Eureka,
I have the solution!” The experiment was performed, the king was notified that his crown
was not pure gold and the royal metal workers lost their lives.

The point of the story is that a body submerged in a liquid displaces a volume of water equal to
its own volume. A corollary is that a body that floats in a liquid displaces a volume of liquid less
than its volume since some portion of the body is above the water level.

Archimedes’ principle, states that a body immersed in a fluid is buoyed up by a force equal to
the weight of the displaced fluid. The principle applies to both floating and submerged bodies
and to all fluids, i.e., liquids and gases. It explains not only the buoyancy of ships and other
vessels in water but also the rise of a balloon in the air and the apparent loss of weight of
objects underwater.

In determining whether a given body will float in a given fluid, both weight and volume must be
considered; that is, the relative density, or weight per unit of volume, of the body compared to
the fluid determines the buoyant force. If the body is less dense than the fluid, it will float or, in
the case of a balloon, it will rise. If the body is denser than the fluid, it will sink.

Relative density also determines the proportion of a floating body that will be submerged in a
fluid. If the body is two thirds as dense as the fluid, then two thirds of its volume will be
submerged, displacing in the process a volume of fluid whose weight is equal to the entire
weight of the body.

In the case of a submerged body, the apparent weight of the body is equal to its weight in air
less the weight of an equal volume of fluid. The fluid most often encountered in applications of
Archimedes’ principle is water, and the specific gravity of a substance is a convenient
measure of its relative density compared to water.

2-188 Total Training Support Ltd


Issue 2 ~ September 2016 ©Copyright 2016
Module 2.2 Mechanics
Module 2.2 Mechanics

Archimedes’ principle

The buoyancy force applied to the object as it is lowered into the water is
equal to the mass of the water displaced by the object

Total Training Support Ltd 2-189


© Copyright 2016 Module 2.2 Mechanics Issue 2 - September 2016
Module 2.2 Mechanics
In calculating the buoyant force on a body, however, one must also take into account the shape
and position of the body. A steel row-boat placed on end into the water will sink because the
density of steel is much greater than that of water. However, in its normal, keel-down position,
the effective volume of the boat includes all the air inside it, so that its average density is then
less than that of water, and as a result it will float.

Archimedes’ principle applied to bodies that float


A body will float in any liquid that has a weight density greater than the weight density of the
body. For example, a body of weight density 63.4 tb/ft3 would float in ocean water
(D = 64.4 lb/ft3) and sink in lake water (D = 62.4 lb/ft3).

When bodies float they can float ‘high’ or float ‘low’. The ratio of the weight density of the
floating body relative to the weight density of the liquid determines exactly how high or low a
body will float.

In order to understand Archimedes’ principle as applied to floating bodies, let us consider a


submarine and imagine that a block of wood of weight density 48.3 lb/ft3 and volume 2 ft3 is
thrust out of the hatch of a submarine into the ocean water. We know intuitively that this block of
wood will rise to the ocean surface.

The weight of the block is (48.3 lb/ft3) (2 ft3) = 96.6 lbs. As long as the block is below the water
surface (while it is rising to the top), it displaces 2 ft3 of ocean water.

We know that:

buoyancy force = weight of displaced ocean water


= (64.4 lb/ft3) (2 ft3)
buoyancy force = 128.8 lbs

We can see why the block rises. How far will the block rise? It will rise until the buoyancy force
exactly equals its weight. In our example, it will rise until the buoyancy force has been reduced
to 96.6 lbs (the weight of the block). The buoyancy force will be reduced as the block emerges
from the water. In our example, it will rise until 25% of the block’s volume is above the water
surface. It follows that 75% of 2 ft3 (= 1.5 ft3) will be below the water surface. When this occurs,
the buoyancy force on the block is (64.4 lb/ft3) (1.5 ft3) equals 96.6 lbs. Note again that the
buoyancy force equals the weight of the block while the block is floating.

In the preceding example, note that the ratio of the weight density of the block (48.3 lb/ft3) to the
weight density of the ocean water (64.4 lb/ft3) was 0.75. We recall that 75% of the floating block
was under water. This is generally true and makes a much easier procedure to determine how
low a block will float in a given liquid.

2-190 Total Training Support Ltd


Issue 2 - September 2016 © Copyright 2016
Module 2.2 Mechanics
Module 2.2 Mechanics


' v?^S|Sfe>S-% ■: WW -
Liquids Non-metallic solids
Water 9,807 62.4 Ice"' ' .......... .....~ ........... 9,040 57J
Sea water 10,100 64.4
HiaiMSBasaai^HBsaffl Earth (packed) 14,700 94
Carbon tetrachloride
SBSWRMSBSBB-i^MKaiSS
15,630 99.5 aSK®S®5
Granite
• ' 2si«SSSSi
26,500'~ 169
Petrol 6,670 42.5 liBasssias^
^SsSSBSSBiilisaiOBHS Balsa 1,270 8
Lubricating oil 8,830
’®BOSil8sSSiO'SiIliSSBlBSS
56.2 lIBIligSiiaaiSISHBMi
Maple 6,300 40
Sulphuric acidT(100%) *7^690^ 114.3 WHSSggSSBlgBigiWSWBBia 45
IHSiSSiaiSSiiSISHSiSSSH?
Solid metals

Cast iron 70,600 449

Gold 189,300 1,205


SliSlIMiilSS
Magnesium ... 17J00 ... 109
.SBgjggHBBlSSsB
Silver 103,000 656
fisafiftSS^Mgajs®
Tungsten 186,000 1,190
§®SS:; SIIHISRi
85,300^
Brass or bronze

Weight densities for common substances

Archimedes’ principle

Total Training Support Ltd 2-191


© Copyright 2016 Module 2.2 Mechanics Issue 2 - September 2016
Module 2.2 Mechanics
In dealing with bodies that float, it is important to note that boats, made of materials more dense
than water, are shaped in such a way that the total weight density is less than water.

To understand this, consider the rowing boat with contents (people, lunch, fishing gear, etc.)
shown in the figure opposite. Note that some of the boat is below the water surface. Suppose
that the row boat floats in such a way that it displaces 8 ft3 of lake water. The weight of the
displaced water is 8 ft (62.4 lb/ft3) or 499 lbs. Therefore, the buoyancy force is 499 lbs. The boat
and contents must weigh 499 lbs to float at this level. If the boat weighs 150 lbs the contents
must weigh 349 lbs.

This is realistic (father 200 lbs, son 75 lbs, lunch 25 lbs, fishing gear 49 lbs).

One final comment should be made regarding submarines. Submarines cruising at a definite
depth in ocean water have a total weight density equal to the ‘weight density of ocean water,
64.4 lb/ft3 This means that the total weight of the submarine (metal shell, air, crew, load, ballast,
etc.) divided by the total volume is 64.4 lb/ft3. The ballast used in submarines is ocean water.
These vessels can take on water or pump out water. If the submarine wants to descend, it takes
on water. If it wants to rise toward the surface it pumps out water.

Archimedes principle and buoyant force I Fluids I Physics I Khan Academy


https ://youtu. be/vzl D7ds600c

Science - Archimedes1 Principle


https://youtu.be/OrpeXFpHLmw

Example:

A block of oak (D = 45 lb/ft3) is placed in a tank of benzene (D = 54,9 lbs /ft3). The oak floats
since its weight density is less that the weight density of the benzene. What percentage of the
oak will be below the surface of the benzene?

We find the ratio of the two weight densities.


45 lbs/ft3
= 0.82
54.9 lbs/ft3

We conclude that 82% of the oak block will be below the surface of the benzene.

2-192 Total Training Support Ltd


Issue 2 - September 2016 ©Copyright 2016
Module 2.2 Mechanics
Module 2.2 Mechanics

Boats, made of materials more dense than water, are shaped in


such a way that the total weight density is less than water

Total Training Support Ltd 2-193


©Copyright 2016 Module 2.2 Mechanics Issue 2 - September 2016
Module 2.2 Mechanics

Intentionaliy Blank

*
2-194 Total Training Support Ltd
Issue 2 ~ September 2016
Module 2.2 Mechanics ©Copyright 2016
Module 2.2 Mechanics
Archimedes9 principles as applied to airships and balloons
fn all of the above materials, we have talked about Archimedes’ principle as if it applied only to
liquids. Since most of our experience with this principle is with liquids, it seemed easier to do
this at first.

However, it must now be emphasized that buoyant forces exist also with gases. The obvious
example is that of a hot air balloon or a lighter-than-air aircraft.

Example:

The bag of a balloon is a sphere of radius 25 m filled with hydrogen of weight density
0.882 N/m3. What total weight (in Newtons) of fabric, car, and contents can be lifted by this
balloon in air of weight density 12.6 N/m3?

We first calculate the volume of the spherical balloon by recalling that the volume of a sphere is
given by:

V = -JtR3 =- (3.1416) (25 m)3 = 65,450 m3


3 3

The weight of the hydrogen is found from the formula D V = w:

(0.882 N/m3) (65,450 m3) = 57,700 N

The weight of the displaced air is:

(12.6 N/m3) (65,450 m3) = 824,700 N

Since the weight of the displaced air is the buoyancy force we can say that:

buoyancy force = 824,700 N

This buoyancy force must hold up the hydrogen, fabric, car, and contents. It follows that fabric,
car, and contents weighing 767,000 N can be lifted by this balloon. Note that this number was
obtained by subtracting 57,700 N from 824,700 N.

Usually balloons are not filled with hydrogen since hydrogen is explosive. Of course, since
hydrogen is the lightest of ail gases it is the most efficient. However, the danger of explosion
outweighs this advantage. The next lightest gas is helium of weight density 1.74 N/m3. Usually,
balloons are filled with this gas.

Total Training Support Ltd 2-195


©Copyright 2016 Module 2.2 Mechanics Issue 2 - September 2016
Module 2.2 Mechanics

intentionally Blank

2-196 Total Training Support Ltd


Issue 2 - September 2016
Module 2.2 Mechanics ©Copyright 2016
Module 2.2 Mechanics
Worksheet

1. A solid aluminium object of volume 250 ft3 is resting on the ocean floor. A salvage crew
plans to raise this object. What force will be needed?

2. A solid steel body of volume 125 ft3 is to be raised by a salvaging crew to the surface of a
lake. What force will be needed?

3. What percentage of an iceberg is below the surface of the ocean?

4. A canoe is floating in such a way that it displaces 6 ft3 of lake water. If the canoe weighs
100 lbs, what is the weight of its contents?

5. A balloon is spherical in shape and has a radius of 20 ft. It is filled with helium (weight
density 0.01 lb/ft3) and is floating in air (weight density 0.08 lb/ft3). What is the weight of the
balloon (fabric, crew and contents etc.)?

Total Training Support Ltd 2-197


©Copyright 2016 Module 2.2 Mechanics issue 2 - September 2016
Module 2.2 Mechanics

Intentionally Blank

2-198 Total Training Support Ltd


Issue 2 - September 2016
Module 2.2 Mechanics ©Copyright 2016
Modub 2.2 Mechanics
Answers

1. 26,200 lbs

2. 53,100 lbs

3. 89%

4. 274 lbs

5. 2,240 lbs

Total Training Support Ltd 2-199


© Copyright 2016 Moduie 2.2 Mechanics Issue 2 - September 2016
Module 2.2 Mechanics

Intentionally Blank

2-200 Total Training Support Ltd


Issue 2 - September 2016
Module 2.2 Mechanics ©Copyright 2016
Module 2.2 Mechanics
2.2.2 Kinetics
Linear movement
In this section, we deal with uniform motion in a straight line, and motion under constant
acceleration, including motion under gravity.

When a body is moving in a straight line with constant speed it is not accelerating. We say, in
this case, that it is moving with constant velocity. If a body’s velocity is not constant, it is
accelerating. A body accelerates if it is changing its speed and/or its direction,

When we discuss a body’s straight-line motion, then we do not have any change in direction. In
this special case, any acceleration is due to a change in speed.

The equations of motion


In all of the following discussion, certain symbols will be used. These symbols are summarized
below:

« Vav = average velocity


® t = time
® u = initial velocity
• v = final velocity
« a = acceleration
« s = distance covered*

*Note: that ‘s’ is the traditional notation for distance in almost all physics textbooks. This choice
reduces confusion with the symbol ‘d’ for derivative, a concept from calculus.

Total Training Support Ltd 2-201


©Copyright 2016 Module 2.2 Mechanics Issue 2 - September 2016
Module 2.2 Mechanics
There is a formula dealing with the motion of a body that you have used for many years. In
school, you probably memorised the formula in these words:

Distance ~ Rate (or speed) x Time

Using our above symbols, we could write:

(1) S = Vavt

Note that for the rate, we have used the average speed. We all know that even though
sometimes speed changes, we can always talk about the average speed. Thus, if we travel at
an average speed of 50 mph for 6 hours, we cover 300 miles.

Now we must extend our treatment of motion to include the concept of acceleration.
Acceleration (for straight-line motion) is the rate of change of speed in time. We define
acceleration (for straight-line motion) in the following manner:

In using this formula, a may be either positive or negative. If v is less than u, then our value of a
turns out to be a negative number.

Example:

A truck is initially travelling at a speed of 50 ft/sec. The driver applies his brakes for 15 seconds.
The final speed of the car is 20 ft/sec. What is the acceleration?
20 ft/sec-50 ft/sec
15 sec

-30 ft/sec
15 sec

-2 ft/sec/sec
a 2 ft/sec/sec
sec

a = -2 ft/sec2
Note: that the unit of acceleration has the square of a time unit in its denominator.

A little thought will convince you that an acceleration is positive if the body is increasing speed
and negative when the body is decreasing its speed. If we cross-multiply in formula (2) we
obtain:

at = v -- u

After transposing, we can write:

(3) v = u + at

2-202 Total Training Support Ltd


Issue 2 - September 2016
Module 2.2 Mechanics ©Copyright 2016
Module 2.2 Mechanics
If a car is on a motorway and the driver is increasing speed smoothly and regularly, we note that
his average speed is the average of his initial and final speed.

The equation can be written:


.. _ u + v
Vav —------ —

If this value of Vav is substituted into equation (1), we have:


u+v
(4) Sa t

In this equation, we can substitute for v (= u + at) using the value in equation (3).
s- u t = 2u + at .
S~ 2 2
After a bit of algebra, we obtain:

(5) s = ut + 72 at2

Equation (4) can be written, after cross-multiplication:

2s = (u + v)t

We can now multiply this equation by equation (2). After cancelling time (t) on the right:

2as = (v - u) (v + u)
or 2as = v2 - u2

The final form of this formula is:

(6) v2 = u2 + 2as

These equations are very important. They enable us to deal with all kinds of motion problems
where the body is in straight line motion and is changing its speed. These formulas will be
summarized below. They will be numbered with Roman numerals and can be referred to by
these numbers when used in the problem exercises.

ii. v = u + at

iii. s = ut + 1/a at2

iv. v2 - u2 + 2as

When a body in straight line motion is not changing speed, or in cases where we are interested
only in the average speed, the formula is more simple.

S = Vavt

Total Training Support Ltd 2-203


©Copyright 2016 Module 2.2 Mechanics Issue 2 - September 2016
Module 2.2 Mechanics
Formulas i. through iv. are used in many practical physics problems. Note that each one
involves four quantities. When a problem is given to you to solve, be sure to determine which of
these three quantities are given to you, and which quantity is to be found. Choose the formula
which involves these four quantities. If the formula is not solved for the unknown quantity, solve
for this quantity algebraically. Finally substitute the known quantities and solve for the unknown
quantity.

An example should clarify the above procedure.

Example:

A car has an initial speed of 50 ft/sec and a final speed of 75 ft/sec. While it is undergoing this
change of speed, it travels a distance of 125 ft. What is its acceleration?

In tackling this problem, it is wise to write down exactly what is known and what is unknown.

u = 50 ft/sec V = 75 ft/sec

S = 125ft a=?

Formula iv involves these four quantities. Note that i, ii, and iii do not involve these exact four
quantities. Formula iv is the one to use. First it should be solved for the unknown, a.

V2 = u2 + 2as

V2 - u2 = 2as
_ v2-u2
a 2s

„ (75 ft/sec)2 - (50 ft/sec)2


a™ 2 (125 ft)

3125 ft2/sec
a =----------- ft2 - ~~
-----2 ~ >(2.5----- 1
250 ft sec2 ft
a - 12.5 ft/sec2

2-204 Total Training Support Ltd


Issue 2 - September 2016 © Copyright 2016
Module 2.2 Mechanics
Module 2.2 Mechanics
Motion under gravity
Common experience indicates that falling bodies accelerate or increase in speed as they fall.
Close to the surface of the earth this ‘acceleration of a freely falling body’ has been measured to
be about 32 ft/sec2 in the imperial system and 9.8 m/sec2 in the metric system. The ‘about’ in
the preceding sentence indicates that this quantity varies somewhat over the face of our earth.
The values given are average values.

When we use the words ‘freely falling’, we mean that we are neglecting the effects of air
resistance (as if we were in a vacuum). Of course, there is always air resistance, so how can we
neglect it?

When a body is falling with a great speed, air resistance can certainly not be neglected. To use
the acceleration formulas in these cases would give us results that are not valid. However, if a
body is falling close to the surface of the earth, the acceleration formulas do give us valid results
if the height from which it falls is not too great.

Some numerical data should clarify the preceding statements. If a compact body, such as a
stone, is dropped (not thrown) from a height of 324 ft above the surface of the earth, it will take
about 4.5 seconds for the body to reach the ground. It will have obtained a speed of 144 ft/sec
(98 mph). At this speed, the effects of air resistance are still quite negligible. Above this speed
(98 mph), the effects of air resistance are not negligible.

Therefore, we can conclude that the fall of a body from a height of 324 ft or less (or equivalently
during a time of 4.5 seconds or less) can be handled quite accurately with the ordinary
acceleration formulas. The value of the acceleration will be either 9.8 m/sec2 or 32 ft/sec2 if the
body is rising and therefore decreasing its speed the values of the acceleration will be -
9.8 m/sec2 or -32 ft/sec2.

If a body falls from a height greater than about 324 ft above the surface of the earth, the air
resistance becomes very important. As we have said, a height of 324 ft corresponds to a fall of
4.5 seconds. When the time of fall increases to about 8 seconds, the speed of fall has increased
to about 115 mph. When the time of fall is between 4.5 and 8 seconds the speed increases in a
non-linear manner from 98 mph to 115 mph. As the time of fall increases beyond 8 seconds the
speed of fall remains constant at about 115 mph. This speed of fall is called the ‘terminal
velocity’.

All of the above data indicates that it is possible to use the acceleration formulas with accurate
results for many applications dealing with falling bodies. We will limit our applications to cases
where the formulas are valid: heights less than 324 ft and times of fall less than 4.5 seconds.

Example:

A body started from rest and has been falling freely for 3 seconds. At what speed is it falling?

u = o, t - 3 seconds, a = 32 ft/sec2, v=?

We will use formula ii.

v = u + at
ft
v — 0 + 32- (3 sec)
secz

v = 96 ft/sec

Total Training Support Ltd 2-205


© Copyright 2016 Module 2.2 Mechanics Issue 2 - September 2016
Module 2.2 Mechanics
Example:

A body started at rest and has been falling freely for 3 seconds. How far has it fallen?

u = 0, t = 3sec, a = 32 ft/sec2, s=?

We will use formula iii.

s = ut + % at2

ft
s = (0) (3 sec) +% 32 (3 sec)2
sec2

s = 144 ft

Example:

A body is thrown upward with an initial speed of 120 ft/sec. How high does it rise?

u = 120 ft/sec, v = 0, a = -32 ft/sec2, s=?

We will use formula iv.

v2 = u2 + 2as
V2 ~~ u2
s =--------
2a
„ 0 - (120 ft/sec)2
S 2 (-32 ft/sec2)

sec2
s = 225———
sec2
s = 225 ft

The Equations of Motion


https://youtu.be/shK_V9qlLgl

Motion Physics 1: Acceleration due to Gravity


https ://youtu. be/bGAbifg FCbc

2-206 Total Training Support Ltd


Issue 2 - September 2016
Module 2.2 Mechanics © Copyright 2016
Module 2.2 Mechanics
Worksheet

1. A car on the motorway is accelerating at 25 ft/sec2. If it started from rest and has been
accelerating for 5 seconds, how far has it travelled during this time of acceleration?

2. A truck had an initial velocity of 40 ft/sec. It accelerated at 10 ft/sec2 and reached a final
velocity of 60 ft/sec. How far did this truck travel while it was accelerating?

3. A car slowed down from 80 ft/sec to 40 ft/sec while travelling a distance of 100 ft. What was
its acceleration?

4. A car, originally travelling at 25 ft/sec, increases its speed at a rate of 5 ft/sec2 for a period
of 6 sec. What was its final speed?

5. A car has an initial velocity of 40 ft/sec. It slows down at a rate of 5 ft/sec2 and covers a
distance of 60 ft while slowing down. What is its final velocity?

6. A stone is dropped from a high building and falls freely for 4 sec. How far (in metres) has it
fallen during this time?

7. A stone is thrown upward with an initial velocity of 64 ft/sec. How high does it rise?

8. A ball is dropped from a bridge into the river below and 2.5 seconds after the ball is
dropped a splash is heard in the water below. How high is the bridge?

9. A car starts with an initial velocity of 30 ft/sec and accelerates for 5 seconds at 4 ft/sec2.
How far has it travelled during this time?

10. An aeroplane has a take-off run of 900 feet, at the end of which its speed is 80 mph. How
much time does the run take? (Hint: convert mph to ft/sec first).

11. An aircraft, powered by two turbofan engines, has a maximum acceleration during take-off
of 20 ft/sec2. What velocity can it achieve by the end of a 1,000-foot take-off run?

Total Training Support Ltd 2-207


© Copyright 2016 Module 2.2 Mechanics Issue 2 - September 2016
Module 2.2 Mechanics

Intentionally Blank

2-208 Total Training Support Ltd


Issue 2 - September 2016
Module 2,2 Mechanics © Copyright 2016
Module 2.2 Mechanics
Answers

1. 312ft

2. 100 ft

3. -24 ft/sec2

4. 55 ft/sec

5. 32 ft/sec

6. 78 m

7. 64 ft

8. 100 ft or 30.6 m

9. 200 ft

10. 15 sec

11. 200 ft/sec

Total Training Support Ltd 2-209


©Copyright 2016 Module 2.2 Mechanics Issue 2 - September 2016
Module 2.2 Mechanics
Rotational movement
in this section, we deal with uniform circular motion, including centrifugal / centripetal
acceleration

Previously we discussed constant speed and accelerated motion in a straight line and derived
four important formulas which will be reviewed below. In this chapter we will consider motion
which takes place on a circular path. Such motion is very common in our complex society and
we need to understand more about motion in curved paths.

Angular distance
Before we begin our discussion, we need to define a new unit for measuring angles, the radian
(see figure opposite).

A radian is defined as the central angle subtending a length of arc equal to the radius of the
circle.

A radian is approximately equal to 57.3°. The conversion factors for angle units are;

1 revolution = 360°
1 revolution - 2k radians
2k radians = 360°
1 radian = 57.3°

Now let us consider a body (represented by a point) moving in a circular path. An initial
reference line is shown in the figure opposite. As the point moves about the circle in a
counter-clockwise sense, a line drawn between the point and the centre of the circle
continuously sweeps out an angle. This angle can be measured in revolutions, radians or
degrees. We call this angle the angular displacement of the point and use the Greek letter theta
(0) to represent this angular displacement.

Angular velocity and acceleration


If the point moves with constant speed it also has a constant angular velocity. That is, the line
drawn from the point to the centre of the circle sweeps out a definite number of revolutions,
radians, or degrees each second or minute. The symbol used to represent angular velocity is
the Greek letter omega (co).

Angular velocity can be expressed in different units, such as,


radians rev degrees
sec sec sec

radians rev degrees


min min min

It is also possible that the point is not moving with constant angular velocity. It may be
increasing or decreasing its angular velocity. When a CD starts rotating in a CD drive the
angular velocity increases until it reaches a constant value. After the reject button is pushed the
angular velocity decreases until the CD comes to rest.

In both of the above cases we say that the point has an angular acceleration. The Greek letter
alpha (a) is used for angular acceleration. Note that a is positive if the angular velocity is
increasing and negative if the angular velocity is decreasing.

2-210 Total Training Support Ltd


Issue 2 - September 2016
Module 2.2 Mechanics © Copyright 2016
Module 2.2 Mechanics

Total Training Support Ltd 2-211


©Copyright 2016 Module 2.2 Mechanics Issue 2 - September 2016
Module 2.2 Mechanics
Angular acceleration can also be expressed in different units,

radians rev degrees


sec2 sec2 sec2

radians rev degrees


min2 min2 min2

Now as a body moves in a circular path four similar equations hold as in the case of a body
moving in a straight-line path. Both sets of equations will be shown opposite. It is important to
re-memorize the equations for straight-line motion. In this way the other four equations will also
be known, since they are exactly analogous.

A (O1 + C02 .
0 = T“‘

v - u + at 0)2 = 0)1 + at
s ~ ut + 54 at2 0 = (o-tt + 54at
v2 = u2 + 2as (O22 - (012 + 2a0

Study these equations carefully and note that the set to the right, the ‘rotational analogy’ are
easily remembered if the left set is well known. We recall that the subscripts ‘u’ and ‘v’ indicate
‘initial’ and ‘final’.

These four rotational equations help us to solve many practical problems dealing with rotating
bodies.

Example:

A rotating machine part increases in angular velocity from 3 rev/min to 35 rev/min in


3.5 minutes. What is its angular acceleration?

We use the following equation and solve it for a.

0)2 = + at
0)2 - 0)1 Q
-----------=
t
We now substitute our known values.
35 rev/min - 3 rev/min A „ A , . ,
a =---------------------------- = 9.14 rev/min2
3.5min
Example:

A propeller starts from an angular velocity of 900 rev/min and accelerates at 100 rev/min2 for
5 minutes. Through how many revolutions has it turned?

9 = coit + % at2

9 = (900 rev/min) (5 min) + 54 (100 rev/min2)(5 min)2

0 = 5,750 revolutions
2-212 Total Training Support Ltd
Issue 2 - September 2016 ©Copyright 2016
Module 2,2 Mechanics
Module 2.2 Mechanics
Example:

A propeller starts at 1,000 rev/min and accelerates at 100 rev/min2 through 2,000 revolutions.
What is its final angular velocity?

o)22 = coi2 +2cxQ

tO22 = (1,000 rev/min)2 + 2(100 rev/min2) (2,000 rev)

o)2 = 1,180 rev/min

We note that there is an acceleration of the body ‘in the path’, called the tangential acceleration.
The body is increasing or decreasing its speed, or traversing the circle. We recall also that when
a body moves in a circle there is also a centripetal acceleration, V2/R, that is always directed
toward the centre of the circular path.

Total Training Support Ltd 2-213


©Copyright 2016 Module 2.2 Mechanics Issue 2 - September 2016
Module 2.2 Mechanics
Thus, when a body is increasing speed as it moves in a circular path there are two acceleration
vectors, one tangential to the path, and the other directed to the centre of the path (centripetal
acceleration). In the figure opposite, the body is increasing speed in the counter-clockwise
sense. The directions of the two acceleration vectors are shown.

Arc distance
In the figure opposite (right), ‘s’ is the length along the path. We would like to relate this distance
to the size of the central angle (0) and the radius (R) of the circular path. In our preceding
discussion, the angle (0) was measured in any of three different units, degrees, revolutions, or
radians.

The equation that relates s to 0 and R is a very simple one if we limit the angular unit to radians.
This equation is:

S = R0

We see that this equation is true if we look at the figure opposite. We note, by measuring, that
the equation is satisfied. We also see that it would not be true if the angle 0 was in revolutions
or degrees.

We now have a new problem to deal with in our treatment of rotational motion. There is a limit to
the units that may be used in this equation. We repeat that, for this equation, we must use
radian measure. Also, any equation that is derived from s0=0R0 will have this same restriction.

Suppose that a body moves a small distance along the path and sweeps out a small central
angle.

The usual mathematical notation for a very small quantity is the use of the Greek letter delta (A).

As = RA0
Tangential velocity
Let us divide both sides of this equation by the time, (At) during which the motion occurred.
As _ A0
At At
We can write:

V = Rco

2-214 Total Training Support Ltd


Issue 2 - September 2016
Module 2.2 Mechanics ©Copyright 2016
itodute 2.2 Mechanics

Tangential acceleration (at)


and centripetal acceleration (ac)

Total Training Support Ltd 2-215


©Copyright 2016 Module 2.2 Mechanics Issue 2 - September 2016
Module 2.2 Mechanics
Tangential acceleration
If this velocity in the path is changing, there is also a change in the angular velocity. Assume
that this change occurs in the small time interval (At).

We can write:

Av = RA©
Next, we divide left and right members by At.
As _ p A0
At At

The tangential acceleration (a) in the left side is the rate at which a body moving in a circular
path is picking up speed in the path. It is equal to the radius times the angular acceleration (a).

We can write:

A = Ra

Let us summarize the three important equations we have derived:

s = RG
v = R©
a = Ra

All three of these equations require the use of radian measure. This means that:

0 must be in radians
© must be in rad/min or rad/sec
a must be in rad/min2 or rad/sec2

Note that the radian is called a ‘dimensionless’ unit. We put it in or take it out for clarity.

Example:

A car is moving on a circular racetrack of radius 150 ft. It sweeps out an angle of 200°. How far
has it travelled?

We note that:

6.28
0 = 200 x — — rads = 3n 49 rads
360°
s = R0

s = (150 ft) (3.49 rad)

s = 523 ft

s = 3.36 rev/min

2-216 Total Training Support Ltd


Issue 2 ~ September 2016
Module 2.2 Mechanics © Copyright 2016
Module 2.2 Mechanics
Example:

A race car is travelling at a speed of 176 ft/sec (120 mph) around a circular racetrack of radius
500 ft. What is the angular velocity of this car in rev/min?

Use the equation:

s = Roo

v _ 176 ft/sec
or R 500 ft

co = 0.352 rad/sec

Note that we knew that the unit of our answer is rad/sec and not rev/sec since the equation we
used always is in radian measure. The units in the right side of the second equation above
actually come out as "nothing’Vsec. We put in the radian unit in the numerator for clarity.

In order to find our answer in rev/min we use the proper conversion factors.
0.352 rad/sec x 60 sec
o 3.36 rev/min
6.28 rad
Example:

A race car is moving on a circular racetrack of radius 4,000 ft. It is increasing its speed at a rate
of 15 ft/sec2. What is its angular acceleration rev/sec2?

We use the equation:

a = Ra
a- a ~ ft/sec2
~ R ~ 4,000 ft

a = 0.00375 rad/sec2

We note that the unit is rad/sec2 because the equation that we have used requires radian
measure.

To obtain a in rev/sec2, we must use the standard conversion factor.


~ 0.00375 rad/sec2
6.28 rad

a = 0.000597 rev/sec2

Tota! Training Support Ltd 2-217


© Copyright 2016 Module 2.2 Mechanics Issue 2 - September 2016
Module 2.2 Mechanics

Intentionally Blank

2-218 Total Training Support Ltd


Issue 2 - September 2016
Module 2.2 Mechanics ©Copyright 2016
Module 2.2 Mechanics SSfllF
Worksheet

1. A propeller starts from rest and accelerates at 120 rev/sec2 for 4 seconds. What is its final
angular velocity in rev/sec? In rev/min?

2. A rotating turntable starts from rest and accelerates at 5 rev/min2 for 3 minutes. Through
how many revolutions has it turned?

3. A helicopter main rotor starts from an initial angular velocity of 2 rev/min and accelerates at
60 rev/min2 while turning through 400 revolutions. What is its final angular velocity?

4. An aeroplane is circling an airport in a circular pattern of radius 15,000 ft. It sweeps out an
angle of 340°? How far has it travelled?

5. An aeroplane is circling an airport in a circle of radius 5,000 ft. How far has it travelled after
4 revolutions?

6. A race car is moving on a circular track of radius 600 ft. It is travelling at a speed of 100 ft/s.
What is its angular velocity in rev/min?

7. A race car is moving on a circular racetrack of radius 800 ft. It is accelerating at a rate of
10 ft/sec2 What is its angular acceleration in rev/sec2?

8. A helicopter tail rotor starts with an initial angular velocity of 15 rev/sec and decelerates at a
rate of 2.00 rev/sec2 until it comes to rest. Through how many revolutions has the rotor
turned while it comes to rest?

Total Training Support Ltd 2-219


© Copyright 2016 Module 2.2 Mechanics Issue 2 - September 2016
Module 2.2 Mechanics

Intentionally Blank

2-220 Total Training Support Lid


Issue 2 - September 2016
Module 2.2 Mechanics ©Copyright 2016
Module 2.2 Mechanics
Answers

1. 480 rev/sec, 28,800 rev/min

2. 22.5 rev

3. 219 rev/min

4. 89,000 ft

5. 23.8 miles

6. 1/6 rad/s, 5/n rev/min

7. 1/160ic rev/sec2

8. 56.3 rev

Total Training Support Ltd 2-221


© Copyright 2016 Module 2.2 Mechanics Issue 2 - September 2016
Module 2.2 Mechanics
Centrifugal / centripetal acceleration
A ball whirled in a circle experiences an acceleration toward the centre of the circle.

This can be proven by considering that the ball is continually changing direction as it moves in a
circle. Newton’s first law tells us that the ball would prefer to follow a straight path, and that for it
to deviate from a straight path, a force must be applied to it.

It is a direct result of Newton’s first law that a hammer thrower (figure opposite) must continually
pull towards the centre of rotation, applying his full weight to make the hammer accelerate
continually towards the centre of rotation. As soon as the athlete stops applying the force
towards the centre (i.e. releases the hammer) the hammer travels in a straight line, at a tangent
to the circle.

This acceleration is in the same direction as the force which makes it move in a circle. This
force is called centripetal force (from the Latin meaning centre-seeking)

Since we have a constant change in the direction of the motion of the hammer, we have a
constant acceleration. This is called centripetal acceleration and can be calculated by the
square of the velocity divided by the radius of the circular path, thus:
V2
Centripetal acceleration =—

Newton’s second law connects acceleration and force, by Force = Mass x Acceleration. Thus,
we can write the equation:
_ _ mV2
rnet - mac Or rnet “
R
Units of force
The units which we will use in our discussion of Newton’s laws are the same as the units used
in the formula relating weight to mass (w = mg). These units are reviewed and summarized in
the table opposite.

Each set of units, (pound, slug, ft/sec2) in the imperial system, or (newton, kilogram, m/sec2) in
the metric system is said to be consistent in the following sense: a force of 1 lb when applied
to a mass of 1 slug gives it an acceleration of 1 ft/sec2.

Circular Motion I A-Level Physics I Doodle Science


https://youtu.be/CzcmUiD39VI

2-222 Total Training Support Ltd


Issue 2 - September 2016 ©Copyright 2016
Module 2,2 Mechanics
Modute 2.2 Mechanics

Centripetal force exerted by a hammer thrower

Force pound (lb) newton (N)

Acceleration ft/sec2 m/sec2


Units of force, mass and acceleration

Total Training Support Ltd 2-223


© Copyright 2016 Module 2.2 Mechanics issue 2 - September 2016
Module 2.2 Mechanics
Similarly, a force of 1 newton applied to a mass of 1 kilogram causes it to accelerate at
1 m/sec2.

Using Newton’s second law, we can write:

1 newton = 1 kilogram m/sec2

and

1 pound = 1 slug ft/sec2

We note that Newton’s second law is correctly written as:

Fnet = ma

However, we often assume that the force acting on mass (m) is the net force. Thus, we usually
write the second law simply as:

F = ma

or, for circular motion,

F = —~ (Centripetal force)
R
Newton’s second law when applied to bodies moving in a circular path states that the force
directed toward the centre of the path must equal the mass of the body times the square of the
speed of the body divided by the radius of the path. This force is called the centripetal (centre­
seeking) force.

Example:

Find the acceleration of a 3 slug object acted upon by a net force of 1.5 lbs
F
a =—
m

1.5 lbs
a =---------
3 slugs

a = 0.5 ft/sec2

Example:

A mass of 6 kilograms accelerates at 5 m/sec2. Find the force which is acting on this object.

F= ma

F=(6 kg) (5 m/sec2) = 30 N

2-224 Total Training Support Ltd


Issue 2 - September 2016
Module 2,2 Mechanics © Copyright 2016
Module 2.2 Mechanics
Problems

1. What centripetal force is needed to keep a 3 slug ball moving in a circular path of radius
2 feet and speed 4 ft/sec?

2. A boy is swinging a stone at the end of a string. The stone is moving in a circular path. The
speed of the stone is 5 ft/sec and the radius of the path is 1.5 ft. What is the centripetal
acceleration of the stone?

Totai Training Support Ltd 2-225


© Copyright 2016 Moduie 2.2 Mechanics Issue 2 - September 2016
Module 2.2 Mechanics

Intentionally Blank

2-226 Total Training Support Ltd


Issue 2 - September 2016
Module 2.2 Mechanics ©Copyright 2016
Module 2.2 Mechanics
Answers

1. 24 lb

2. 16.67 ft/sec2

Total Training Support Ltd 2-227


© Copyright 2016 Module 2.2 Mechanics Issue 2 - September 2016
Module 2.2 Mechanics
Periodic motion
In this section we deal with pendula movement. This includes simple pendulums and the
motion of spring/mass systems.

Pendula movement
A simple pendulum is one which can be considered to be a point mass suspended from a string
or rod of negligible mass. It is a resonant system with a single resonant frequency. For small
amplitudes, the period of such a pendulum can be approximated by:

Where: L =the length of the pendulum is m, or ft

g = the magnitude of acceleration due to gravity

= 9.81 m/s2 or 32 ft/s2

Note: the natural frequency of oscillation is independent of the mass of the pendulum, and of
the amount of initial displacement.

This expression for period is reasonably accurate for angles of a few degrees, but the treatment
of the large amplitude pendulum is much more complex. It is interesting to note that the
pendulum will oscillate at only one frequency, regardless of how far the pendulum is initially
displaced, or for how long the pendulum is left to oscillate. The only factor that changes, is the
linear velocity of the mass. This fixed frequency is known as the natural frequency of
oscillation.

If we consider only the horizontal motion of the mass and neglect its vertical motion as it swings
(an assumption which can be made if the string is long compared to the amplitude of swing),
then the periodic motion is said to be simple harmonic motion (shm).

Time period (T) and frequency (f) can also be related to each other by the formulae:

2-228 Total Training Support Ltd


Issue 2 - September 2016 ©Copyright 2016
Module 2.2 Mechanics
Module 2.2 Mechanics

A simple pendulum

Total Training Support Ltd 2-229


© Copyright 2016 Module 2.2 Mechanics Issue 2 - September 2016
Module 2.2 Mechanics
Mass and spring
When a mass is acted upon by an elastic force which tends to bring it back to its equilibrium
position, and when that force is proportional to the distance from equilibrium (e.g., doubles
when the distance from equilibrium doubles - a Hooke’s law force), then the object will undergo
periodic motion when released.

A mass on a spring is the standard example of such periodic motion. If the displacement of the
mass is plotted as a function of time, it will trace out a pure sine wave. The motion of the
medium in a travelling wave is also simple harmonic motion as the wave passes a given point in
the medium.

It is interesting to note that the spring/mass system will oscillate at only one frequency,
regardless of how far the mass is initially displaced, or for how long the system is left to
oscillate. The only factor that changes, is the linear velocity of the mass. The fixed frequency is
known as the natural frequency of oscillation, and can be calculated from the formula:

Where: L = the length of the pendulum is m, or ft


g = the magnitude of acceleration due to gravity

Note: The natural frequency of oscillation is independent of the magnitude of gravity, and of the
amount of initial displacement.

2-230 Total Training Support Ltd


Issue 2 - September 2016 ©Copyright 2016
Module 2.2 Mechanics
Module 2.2 Mechanics

Sinusoidal motion of a spring/mass system

Total Training Support Ltd 2-231


©Copyright 2016 Module 2.2 Mechanics Issue 2 - September 2016
Module 2.2 Mechanics
Simple harmonic motion (SHM)
Motion which repeats itself precisely and can be described with the following terms:

• Period: the time required to complete a full cycle, T in seconds.


• Frequency: the number of cycles per second, f in Hertz (Hz),
• Amplitude: the maximum displacement from equilibrium, A

and if the periodic motion is in the form of a travelling wave, one needs also:

• Velocity of propagation: V,
• Wavelength: repeat distance of wave, X

Simple harmonic motion is the motion of a simple harmonic oscillator (such as a pendulum or
spring/mass system), a motion that is neither driven nor damped. The motion is periodic, as it
repeats itself at standard intervals in a specific manner - described as being sinusoidal, with
constant amplitude. It is characterized by its amplitude, its period which is the time for a single
oscillation, its frequency which is the number of cycles per second, and its phase, which
determines the starting point on the sine wave. The period, and its inverse the frequency, are
constants determined by the overall system, while the amplitude and phase are determined by
the initial conditions (position and velocity) of that system.

A single frequency travelling wave will take the form of a sine wave. A snapshot of the wave in
space at an instant of time can be used to show the relationship of the wave properties
frequency, wavelength and propagation velocity.

The motion relationship ‘distance = velocity x time’ is the key to the basic wave relationship.

With the wavelength as distance, this relationship becomes X= vT. Then using f = 1/T gives the
standard wave relationship

V = fX,

This is a general wave relationship which applies to sound and light waves, other
electromagnetic waves, and waves in mechanical media.

Properties of SHM
Considering the motion of a mass on the end of a spring, or the horizontal motion of a
pendulum, the following properties can be observed:

• The velocity of the body is always changing. If is maximum at the undisturbed position
(centre of its motion) and zero at the extremities of its motion (maximum displacement
position)
• The acceleration of the body is always changing. It is maximum at the extremities of its
motion (maximum displacement position) and zero at its undisturbed position (centre of
motion).

In other words, when its velocity is zero, its acceleration is a maximum, and when its
acceleration is zero, its velocity is a maximum.

Physics Periodic Motion


https://youtu.be/D6oVrp2QTKM

2-232 Totai Training Support Ltd


Issue 2 - September 2016 ©Copyright 2016
Module 2.2 Mechanics
Moduie 2.2 Mechanics

Velocity of
propagation
Amplitude

The sinusoidal waveform terminology

Sinusoidal motion of a spring/mass system

Total Training Support Ltd 2-233


© Copyright 2016 Module 2.2 Mechanics Issue 2 - September 2016
■erULa. ESrl fti®® X ■ ’
teiffifiBw
Module 2.2 Mechanics
Simple theory of vibration

Vibration
Vibration refers to mechanical oscillations about an equilibrium point. The oscillations may be
periodic such as the motion of a pendulum or random such as the movement of a tire on a
gravel road.

Vibration is occasionally desirable. For example, the motion of a tuning fork, the reed in a
woodwind instrument or harmonica, or the cone of a loudspeaker is desirable vibration,
necessary for the correct functioning of the various devices.

More often, vibration is undesirable, wasting energy and creating unwanted sound - noise. For
example, the vibrational motions of engines, electric motors, or any mechanical device in
operation are typically unwanted. Such vibrations can be caused by imbalances in the rotating
parts, uneven friction, the meshing of gear teeth, etc. Careful designs usually minimise
unwanted vibrations.

The study of sound and vibration are closely related. Sound, pressure waves, are generated by
vibrating structures (e.g. vocal cords) and pressure waves can generate vibration of structures
(e.g. ear drum). Hence, when trying to reduce noise it is often a problem in trying to reduce
vibration.

Types of vibration

Free vibration occurs when a mechanical system is set off with an initial input and then allowed
to vibrate freely. Examples of this type of vibration are pulling a child back on a swing and then
letting go or hitting a tuning fork and letting it ring. The mechanical system will then vibrate at
one or more of its natural frequencies and damp down to zero.

Forced vibration is when an alternating force or motion is applied to a mechanical system.


Examples of this type of vibration include a shaking washing machine due to an imbalance,
transportation vibration (caused by truck engine, springs, road, etc.), or the vibration of a
building during an earthquake. In forced vibration the frequency of the vibration is the frequency
of the force or motion applied, but the magnitude of the vibration is strongly dependent on the
mechanical system itself.

2-234 Total Training Support Ltd


Issue 2 - September 2016 ©Copyright 2016
Module 2.2 Mechanics
Module 2.2 Mechanics

Total Training Support Ltd 2-235


© Copyright 2016 Module 2.2 Mechanics Issue 2 - September 2016
Module 2.2 Mechanics
Resonance

What is resonance?
Resonance is the phenomenon of producing large amplitude of vibrations by a small periodic
driving force. It is the tendency of a system to oscillate at maximum amplitude at a certain
frequency. This frequency is known as the system’s resonance frequency (or resonant
frequency). When damping is small, the resonance frequency is approximately equal to the
natural frequency of the system, which is the frequency of free vibrations. Under resonance
condition the energy supplied by the driving force is sufficient enough to overcome friction.

Examples of resonance
One familiar example is a playground swing, which is a crude pendulum. When pushing
someone in a swing, pushes that are timed with the correct interval between them (the resonant
frequency), will make the swing go higher and higher (maximum amplitude), while attempting to
push the swing at a faster or slower rate will result in much smaller arcs.

Other examples:

• acoustic resonances of musical instruments


• the oscillations of the balance wheel in a mechanical watch
• electrical resonance of tuned circuits in radios that allow individual stations to be picked
up
• the shattering of crystal glasses when exposed to a strong enough sound that causes the
glass to resonate.

A resonator, whether mechanical, acoustic, or electrical, will probably have more than one
resonance frequency (especially harmonics of the strongest resonance). It will be easy to
vibrate at those frequencies, and more difficult to vibrate at other frequencies. It will 'pick out’ its
resonance frequency from a complex excitation, such as an impulse or a wideband noise
excitation. In effect, it is filtering out all frequencies other than its resonance.

2-236 Total Training Support Ltd


Issue 2 - September 2016 © Copyright 2016
Module 2.2 Mechanics
Module 2.2 Mechanics

Pushing a child on a swing is an example of


resonance. The push impulses are timed to be in
phase with the natural frequency of the swing

Total Training Support Ltd 2-237


©Copyright 2016 Module 2.2 Mechanics Issue 2 - September 2016
Module 2.2 Mechanics
What causes resonance?
Resonance is simple to understand if you view the spring and mass as energy storage elements
- the mass storing kinetic energy and the spring storing potential energy. When the mass and
spring have no force acting on them they transfer energy back forth at a rate equal to the
natural frequency. In other words, if energy is to be efficiently pumped into the mass and spring
the energy source needs to feed the energy in at a rate equal to the natural frequency. Applying
a force to the mass and spring is similar to pushing a child on swing - you need to push at the
correct moment if you want the swing to get higher and higher. As in the case of the swing, the
force applied does not necessarily have to be high to get large motions. The pushes just need
to keep adding energy into the system.

A damper, instead of storing energy dissipates energy. Since the damping force is proportional
to the velocity, the more the motion the more the damper dissipates the energy. Therefore, a
point will come when the energy dissipated by the damper will equal the energy being fed in by
the force. At this point, the system has reached its maximum amplitude and will continue to
vibrate at this amplitude as long as the force applied stays the same. If no damping exists, there
is nothing to dissipate the energy and therefore theoretically the motion will continue to grow to
infinity.

Such catastrophic resonance can be witnessed frequently, in, for example, the failure of
complete aircraft wing structures during control surface ‘flutter’, failure of helicopter structural
components, and even the collapse of road bridges in gale force winds, as experienced at
Tacoma Bridge on November 7th, 1940.

Resonance
https://youtu.be/XwTwTc9yQY8
Resonance in a Pendulum
https ://youtu. be/BJyx7 a RkH PU
Tacoma Bridge
https://youtu.be/3mclp9QmCGs

2-238 Total Training Support Ltd


Issue 2 - September 2016 © Copyright 2016
Module 2.2 Mechanics
Module 2.2 Mechanics

Resonance begins as a relatively small, periodic stimulus of a


mechanical system, such as wind buffeting a bridge. These
vibrations, however, are more or less in harmony with the bridge’s
natural vibrations. If unchecked, the vibration can increase
drastically, sending destructive, resonant vibrations traveling
through a bridge in the form of torsional waves.

The most noteworthy example of resonance occurred in 1940, when


resonant vibrations destroyed the Tacoma Narrows Bridge in
Washington. The incident was especially shocking at the time as the
structure was designed to withstand winds of up to 120 miles per
hour (193 km) and collapsed in a mere 40 mile per hour (64 km)
wind.

Total Training Support Ltd 2-239


© Copyright 2016 Module 2,2 Mechanics Issue 2 - September 2016
Module 2.2 Mechanics
Design implications of resonance
Designers of aircraft must be seriously concerned about the phenomenon of resonant frequency
because if a certain component of an aeroplane or helicopter is caused to vibrate at its resonant
frequency the amplitude of the vibration can become very large and the component will destroy
itself by vibration.

Let us examine the case of a helicopter which has a tail boom with a natural or resonant
frequency of 1 Hz. That is, if you were to strike the boom with your fist it would oscillate once
each second. The normal rotational speed of the rotor is 400 RPM and the helicopter has
3 blades on its main rotor. Each time a rotor blade moves over the tail boom the blade is going
to cause a downward pulse of air to strike the tail boom. The designer must determine the
speed at which the pulses will be equal to the resonant frequency of the boom. One cycle per
second is equivalent to 60 cycles/minute. Since each of the three blades causes a pulse each
revolution, there will be 3 x 60 or 180 pulses/minute.

Therefore, a rotor speed of 180 RPM would be critical and the pilot would be warned against
operating at that speed. Since the boom also has a secondary, or overtone, resonant frequency
of twice the fundamental resonant frequency, 360 RPM would also have to be avoided but
would not be as critical as 180 RPM. The third frequency of concern would be 3 x 180 or 540,
but that is above the rotor operating speed, so is not a problem.

The natural frequency of vibration is also an extremely important consideration in designing the
wings, horizontal and vertical stabilizers of an aircraft. The designer must be certain that the
resonant frequency when the surface is bent is different from that resonant frequency when it is
twisted. If that is not the case, an aerodynamic interaction with the elasticity of the surface can
result in ‘flutter’ which can cause the surface to fracture in a fraction of a second after it begins.

Harmonics
The harmonic of an oscillation is a component frequency of the oscillation that is a multiple of its
natural frequency (known as the fundamental frequency). For example, if the fundamental
frequency is f, the harmonics have frequency 2f, 3f, 4f, etc. The harmonics have the property
that they are all periodic at the input frequency.

Thus, if an oscillating body (e.g. a spring/mass system) can be oscillated by an excitation input
of frequency equal to its natural frequency (the ‘fundamental frequency’), it will also be
oscillated at frequencies that are harmonics of that natural frequency.

2-240 Total Training Support Ltd


Issue 2 - September 2016 © Copyright 2016
Module 2.2 Mechanics
Moduie 2.2 Mechanics

Total Training Support Ltd 2-241


© Copyright 2016 Module 2.2 Mechanics Issue 2 - September 2016
Module 2.2 Mechanics

Intentionally Blank

2-242 Total Training Support Ltd


Issue 2 - September 2016
Module 2.2 Mechanics ©Copyright 2016
Module 2.2 Mechanics
Worksheet

1. A pendulum has a length of 07 m. What Is its frequency of oscillation, and how long will it
take io oscillate 10 times?

2. A pendulum has a mass of 0.05 slugs. It takes 15 seconds to oscillate 10 times. What is its
length?

3. A mass of 0.4 kg oscillates freely on the end of a spring. The stiffness of the spring is
2 N/m. What is its natural frequency of oscillation and its time period?

4. A ball on the end of a spring bounces such that it nearly hits the floor 30 times in a minute.
The spring has a stiffness of 0.5 Ib/in. What is the value of the mass of the ball?

Total Training Support Ltd 2-243


© Copyright 2016 Module 2.2 Mechanics Issue 2 - September 2016
Module 2.2 Mechanics

Intentionally Blank

2-244 Total Training Support Ltd


Issue 2 - September 2016
Module 2.2 Mechanics ©Copyright 2016
Siodule 2.2 Mechanics
Answers

1. 0.6 Hz, 16.8 seconds

2. 0.625 m

3. 0.36 Hz, 2.8 seconds

4. 0.05 slugs

Total Training Support Ltd 2-245


©Copyright 2016 Module 2.2 Mechanics Issue 2 - September 2016
Module 2.2 Mechanics

Intentionally Blank

2-246 Total Training Support Ltd


Issue 2 - September 2016
Module 2.2 Mechanics © Copyright 2016
Module 2.2 Mechanics
Velocity ratio, mechanical advantage and efficiency

Simple machines and the principle of work


The definition of work is as follows:

W = FD cos 0

The symbol for ‘distance’ has been switched from S to D, to emphasize that we are dealing with
distances in our treatment of simple machines.

The angle (0) in this definition is the angle between the direction of the force vector and the
direction of the displacement vector.

In this section, we will assume that in all the cases we will study the force and displacement
vectors act in the same direction. This implies that the angle (0) is a 0° angle and since the
cosine of a 0° angle equals one, the equation for work becomes the simple equation:

W = FD

In this chapter, we will study six simple machines:

. The lever
® The pulley
• The wheel and axle
® The inclined aeroplane
® The screw
• The hydraulic press

General theory of all machines


In discussing machines, we will assume that there is an object on which work is to be done. We
will call this object the load. In most cases, it is required that the load be raised a certain
distance in a gravitational field. For example, we wish to put cement blocks originally on the
ground into the bed of a truck.

A machine is a device for doing this work. The input work is, by definition, the work done by the
worker, that is, the force applied by the worker multiplied by the distance through which the
worker’s force acts. The output work is, by definition, the force that actually acts on the load
multiplied by the distance the load is raised.

We note that one way to do work is to do it directly. For example, it is possible for the worker to
raise each cement block directly to the truck bed. This is possible but can be difficult if each
block weighs, say, 175 lbs. In this case it would be better to use a machine since a machine
usually decreases the force supplied by the worker and increases the distance through which
his force acts.

In the equations which follow, the subscript ‘o’ will stand for output and the subscript T will
indicate input. We will use the following defining equations:
Wo = FoDo

Wi = FiDi

Total Training Support Ltd 2-247


© Copyright 2016 Module 2.2 Mechanics Issue 2 - September 2016
Module 2.2 Mechanics
It is important to realize that there is no perfect machine. In our real world, on our earth, there is
always some friction. We always have, at least, air resistance. In addition, there is friction due to
the nooks and crannies that we would see if we inspected the surfaces of our machine parts
with a high-powered microscope.

Because of the constant presence of friction, the input work is always greater than the output
work. Some of the input work is not useful work but serves to produce sound energy (a squeak),
light energy (a spark), or heat energy.

We will use the symbol W to represent work lost because of friction.

Wi = Wo + Wf

We define two kinds of ‘mechanical advantage’. The actual mechanical advantage (AMA) is
the ratio of the output force to the input force. This actual mechanical advantage tells us how
much easier it is for the worker. The ideal mechanical advantage (IMA) is the mechanical
advantage that would exist if there were no friction in the machine. It is the ratio of input
distance to the output distance.

AMA = ^ IMA = —
Fi Do
The ideal mechanical advantage of a machine can always be determined by measurements
made on the machine itself.

Efficiency
The efficiency (Eff) of a machine is the ratio of the output work to the input work.
.. Wo

The efficiency can be expressed as a decimal or as a percentage. For example, if the efficiency
is calculated as 0.78, we can express it as 78%.

One final point should be made regarding efficiency. There is no machine that is 100% efficient.
We always have some friction. However, sometimes we assume that there is no friction and that
the machine is perfect or ideal! If a problem says that the efficiency is 100%, we are doing a
make-believe problem. This kind of a problem is not meaningless, however, because it tells us
the best that this machine can do. In this ideal case the AMA equals the IMA.

Introduction to mechanical advantage I Work and energy I Physics I Khan Academy


https://youtu.be/pfzJ-z5lj48

Mechanical advantage (part 2) I Work and energy | Physics I Khan Academy


https://youtu.be/DiBXxWBrV24

2-248 Total Training Support Ltd


Issue 2 - September 2016 © Copyright 2016
Module 2.2 Mechanics
Modtib 2.2 Mechanics
Examples:
A worker is able to raise a body weighing 300 lbs by applying a force of 75 lbs. What is the AMA
of the machine that he is using?
Fo „ 300 lbs
AMA~ =4
Fi 75 lbs

A worker applied his force through a distance of 15 ft. The load is raised a distance of 2.5 ft.
What is the Ima of the machine that he used?
Di. ------
15ft = 6*
IMA =
Do 2.5 ft
The actual mechanical advantage of a machine is 8 and the efficiency of this machine is 78%.
What is the ideal mechanical advantage?
AMA 8
IMA 10.3
Eff 0.78
A worker uses a machine to raise a load of 500 lbs a distance of 2 ft. He does this by applying a
force of 100 lbs through a distance of 12 ft. What was the efficiency of the machine?
.... 500 lbs c
Method 1: AMA = ~---- — = 5
100 bs

IMA = ^i = 6
2ft

Effx^M^=i5.=:83%
IMA 6

Method 2: Wo = (500 lbs) (2 ft) = 1,000 ft.lbs

Wi = (100 lbs) (12 ft) = 1,200 ft.lbs

Eff = — ~ I’000 x 0,83 = 83%


Wi 1,200 ft.lbs
Velocity ratio
Velocity ratio is also defined as the ratio of a distance through which any part of a machine
moves, to that which the driving part moves during the same time.

Note that it is the ratio of the output to the input ‘distances’ (not velocities). This is because
velocity is the distance divided by time, and since the time period in which the machine’s input
moves is the same as the time period in which the machine’s output moves, when the velocities
are divided to calculate the ratio, the time will cancel, leaving simply a ratio of the output to the
input distances.

Iota! Training Support Ltd 2-249


© Copyright 2016 Module 2.2 Mechanics Issue 2 - September 2016
Module 2.2 Mechanics
Simple machines
We will next consider six simple machines. In each of these cases the IMA is expressed, not as
the ratio Di/D0, but in some other manner. We will study the geometry of each of these simple
machines to determine how to express the IMA in some simple equation.

The lever
Consider the diagram in the figure opposite. Note that the lever always pivots about some point
called the fulcrum. The input force (Fi) is downward force and in our diagram, is applied at the
right end of the lever. This input force gives rise to an upward force at the left end in our
diagram. This upward force causes the load to be raised and is called To’.

In the figure opposite, note that the input force acts through a distance (DQ and the load is
raised a distance (Do).

The distance from the input end of the lever to the fulcrum is called the input lever arm (Li) and
the distance from the output end to the fulcrum is called the output lever arm (Lo).

Recall that:

However, the figure opposite shows that the ratios of lever arms and distances are equal:

= L_
Do Lo

Since it is much easier to measure lever arms than the distances of rotation, we always use the
ratio on the right hand side of the above equation to express the IMA of a lever.

There are three classes of levers:

(Lever)

• 1st Class: The fulcrum is between the load and the applied force. Examples are the claw
hammer, scissors, and crowbar.
• 2nd Class: The load is between the fulcrum and the applied force. Examples are the
nutcracker and wheelbarrow.
• 3rd Class: The applied force is between the load and the fulcrum. An example is ice
tongs.

In a third class lever, the IMA is less than one. There is no force advantage. However, there is a
speed advantage. The work can be done in less time.

2-250 Total Training Support Ltd


Issue 2 - September 2016
Module 2.2 Mechanics © Copyright 2016
I

SioduSe 2.2 Mechanics


© Copyright 2016
Total Training Support Ltd

Claw hammer Wheel barrow Human arm

E A L
Module 2.2 Mechanics
z -z b l

Piters Nut-cracker Sugar tongs


Distances moved in a simple lever system
Examples lever system

Load Effort
Issue 2
- September 2016

Pivot

First class lever Second class lever Third class lever

Classes of lever system


Module 2.2 Mechanics
The pulley
Some pulleys are firmly attached to an overhead support while other pulleys move up or down
with the load. We will refer to pulleys as ‘fixed’ or ‘movable’.

In figure (A), we have shown a single fixed pulley, if a length of pulley cord (DQ is pulled down
by a worker, the load will be raised a distance (Do). We see from the diagram that these
distances equal each other. Therefore, we conclude that the IMA of this type of pulley is one.
For example, it would take 100 lbs of force to raise a 100 lbs load. The advantage of using this
type of pulley is that the worker is able to pull down on the pulley cord and in this way an
upward force is applied to the load. We say that a single fixed pulley is a ‘direction changer’.

In figure (B), there is a single movable pulley. A study of the diagram shows that Di is always
twice Do. For example, if the load is to be raised 2 ft. the worker must pull in 4 ft. of cord.

Note also that there are 2 strands supporting the load. The IMA of a single movable pulley is 2.

In figure (C), there is a single movable pulley and a single fixed pulley. The fixed pulley again
serves to change the direction of the input force. The IMA is still 2. Note also that there are
again 2 strands supporting the load.

We conclude that the IMA of a pulley equals the number of strands supporting the load.

IMA = number of strands supporting the load (pulley)

Several other examples of various types of pulley blocks are shown in the figures D, E and F.

2-252 Total Training Support Ltd


issue 2 ~ September 2016
Module 2.2 Mechanics © Copyright 2016
Module 2.2 Mechanics

(B)

Simple pulley systems

Pulley systems are frequently used in mountaineering and rescue etc.

Total Training Support Ltd 2-253


©Copyright 2016 Module 2.2 Mechanics issue 2 - September 2016
Module 2.2 Mechanics
The wheel and axle (windlass)
Note that one cord is wrapped around the axle of radius (r). The load is attached to this cord.
Another cord is wrapped around the wheel of radius (R). The worker applies his force to this
second cord.

Both wheel and axle turn together. This means that if the wheel rotates through one revolution
the axle also turns through one revolution.

Let us suppose that the worker pulls in a length of cord equal to one circumference of the wheel
(Di) The load will be raised a distance equal one circumference of the axle, (Do),

Di _ 2kR ~ R
IMA =
Do 2%r r

(Wheel and axle) IMA = ™


r
Example:

The radius of the wheel in a windlass (wheel and axle) is 3.5 ft and the radius of the axle is
0.27 ft. The efficiency of the machine is 60%. What load can be lifted by this machine by using a
force of 75 lbs?
3,5 ft
IMA = = 13.0
0.27 ft

AMA = (Eff) (Ima)

AMA =(0.60) (13.0) = 7.8

F0=(Ama) (Fi)

FO=(7.8) (75 lbs) = 585 lbs

2-254 Total Training Support Ltd


Issue 2 - September 2016 ©Copyright 2016
Module 2.2 Mechanics
Module 2.2 Mechanics

Wheel and axle

Total Training Support Ltd 2-255


©Copyright 2016 Module 2.2 Mechanics Issue 2 - September 2016
Module 2.2 Mechanics
The inclined plane
In the inclined plane shown in the figure opposite we note that the worker slides the load up the
incline. The input distance (Di) is therefore equal to the length of the incline (L). The effect of this
is that the load is raised a distance (h). This means that the output distance (Do) equals h also.

We note that the sine of the angle of inclination (0) is also h/L. Therefore, we can write the
expression for the IMA as follows:
1
(inclined plane) IMA - ——

Example:

An inclined plane has a 32° angle of incline. A force of 50 lbs is required to slide a 90 lbs load
up the incline. What is the efficiency of this machine?
1
IMA = 1.89
sin 32°

90 lbs
AMA =
50 lbs

AMA„ 1,8
0.95 = 95%
IMA 1.98

2-256 Total Training Support Ltd


Issue 2 - September 2016 ©Copyright 2016
Module 2.2 Mechanics
itodute 2.2 Mechanics

Total Training Support Ltd 2-257


©Copyright 2016 Module 2.2 Mechanics Issue 2 - September 2016
Module 2.2 Mechanics
The screwjack
The screwjack is a development of the inclined plane.

The pitch of the screw (p) is the distance between adjacent threads (figure opposite). As the
handle is turned through one revolution, a distance given by 2rcr, the load is raised a distance of
one pitch.

Therefore, we have the relation:

(Screw jack) IMA =


P
A screwjack has a great deal of friction. Therefore, its efficiency is usually very low. However,
the distance through which the input force acts in comparison to the pitch is usually very large.
This gives a screwjack a large mechanical advantage.

2-258 Total Training Support Ltd


Issue 2 - September 2016 ©Copyright 2016
Module 2.2 Mechanics
itodufe 2,2 Mechanics

The common fly press


and vice in a
metalwork shop both
work on the principle
of the screwjack

Total Training Support Ltd 2-259


©Copyright 2016 Module 2.2 Mechanics Issue 2 - September 2016
Module 2.2 Mechanics
The hydraulic press
A cross section of a hydraulic press is shown in the figure opposite. The small rectangles are
cross sections of the circular input and output pistons. Usually, we talk about the areas of the
input and output pistons (Ai and Ao). We note that the smaller of the two pistons is the input
piston (radius = r) and, of course, the larger piston is the output piston (radius = R).

A hydraulic press is filled with some fluid (gas or liquid). This fluid exerts a common fluid
pressure throughout the device.

As the smaller piston moves downward a distance (di) the larger piston moves upward a
distance (do). We recall that the volume of a cylindrical shape is equal to the area of the circular
base x the height Also, a volume of fluid is transferred from the input (left) cylinder to the output
(right) cylinder. The volume of fluid is constant since the pressure is constant. Therefore, we can
write the equation:

irr2di = TrR2do

We can cancel the common factor (it) and rearrange the equation. We obtain:

do " r2

The left member of this equation is, by definition, the IMA, Therefore, the IMA is also equal to
the right member of this equation. Thus, we can finally say that:

(Hydraulic Press) IMA » —

We have obtained equations for the IMA of each of the six simple machines. We will do an
example of a typical problem dealing with machines. Note that any one of the six could be
chosen as an example. In the problems that follow the example, be sure to use the correct
formula for the IMA.

Simple Machines
https://youtu.be/loBkujpoowO

2-260 Totai Training Support Ltd


issue 2 - September 2016 © Copyright 2016
Module 2.2 Mechanics
Module 2.2 Mechanics

The
hydraulic
'^j' L?- L-'-"., ' ;.
press
~~S?W';
:-■;■■.■_r:'■' ■' ’. ' ■■': :■ '■■=.:_■<■■. '■■■■/.I -.; 't :-:■'■■-J:
. . lllll
■ ■ •flit
■■

the principle of the hydraulic


press

Total Training Support Ltd 2-261


©Copyright 2016 Moduie 2.2 Mechanics Issue 2 ~ September 2016
Module 2.2 Mechanics

Intentionally Blank

2-262 Total Training Support Ltd


Issue 2 - September 2016
Module 2.2 Mechanics ©Copyright 2016
Module 2.2 Mechanics
Worksheet

1. It takes a force of 80 lbs to raise a body that weighs 240 lbs. What is the actual mechanical
advantage of the machine that was used?

2. K load is raised a distance of 6 ft by a force acting through a distance of 18 ft. What is the
ideal mechanical advantage of the machine that was used?

3. What is the efficiency of a machine having an IMA of 7 and an AMA of 5?

4. A load weighing 120 lbs is raised a distance of 4 ft by a machine. The worker using the
machine exerts a force of 50 lbs through a distance of 12 ft. What was the efficiency of the
machine?

5. The radius of the wheel of a windlass is 4.0 ft and the radius of the axle is 0.2 ft. The
machine is 75% efficient. What force must be exerted to raise a load of 500 lbs with this
machine?

6. The large piston of a hydraulic press has area 1.5 ft2 and the small piston has area 0.30 ft2.
Assume that the machine is 100% efficient. What load can be raised by a force of 75 lbs?

7. A pulley system has four strands supporting the load. A force of 55 lbs is needed to raise a
load of 200 lbs. What is the efficiency of this pulley system?

8. A light aircraft has a hydraulic braking system. Each rudder pedal is connected to a master
cylinder which provides braking for one of the main landing gear wheels. Each master
cylinder has a radius of % inch. The cylinder on the wheel has a radius of 1.0 inch. If the
system is 95% efficient and the pilot exerts a force of 55 lbs on the pedal, how much force
is exerted on the brake disc by the wheel cylinder?

Total Training Support Ltd 2-263


©Copyright 2016 Module 2.2 Mechanics issue 2 - September 2016
Module 2.2 Mechanics

Intentionally Blank

2-264 Total Training Support Ltd


Issue 2 - September 2016
Module 2.2 Mechanics ©Copyright 2016
Module 2.2 Mechanics
Answers

1. 3

2. 3

3. 71%

4. 80%

5. 33.3 lbs

6. 375 lbs

7. 90.9%

8. 836 lbs

Total Training Support Ltd 2-265


© Copyright 2016 Module 2.2 Mechanics Issue 2 - September 2016
Module 2.2 Mechanics
Gear trains and gear ratios

Simple gear train


The illustration shows two meshed spur gears of different sizes. This is a simple gear train, with
one wheel on each shaft. The smaller 9 toothed pinion will have to turn two revolutions for each
revolution of the larger 18 toothed wheel and so when the wheel is used as the driver (input)
gear output motion will be faster than the input. In this case:

Velocity Ratio =
Angular movement of wheel (effort) ~ 1

or
VR =—= number driven teeth
18 number of driver teeth
The velocity ratio for a gear system is usually referred to as the gear ratio and the above value
would be quoted as a gear ration of 1:2. It equals the ratio of the speeds of the driving and
driven gears.

Sometimes the gear ratio is defined as the ratio of the driven gear speed to that of the driver. To
avoid confusion, the gear ratio should be clearly specified.

Simple train with idler gear


Gear Ratio =
Product of number of teeth in driven gears _100 x 50 5
Product of number of teeth in driver gears 3Q x 100 ~ 3

Without Idler
Gear Ratio as above
5
Gear ratio = or 5:3
3
The idler does not alter the overall Velocity Ratio.

Note: This means that gear ‘C’ travels or rotates at 3/5 the speed of gear ‘A’.

2-266 Total Training Support Ltd


Issue 2 - September 2016 © Copyright 2016
Module 2.2 Mechanics
Siodufe 2.2 Mechanics

Total Training Support Ltd 2-267


©Copyright 2016 Module 2.2 Mechanics Issue 2 - September 2016
Module 2.2 Mechanics
Compound gear train
In a compound train at least one shaft carries a compound gear or two wheels which rotate at
the same speed. The advantage of a compound gear train is that it can produce a high gear
ratio without the disproportionate gear sizes that would be necessary in a simple gear train.

Consider that in the compound gear train illustrated, the drive gear A has 15 teeth and an
angular speed of 240 rev/min. For one revolution of gear A the meshing gear B (30 teeth) will
rotate half a revolution. Gear C is mounted on the same shaft as gear B and will therefore also
rotate half a turn. For half a turn of gear C (1/2 x 18 = 9 teeth), gear D (36 teeth) will rotate
through one quarter of a revolution.

Gear ratio = ■ Pig!gEg..,ggyg^.. =1 or 4:1


Distance moved by output %

Generally, for a compound gear train, in which A and C are the driver gears and B and D are
the driven gears:

Product of number of teeth in driven gears


Gear ratio =
Product of number of teeth in driver gears
Using the figures for the previous gear train:

Gear Ratio = 30 *36 ~ 4:1 (as before)


15x18 1

Note: This means that gear ‘D’ travels or rotates at % of the speed of gear ‘A’.
Gear ratio = Product of number of teeth in driven gears
Product of number of teeth in driver gears

Driven gear is running at Z of driver gear

If driver is rotating at 1,000 rpm then the final driven gear is rotating at
1,000x^- = 300 RPM

2-268 Total Training Support Ltd


issue 2 - September 2016 © Copyright 2016
Module 2.2 Mechanics
Module 2.2 Mechanics

Total Training Support Ltd 2-269


© Copyright 2016 Module 2.2 Mechanics Issue 2 - September 2016
Module 2.2 Mechanics
Worm gears
One revolution of the worm moves the gear one tooth and so the ratio of the angular velocities
is given by the number of teeth on the wheel.
Angular speed of worm Al , ,, .
- - ;----c———-—r = the number of teeth in wheel
Angular speed of wheel

therefore

gear ratio = the number of teeth in wheel

Epicyclical reduction gear


There are several basic types of planetary gear arrangements. In one type the ring gear is fixed
and the sun gear is the driven gear. The sun gear meshes with and drives three equal-spaced
gears known as planet gears. These gears are mounted on a carrier or spider and rotate
independently on their axles.

Surrounding this gear train is an internally toothed wheel known as the annulus or ring gear
whose teeth are in mesh with the planet gears, if the ring gear is fixed, rotation of the sun gear
will cause the planet gears to rotate about their axes and at the same time to move around the
ring gear. This causes the planet gear spider to rotate at a lower speed than the sun gear.
When high torque is to be transmitted, the gear tooth is helical.

The speed reduction of this arrangement is


Gear ratio = on sun 9ear + teeth on rin^ 9ear
Teeth on sun gear

Another type of planetary gear system has the ring gear as the driven gear and the sun gear is
fixed. The output shaft is mounted on the spider that holds the planetary gears. Again the output
turns more slowly that the input.
Gear ratio = ~^eeth on sun gear + teeth on rjn9 ^ear
Teeth on sun gear

In a third type the spider is fixed, input being provided by a shaft attached to the sun gear, and
output by a shaft connected to the rotating ring gear. This arrangement is commonly used in
gas turbine air driven starter motors.

Epicyclical reduction gears are also used in the reduction gear assemblies of turbo prop
engines.

2-270 Total Training Support Ltd


Issue 2 - September 2016 ©Copyright 2016
Module 2.2 Mechanics
BioduBe 2.2 Mechanics

Output
.shaft
Fixed ring gear

Sun gear driven


by input shaft

Planetary gears
mounted on a:
spider attached to
output shaft

Ring gear driven


by input shaft

Fixed sun gear

Planetary gears;
mounted on a:
spider attached to
output shaft

Total Training Support Ltd 2-271


© Copyright 2016 Module 2.2 Mechanics Issue 2 ~ September 2016
Module 2.2 Mechanics
Reduction gears
Reduction gears are driven by gas turbine engines, and normally drive a propeller or helicopter
rotor.

Power turbines run at speeds, which suit the design characteristics of the rest of the engine.
This does not have anything in common with the speed of the propeller, which is set by its own
characteristics, chiefly blade diameter.

This, as has already been seen, compromises the design and operation of the coupled turbine
engine but is much less problematic in a free turbine design.

As power turbines can be spinning at up to 38,500 RPM and anything much over 2,000 RPM is
considered quite fast for a propeller, it is obvious that a means of reducing this speed difference
must be found. A suitable gear train will carry out this function.

Types of reduction gear


There are two main types available to the designer.

• The parallel spur gear type


• The epicyclic type

Parallel spur gears


This type of gear train has the advantage of being mechanically simple and therefore relatively
cheap to manufacture.

2-272 Total Training Support Ltd


Issue 2 - September 2016 © Copyright 2016
Module 2.2 Mechanics
Module 2.2 Mechanics

Wheel
(internal teeth) Pinion

Parallel spur gears - external and

Total Training Support Ltd 2-273


©Copyright 2016 Module 2.2 Mechanics Issue 2 - September 2016
Module 2.2 Mechanics
Epicyclical reduction gears
A gear train consisting of a sun (driving) gear meshing with and driving three or more equal­
spaced gears known as ‘planet pinions’. These pinions are mounted on a carrier and rotate
independently on their own axles. Surrounding the gear train is an internally toothed ‘annulus
gear’ in mesh with the planet pinions.

If the annulus is fixed, rotation of the sun wheel causes the planet pinions to rotate about their
axes within the annulus gear, this causes the planet carrier to rotate in the same direction as
sun wheel but at a lower speed. With the propeller shaft/secured to the planet pinion carrier, a
speed reduction is obtained with the turbine shaft (input shaft) and propeller shaft (output shaft)
in the same axis and rotating in the same direction.

If the annulus is free, rotation of the sun wheel causes the planet pinions to rotate about their
axles within the annulus gear. With the planet pinion carrier fixed and the propeller shaft
attached to the annulus gear, rotation of the planet pinions causes the annulus gear and
propeller to rotate in the opposite direction to the sun wheel and at a reduced speed.

Compound spur epicyclic


Compound epicyclic reduction gears enable a greater reduction in speed to be obtained without
resorting to larger components. They may be of either the fixed or free annulus type.

Gear train/epicyclic combination


Some turbo-props will use a gear train or a combination of gear train and epicyclic.

Small Large
planet wheel planet wheel

Planet carrier

An epicyclic gear

2-274 Total Training Support Ltd


Issue 2 - September 2016 ©Copyright 2016
Module 2.2 Mechanics
Modute 2.2 Mechanics

Planet pinions

Annulus gear (fixed)

Planet pinions

Carrier (fixed)

Annulus gear Epicyclic gear train with fixed planet gear

Total Training Support Ltd 2-275


©Copyright 2016 Module 2.2 Mechanics Issue 2 - September 2016
Module 2.2 Mechanics

Intentionally Blank

2-276 Total Training Support Ltd


Issue 2 - September 2016
Module 2.2 Mechanics © Copyright 2016
Module 2.2 Mechanics
2.2.3 Dynamics
Mass
Mass (symbolized ‘m’) is a dimensionless quantity representing the amount of matter in a
particle or object. The standard unit of mass in the International System (SI) is the kilogram (kg),
and the slug in the imperial system.

1 slug = 14.59 kilogram

In physics, mass is a property of a physical body. It is a measure of an object’s resistance to


acceleration (a change in its state of motion) when a force is applied. It also determines the
strength of its mutual gravitational attraction to other bodies.

Force and inertia

inertia
Inertia is the resistance of any physical object to any change in its state of motion (this includes
changes to its speed, direction or state of rest). It is the tendency of objects to keep moving in a
straight line at constant velocity.

Newton’s laws
The rapid advance in aviation in the first half of the last century can be attributed in large part to
a science of motion which was presented to the world three centuries ago by Sir Isaac Newton,
a British physicist. Published in 1686, Newton’s treatise on motion, The Principal, showed how
all observed motions could be explained on the basis of three laws. The applications of these
laws have led to great technological advances in the aerodynamics, structure, and power plant
of aircraft. It is safe to say that any future improvements in the performance of aircraft will be
based on these laws. This chapter will be devoted to Newton’s laws, examining some of their
applications in aviation.

Newton’s first law (the ‘inertia’ law)


The old magician’s trick of pulling a cloth out from under a full table setting is not only a
reflection of the magician’s skill but also an affirmation of a natural tendency which dishes and
silverware share with all matter. This natural tendency for objects at rest to remain at rest can
be attested to by any child who ever tried kicking a large rock out of his path. It is also a well-
known fact that once a gun is fired, the command “stop” has no effect on the bullet Only the
intervention of some object can stop or deflect it from its course. This characteristic of matter to
persist in its state of rest or continue in whatever state of motion it happens to be in is called
inertia. This property is the basis of a principle of motion which was first enunciated by Galileo in
the early part of the 17th century and later adopted by Newton as his first law of motion.

The first law of motion is called the law of inertia. It can be summarized:

A body at rest remains at rest and a body in motion continues to move at


constant velocity unless acted upon by an unbalanced external force.

The importance of the law of inertia is that it tells us what to expect in the absence of forces,
either rest (no motion) or straight line motion at constant speed. A passenger’s uncomfortable
experience of being thrown forward when an aircraft comes to a sudden stop at the terminal is
an example of this principle in action. A more violent example is the collision of a vehicle with a
stationary object. The vehicle is often brought to an abrupt stop.

Total Training Support Ltd 2-277


© Copyright 2016 Module 2.2 Mechanics issue 2 “ September 2016
Module 2.2 Mechanics

Intentionally Blank

2-278 Total Training Support Ltd


Issue 2 - September 2016
Module 2.2 Mechanics © Copyright 2016
Module 2.2 Mechanics IjjjB
Unconstrained passengers continue to move with the velocity they had just prior to the collision
only to be brought to rest (all too frequently with tragic consequences) by surfaces within the
vehicle (dashboards, wind-shields, etc.).

A less dramatic example of Newton’s first law comes from the invigorating activity of shovelling
snow. Scooping up a shovel full of snow, a person swings the shovel and then brings it to a
sudden stop. The snow having acquired the velocity of the shovel continues its motion leaving
the shovel and going off onto the snow pile,

Newton’s second law


An aeroplane accelerates down the runway a distance of 3,000 feet, takes off and begins its
climb at 6,000 feet per minute quickly reaching a cruising altitude of 35,000 feet, where it levels
off at a speed of 260 knots. Subsequently, the aeroplane may have to perform a variety of
manoeuvres involving changes in heading, elevation, and speed. Every aspect of the
aeroplane’s motion is governed by the external forces acting on its wings, fuselage, control
surfaces and power plant. The skilled pilot using his controls continually adjusts these forces to
make the aeroplane perform as desired.

The interplay between force and motion is the subject of Newton’s second law. An
understanding of this law not only provides insight into the flight of an aeroplane, but allows us
to analyse the motion of any object.

While Newton’s first law tells us that uniform velocity is to be expected when an object moves in
the absence of external forces, the second law states that to have a change in speed or
direction an unbalanced force must act on the object. Using acceleration to describe the change
in motion of an object, the second law can he expressed:

Fnet = ma

In words, the second law states that a net or unbalanced force acting on an object equals the
mass of the object times the acceleration of that object.

Here, the net force is the total force acting on the object, obtained by adding vectorially all of the
forces influencing the object. The mass is a scalar quantity. However, both the net force and the
acceleration are vector quantities. Mathematically, this means that they must always point in the
same direction. That is, at each instant the acceleration is in the same direction as the net force.

Before we consider cases where the net force acting on a body is not zero, it is most important
to understand that sometimes the net force acting on a body is zero. The vector sum of the
forces acting on the body in the x-direction is zero and the vector sum of the forces acting on
the body in the y-direction is also zero. In this case we say that the body is in equilibrium. From
the law, net force equals mass times acceleration, we know that since the net force is zero the
acceleration is also zero. Zero acceleration means that the velocity of the body in not changing
in direction or in magnitude. This means that the body is moving in a straight line with constant
speed or it has the constant speed, zero (it is at rest). If we observe that a body is at rest, we
know that ail the forces on this body are balanced. Similarly, if a body is moving in a straight line
with constant speed, all the forces acting on this body are balanced.

Totai Training Support Ltd 2-279


©Copyright 2016 Module 2.2 Mechanics Issue 2 - September 2016
Module 2.2 Mechanics
For example, if an aeroplane is travelling on a straight stretch of runway at constant speed,
there are four forces acting on this aeroplane: the earth is pulling down on the aeroplane (its
weight), the earth is pushing up on the aeroplane (the normal force), the engine is giving a
forward thrust to the aeroplane, and frictional forces (air resistance, tires on runway, etc.) are
acting backward. This is illustrated in the figure below.

Next, we must consider some examples where the net force acting on a body is not zero. The
body is accelerating. The body is experiencing a change in its direction or in its speed or both.
As a first example, an aeroplane accelerating down a runway gets a change in velocity in the
direction of its motion. This is the same direction as the thrust provided by the power plant.

In the figure below, note that the thrust is greater than the frictional forces. The net forward force
is the thrust minus the friction. It is this net forward force that results in the acceleration of the
aeroplane.

The four forces acting on an aeroplane

Thrust - Drag = (mass) (acceleration)


T - D =ma

Trust

Thrust, drag and acceleration forces

2-280 Total Training Support Ltd


Issue 2 - September 2016 ©Copyright 2016
Module 2.2 Mechanics
A propeller produces thrust be accelerating a large mass of air (per second) rearwards.

Newton’s third law is used to describe how the action of propelling a mass of air
rearwards, produces a forward force known as ‘thrust’.

Compressor Shaft Turbine

Force = Mass x Acceleration

A jet produces thrust by accelerating a small mass of air (per second) rearwards, but
with a much greater acceleration than a propeller would.

Total Training Support Ltd 2-281


©Copyright 2016 Module 2.2 Mechanics Issue 2 - September 2016
Module 2.2 Mechanics
Newton’s third law
Newton’s third law is sometimes referred to as the law of action and reaction. This law focuses
on the fact that forces, the pushes and pulls responsible for both the stability of structures as
well as the acceleration of an object, arise from the interaction of two objects, A push, for
example, must involve two objects, the object being pushed and the object doing the pushing.

Every action has an equal and opposite reaction

The third law states that no matter what the circumstance, when one object exerts a force on a
second object, the second must exert an exactly equal and oppositely directed force on the first.
An apple hanging from a tree is pulled by the earth with a force which we call its weight.
Newton’s third law tells us that the apple must pull back on the earth with an exactly equal force.
The weight of the apple is a force on the apple by the earth, directed downward. The force
which the apple exerts back on the earth, is a pull on the earth directed upward. Another force
acting on the apple is the upward puli exerted by the branch. The law of action and reaction tells
us that the apple must be pulling down on the branch with the same magnitude of force.

People are often confused by this principle because it implies, for instance, that in a tug of war
the winning team pulls no harder than the losing team. Equally enigmatic is how a horse and
wagon manage to move forward if the wagon pulls back on the horse with the same force the
horse pulls forward on the wagon. We can understand the results of the tug of war by realizing
that the motion of the winning team (or losing team) is not determined exclusively by the pull of
the other team, but also the force which the ground generates on the team members’ feet when
they ‘dig in’. Recall, it is the net force, the sum of all of the acting forces which determines the
motion of an object.

The results of a ‘tug of war’ can be quite different if the ‘winning team’, no matter how big and
strong, is standing on ice while the ‘losing team’ is able to establish good solid footing on rough
terrain.

Similarly, the horse moves forward because the reaction force which the ground exerts in the
forward direction on its hooves is greater than the backward pull it receives from the wagon. By
focusing now on the wagon, we see that it moves forward because the forward pull of the horse
is greater than the backward pull of friction between its wheels and the ground.

2-282 Total Training Support Ltd


Issue 2 - September 2016 © Copyright 2016
Module 2.2 Mechanics
Module 2.2 Mechanics

The apple pulls down


The earth pulls down
on thebranch.
on the appie.
The branch pulls up
The apple pullsup
onThe apple.
on the earth.

Gravitational force between two objects

Equal and opposite forces of Newton’s 3rd law

Total Training Support Ltd 2-283


©Copyright 2016 Module 2.2 Mechanics Issue 2 - September 2016
Module 2.2 Mechanics
One of the main difficulties people have with the third law comes from not realizing that the
action and reaction forces act on different objects and therefore can never cancel. Another
difficulty comes from forgetting that the motion of an object is determined by the sum of all of
the forces acting on that object.

In canoeing or rowing, a paddle is used to push water backward. The water reacts back on the
paddle generating a forward force which propels the boat.

Consider now a propeller as shown in the figure opposite.

The plane of rotation of the propeller is assumed to be perpendicular to the plane of the paper.
The flow of air is from left to right. We can imagine the action of the propeller is to take a
mass (m) of air on the left and accelerate it from some initial velocity (u) to a final velocity (v) to
the right of the propeller. The acceleration of this air mass requires a force which is provided by
the propeller. The air mass, in turn, reacts with an equal and opposite force on the propeller.
This reaction force of the air on the propeller provides the thrust for a propeller driven
aeroplane. The acceleration of the air mass is:

Substituting this into Newton’s second law, we find for the net force on the air mass:
- v-u
F = m-------
t

Both of the velocities (u and v) are the velocities relative to the plane of rotation of the propeller.
The time (t) is the time involved in accelerating the air mass from u to v.

By Newton’s third law, the thrust, which is the force the air mass exerts back on the propeller, is
equal in magnitude to F. Therefore, the thrust (T) is given by:

Recall that we have a symbol for ‘change in’, this means that we can write the above formula
as:

The velocities of the air mass are relative to the aeroplane, and therefore change as the
aeroplane’s speed changes. Also the time involved in accelerating the air mass changes with
the speed of the aeroplane. This causes considerable variation in the thrust provided by a
propeller.

Example:

Each second a propeller accelerates an air mass of 12.2 slugs from rest to a velocity of
137 ft/sec. How much thrust is provided?
137 ft/sec-0
T = (12.2 slugs)
1 sec

T = 1,670 lbs

2-284 Total Training Support Ltd


Issue 2 - September 2016
Module 2.2 Mechanics © Copyright 2016
Modute 2.2 Mechanics

..:,-.- ■ ^r>.;■■-■:■/rs?S.;”r »• .'■ :v ■ . •■• !*•.'» ■..■■;■■. :><■

Forte of the paddle Force of the water


on the water on the paddle

Equal and opposite forces of a paddle

"Tt—

Action of a propeller

Total Training Support Ltd 2-285


© Copyright 2016 Module 2.2 Mechanics Issue 2 - September 2016
IgllF Module 2.2 Mechanics
In contrast to the reciprocating engine driven propeller which imparts a small change in velocity
to a relatively large mass of air, a turbojet induces a large change in velocity to a relatively small
mass of air. Here, the sole action of the Jet engine is considered to be the intake of a mass of
air at some velocity (u) and its exhaust at a higher velocity (v).

The figure opposite is a sketch of a turbojet engine. The velocity (u) in the figure denotes the
relative intake velocity and v denotes the exhaust gas velocity. The thrust formula which was
obtained above for the propeller will now be applied to a Jet engine. The thrust formula above
can be rewritten:
T m v------
T-— mu
t t

T = Gross thrust - Ram drag

The gross thrust is provided by the exhaust gases. The ram drag of the incoming air is due to
the speed of the aeroplane. The effect of the ram drag is to reduce the thrust provided by the
engine as the speed of the aeroplane increases.

Example:

A jet engine has a mass air flow of 23 kg/sec. During a static test (initial velocity = 0) the
exhaust velocity was measured to be 580 m/sec. Determine the thrust produced.

Note that the ram drag is zero since v1 is zero. Therefore, the thrust is equal simply to the gross
thrust.
-r m
T =— v
t

Substituting the given values, we have:

T= (580 m/sec) = 13.300N


1 sec

2-286 Total Training Support Ltd


Issue 2 - September 2016
Module 2.2 Mechanics © Copyright 2016
Modute 2.2 Mechanics

Total Training Support Ltd 2-287


©Copyright 2016 Module 2.2 Mechanics Issue 2 - September
Module 2.2 Mechanics
Example:

What would the thrust have been if the jet engine in the previous example had been in an
aeroplane moving at 250 knots? Assume the same mass flow and exhaust velocity.

Note that the ram drag is not zero in this case. In order to calculate this ram drag we must use
the formula:

ram drag =y-u

Before substituting, we must express the initial velocity in m/sec.


t 1.668 ft/sec 0.3048 m
u = 250 knots x---------------x-------------
1 knot 1 ft
= 129 m/sec
23 ka
ram drag =------~ 129 m/sec
1 sec
ram drag = 2,970 N

T = gross thrust - ram drag

T = 13,300 N~ 2,970 N

T = 10,300 N

2-288 Total Training Support Ltd


Issue 2 - September 2016
Module 2.2 Mechanics ©Copyright 2016
Module 2.2 Mechanics
Example:

During a static test (initial velocity = zero), a jet engine produced a thrust of 16,000 lbs with an
air mass flow of 8.23 slugs/sec. Determine the exhaust gas velocity of the engine.
T = y(V-U)

Since u is zero, the ram drag is zero and T = gross thrust,


m
T ~— v
t

We solve for the final velocity:


T
v=—v
rn
t
16,000 lbs
v - ------ -------------
8.23 slugs/sec

v = 1,940 ft/sec

The air intake velocity of a turbojet will be approximately equal to the airspeed of the aeroplane.
Let us again examine the thrust formula.

It can be seen that the thrust may be increased in two ways, either by increasing the air mass
flow through the engine (M/t) or increasing the exhaust gas velocity (v).

Newtons3Laws
https://youtu.be/SciwDyba8m8

Total Training Support Ltd 2-289


©Copyright 2016 Module 2.2 Mechanics Issue 2 - September 2016
Module 2.2 Mechanics
Example:

An aircraft is powered by two turbofan engines. Flying at sea level with a velocity of
154 m/sec the air intake velocity is 154 m/sec and the air exhaust velocity is 224 m/sec. The
airflow through each engine is 109 kg per second. Determine the thrust of each engine.
T ” -255JS. (224 m/sec - 154 m/sec)
1 sec

T - 7,630 N

Example:

Aa airliner is equipped with four jet engines. Cruising at 220 knots, each engine was found to
be providing 1,420 lbs of thrust. If the airflow through each engine was 1.55 slug/sec, what
was the exhaust gas velocity?

U = 220 knots = 371 ft/sec v = 1,290 ft/sec


Tt = v_u

m
Tt
v=—+u
m
v=(MgO.M....gga +371 ft/sec
1.55 slug

v = 1,290 ft/sec

2-290 Total Training Support Ltd


Issue 2 - September 2016 © Copyright 2016
Module 2.2 Mechanics
Module 2.2 Mechanics
Worksheet

1. Find the mass of an object which accelerates at 5 m/sec2 when acted on by a net force of
one newton.

2. Find the acceleration of a 3 slug object experiencing a net force of 12 lbs.

3. Find the net force on a 5 slug object which is accelerating at 3 ft/sec2.

4. An aeroplane of mass 6,000 kg is observed to accelerate at the start of its take-off at


4 m/sec2. What is the net forward force acting on the aeroplane at this time?

5. During a static test, a Continental engine driving a two blade constant speed propeller was
found to accelerate each second a mass of 140 kg from rest to a velocity of 40 m/sec.
Determine the thrust on the propeller.

6. A piston engine drives a two blade propeller. Each second, 8 slugs of air are given a
change in velocity of 122 ft/sec. How much thrust is generated on the propeller?

7. The turbofan engine has an exhaust gas velocity of 321 m/sec and an airflow of 50 kg/sec.
Find the static thrust of the engine.

8. An aeroplane weighs 36,000 Sbs. The forward thrust on the aeroplane is 20,000 lbs and the
frictional forces (drag) add up to 2,000 lbs. What is the acceleration of this aeroplane? Hint:
Be sure to find the mass of the aeroplane from its weight.

Total Training Support Ltd 2-291


©Copyright 2016 Module 2.2 Mechanics Issue 2 - September 2016
Module 2.2 Mechanics

Intentionally Blank

2-292 Total Training Support Ltd


Issue 2 - September 2016
Module 2.2 Mechanics © Copyright 2016
Module 2.2 Mechanics
Answers

1. 0.2 kg

2. 4 ft/sec2

3. 151b

4. 24,000 N

5. 5,600 N

6. 976 lb

7. 16,050 N

8. 16 ft/sec2

Total Training Support Ltd 2-293


© Copyright 2016 Module 2.2 Mechanics Issue 2 - September 2016
Module 2.2 Mechanics

Intentionally Blank

2-294 Total Training Support Ltd


Issue 2 - September 2016
Module 2.2 Mechanics ©Copyright 2016
Module 2.2 Mechanics
Work, energy and power

Work
Work is done on a body when a force acts through a distance. The definition of work involves
the force acting on the body (F) the distance through which this force acts (S) and the angle (0)
between the force vector and the distance vector. The definition of work is:

W = FS cos G

Very often the force vector and the distance vector act in the same direction. In this case, the
angle (0) is a zero-degree angle. If you check on your calculator, you will find that the cosine of
a zero-degree angle is equal to one. This simplifies things in this case because then work is
simply equal to the product of force times distance.

The unit of work in the imperial system is the foot-lb. Note that the two units are multiplied by
each other. Students tend to write ft/lb. This is incorrect. The unit is not feet divided by pounds.
In the metric system, the unit is the newton-metre or the joule (J). Note that the newton-metre
has a name, the joule. The foot-lb has no special name.

Example:

A puck lies on a horizontal air table. The air table reduces the friction between the puck and the
table to almost zero since the puck rides on a film of air. A player exerts a force of 70 lbs on this
puck through a distance of 0.5 feet, and he is careful that his force is in the same direction as
the distance through the force is applied. The player has done 35 ft lbs of work on the puck.

Example:

A book weighing 8 pounds is raised a vertical distance by a student demonstrating work. The
book is raised 2 feet. The student has done 16 ft lbs of work.

Example:

A sled is dragged over a horizontal snowy surface by means of a rope attached to the front of
the sled. The rope makes an angle of 28° with the horizontal. The sled is displaced a distance a
50 ft. The worker exerts a force of 35 pounds. How much work does the worker do? We use the
formula:

W = FS cos 0
W = (35 lbs) (50 ft) cos 28°
W = 1,550 ft lbs

Sometimes the force and the displacement are in the opposite directions. This situation gives
rise to negative work. Note, in this case, the angle between the force and the displacement is a
180° angle. The cosine of 180° is negative one.

One example of negative work occurs when a body is lowered in a gravitational field. If a
student carefully lowers a book weighing 15 pounds through a distance of 2 feet, we note that
the displacement vector points downward and the force vector point upwards.

W - Fs cos 0
W =(15 lbs) (2 ft) cos 180°
W = (15 lbs) (2 ft) (-1)
W = -30 ft lbs

Total Training Support Ltd 2-295


© Copyright 2016 Module 2.2 Mechanics Issue 2 - September 2016
Module 2.2 Mechanics
Energy
The concept of energy is one of the most important concepts in ail of physical science. We often
hear of energy sources, alternate energy, shortage of energy, conservation of energy, light
energy, heat energy, electrical energy, sound energy, etc. What is the exact meaning of this
word, energy?

Sometimes energy is defined as the ‘capacity to do work’. This definition is only a partial
definition. However, it has the advantage of immediately relating the concept of energy to the
concept of work. These two ideas are intimately related to each other.

Energy is a quality that a body has after work has been done on this body. Once work has been
done on a body of mass (m) this body has energy. The body can then do work on other bodies.
Consider the following situation:

A body of mass (m) was resting on a horizontal air table. A player exerted a horizontal force (F)
on this mass through a distance (s). Since the angle between the force and the displacement
was a zero-degree angle, the work done on this body was simply Fs. At the instant the player
removed his hand from the body we note two facts. The body accelerated while the force (F)
was acting on the body and the body has acquired a velocity (v) during this time of
acceleration (a). The body has moved through a distance (s) in time (t).

s = % at2

Also note that the force (F) is related to the acceleration by the relation:

F = ma

We now look again at this body at the instant the force (F) has ceased acting. We note that
work (W) has been done on this body and that the body moves with speed (v).

W = Fs = (ma) (% at2)

W = % m (at)2

Now we note that the speed obtained by the body during the time of acceleration is given by the
equation:

V =at

Therefore, we can substitute v for at in the equation above.

W = % mV2

The equation we have obtained is the defining equation for a quantity known as kinetic energy.

Usually, we use the symbol ‘KE’ for kinetic energy.

KE = % mV2

After the work has been done on the mass (m) it moves off on the frictionless air table with this
kinetic energy. This body now is capable of doing work on other bodies that it contacts. For
example, it probably will strike the edge of the table. When this happens this kinetic energy will
be changed into other types of energy such as sound energy or heat energy.

2-296 Total Training Support Ltd


Issue 2 - September 2016 © Copyright 2016
Module 2.2 Mechanics
Module 2.2 Mechanics
We note that the initial kinetic energy of the mass (m) was zero. This is true because the body
was initially as rest We can say that the work done on the body is equal to the change in the
energy of the body.

Gravitational potential energy


Another equally important situation where an agent easily can do work on a body (and thus give
the body energy) occurs when the agent raises a body vertically in a gravitational field, at the
surface of the earth.

In this case, the work done on the body again equals the force applied multiplied by the
distance the body is raised,

W = Fs

W = (weight of body) (distance raised)

We recall that w = mg. Also since the distance is a vertical distance we use the symbol ‘h’ for
height. In our discussion we will assume that the symbol ‘h’ always represents the vertical
distance of the body above the surface of the earth.

Therefore, we write:

W = mgh

Again we have a case where an agent did work on a body and the body has acquired ‘energy’.
This type of energy is known as gravitational potential energy, however, we usually symbolize it
as ‘PE’.

PE = mgh

If we neglect air resistance (which results in loss of energy to heat), we note that there is a
conservation of kinetic and potential energy of a body moving in a gravitational field. As a body
falls from a height (h) and moves closer to the surface of the earth, its potential energy
decreases and its kinetic energy increases while it is falling. Therefore, there is an easy way of
finding the speed of a falling body during any instant of its fall.

The units for energy are the same as the units for work, the joule (J) in the metric system and
the foot-pound in the imperial system.

Example:

A body of mass 4 slugs is held by an agent at a distance of 6 ft above the surface of the earth.
The agent drops the body. What is the speed of the body when it is on the way down and at a
distance of 2 feet above the earth’s surface?

We note that the initial potential energy is equal to the sum of the kinetic and potential energies
on the way down (wd).

PE = PEwd + KEwd
(4 slug) (32 ft/sec2) (6 ft) = (4 slug) (32 ft/sec2) (2 ft) + 1/2 (4 slug) V2

Total Training Support Ltd 2-297


© Copyright 2016 Module 2.2 Mechanics Issue 2 - September 2016
Module 2.2 Mechanics
Example:

A body of mass, 10 kg, falls to the earth from a height of 300 m above the surface of the earth.
What is the speed of this body just before it touches ground?

PEi = KEf

(10 kg) (9.8 m/sec2) (300 m) = (10 kg) V2

2,940 m2/sec2 = 1/2V2

5,880 m2/sec2 = V2

V = 76.7 m/sec

The kinetic energy that the body has just before it reaches the ground immediately changes to
sound energy and heat energy on impact. It may also ‘squash’ any body in its path or make an
indentation in the earth - this is strain energy (energy to deform).

Power
Power is the rate of doing work. The more rapidly a piece of work can be done by a person or a
machine the greater is the power of that person or machine.

We define power by the following equation:


Work Force x Distance
Power;
Time Time
In symbols:

p=f®
t

In the imperial system the unit of power is the horsepower and in the metric system the unit is
the watt.

Conversion factors exist giving information regarding these units.

1 horsepower = 550 5^®. = 33,000


sec min
1,000 watts = 1 kilowatt

Energy
https://youtu.be/F9DPPwsBw6o

2-298 Total Training Support Ltd


issue 2 - September 2016
Module 2.2 Mechanics © Copyright 2016
Module 2.2 Mechanics
Example:

An aircraft engine weighing 4,000 lbs is hoisted a vertical distance of 9 feet to install it in an
aircraft The time taken for this piece of work was 5 minutes. What power was necessary? Give
the answer in ft Ib/sec and in horsepower.
Fs ~ (4,000 lbs) (9 ft)
t 300 sec
1 HP
P= 120 ftlbs/sec
500 ftlbs/sec

P - 0.218 hp

Example:

An elevator cab weighs 6,000 N. It is lifted by a 5 kW motor. What time is needed for the cab to
ascend a distance of 40 m?

(6,000 N) (40 m)
5,000 watts
t =48 seconds

Alternate form for power


We can put our formula for power in another form by recognizing
s
That — is speed (v). This leads to the formula:
t

p = Fv

This form is particularly useful for obtaining an expression for the power output of a turbine
engine. These engines are ordinarily rated in terms of the thrust which they produce. To obtain
an expression for their power output it is necessary to multiply their thrust by the speed of the
aeroplane. This thrust power, which is usually expressed in units of horsepower (THP - thrust
horsepower), can be obtained by multiplying the thrust in pounds by the speed in ft/sec and
dividing by 550 where the conversion 1 hp = 550 ft Ib/sec is used. Thus:
tpw - thrust in lbs x aircraft speed in ft/sec

Alternatively, we can take the speed of the aircraft in mph and use the conversion
1 hp = 375 mi Ib/hr to obtain:
tpw - thrust in lbs x aircraft speed in MPH

Total Training Support Ltd 2-299


© Copyright 2016 Module 2.2 Mechanics Issue 2 - September 2016
Module 2.2 Mechanics
Example:

A gas turbine engine is producing 5,500 lbs of thrust while the aeroplane in which the engine is
installed is travelling 450 mph. Determine the THP.
TPH = (55Q lbs) x (458 MPH) = 6,600 HP
375
It is important to note that while the thrust of a gas turbine engine may not vary much over a
particular range of aircraft speeds, the power must be recalculated each time the aeroplane
changes its speed.

Introduction to work and energy 1 Work and energy | Physics I Khan Academy
https://youtu.be/2WS1sG9fhOk

2-300 Totai Training Support Ltd


issue 2 - September 2016 © Copyright 2016
Module 2.2 Mechanics
Module 2.2 Mechanics
Worksheet

1. How much work is done by a person in raising a 45 lb bucket of water from the bottom of a
well that is 75 ft deep? Assume the speed of the bucket as it is lifted is constant.

2. A tugboat exerts a constant force of 5,000 N on a ship moving at constant speed through a
harbour. How much work does the tugboat do on the ship in a distance of 3 km?

3. A father has his 45 lb son on his shoulders. He lowers the child slowly to the ground, a
distance of 6 ft. How much work does the father do?

4. A 6 slug body has a speed of 40 ft/sec. What is its kinetic energy? If its speed is doubled,
what is its kinetic energy?

5. A 2 kg ball hangs at the end of a string 1 m in length from the ceiling of a ground level
room. The height of the room is 3 m. What is the potential energy of the ball?

6. A body of mass 3 slug is a distance of 77 ft above the earth’s surface and is held there by
an agent. The agent drops the body. What is the speed of the body just before it hits
ground?

7. An aircraft of mass 4 tonnes lands at 30 m/s and the pilot immediately applies the brakes
hard. The brakes apply a retarding force of 2,000N. How far will the aircraft travel before it
comes to rest.

8. A pile driver of mass 1,000 kg, hits a post 3 m below it. It moves the post 10 mm. What is
the kinetic energy of the pile driver?

9. A pile driver of mass 1,000 kg, hits a post 3 m below it. It moves the post 10 mm. With what
force does it hit the post when it hits the post?

10. An aircraft engine weighing 12,000 N is lifted by a 3.6 kW motor a distance of 10 m. What
time was needed?

11. A hand-powered hoist is used to lift an aircraft engine weighing 3,000 lbs a vertical distance
of 8 ft. If the worker required 4 minutes to do this job, what horsepower was developed by
the mechanic?

12. How long does it take a 5 kW motor to raise a load weighing 6,000 lbs a vertical distance of
20 ft?
(Hint: convert kW to ft Ib/sec first)

Total Training Support Ltd 2-301


© Copyright 2016 Module 2.2 Mechanics Issue 2 - September 2016
Module 2.2 Mechanics

Intentionally Blank

2-302 Total Training Support Ltd


Issue 2 - September 2016
Module 2.2 Mechanics ©Copyright 2016
Modute 2.2 Mechanics
Answers

1. 3,370 ft lb

2. 15,000,000 J

3. -270 ft lb (note the negative sign!)

4. 4,800 ft ib; 19,200 ft lb

5. 39.2 J (or 40 of g = 10 m/s2)

6. 70 ft/sec

7. 900 m

8. 30,000 J

9. 3 MN

10. 33 sec

11. 100 ft Ib/sec 0.182 hp

12. 32.5 sec

Total Training Support Ltd 2-303


© Copyright 2016 Module 2.2 Mechanics Issue 2 - September 2016
Module 2.2 Mechanics

Intentionally Blank

2-304 Total Training Support Ltd


Issue 2 - September 2016
Module 2.2 Mechanics © Copyright 2016
Module 2.2 Mechanics
Momentum and conservation of momentum

Definition of momentum
Momentum is a vector quantity defined as the product of mass times velocity. Note that
velocity (V) is also a vector quantity. We write the defining equation as:

Momentum = mV

Momentum is a very important quantity when we are dealing with collisions, because it is con­
served in all such cases.

Conservation of momentum
In a collision, there are always at least two bodies that collide. We will deal only with collisions of
two bodies. We will also limit our discussion to collisions occurring in one dimension. Such
collisions are called ‘head-on’ collisions.

At this time, we need to recall two of Newton’s laws. We need Newton’s second law, F = ma,
and Newton’s third law, which tells us that if two bodies collide, the force that the first body
exerts on the second body is equal in magnitude and opposite in direction to the force that the
second body exerts on the first body. Also recall that the acceleration (a) equals the change in
the velocity (symbolized by the Greek letter delta, A) divided by the time (t).

Now let us visualize two bodies of masses, mi and m2 on a one-dimensional track.

If these two bodies collide, we have four different velocities to consider. We will name these
velocities very carefully.

Introduction to momentum I Impacts and linear momentum | Physics I Khan Academy


https://youtu.be/XFhntPxowOU

VT = the velocity of body one before the collision

Vi” = the velocity of body one after the collision

V2’ = the velocity of body two before the collision

V2” = the velocity of body two after the collision

From Newton’s two laws, we can conclude that:

After cancelling ‘t’, we obtain:

mi(Vi”-Vi’) = -m2(V2”-V2’)

If we remove the parentheses, transpose terms, and switch left and right parts we obtain:

rmVi’ + 1T12V2’ = miVi” + m2V25’

The equation tells us that the total momentum before the collision is equal to the total
momentum after the collision. Sometimes we say simply that ‘momentum is conserved’.

Recoil problems
The simplest example of the conservation of momentum is in recoil problems.
Total Training Support Ltd 2-305
©Copyright 2016 Module 2.2 Mechanics Issue 2 ~ September 2016
Module 2.2 Mechanics
Example:

A boy and a man are both on ice skates on a pond. The mass of the boy is 20 kg and the mass
of the man is 80 kg. They push on each other and move in opposite directions. If the recoil
velocity of the boy is 80 m/sec, what is the recoil velocity of the man?

First we note that both the man and the boy are at rest before the collision occurs.

miVT + m2V2’ = miVi” + m2V2”

(20) (0) + (80) (0) = (20) (80) + (80)V2”

0 = 1,600 + 80V2”

-1,600 = 1,600+ 8OV2”

-1,600 = 8OV2”

V2” = 20 m/sec

The negative sign indicates that the man recoils in the opposite direction from the boy.

2-306 Total Training Support Ltd


Issue 2 - September 2016
Module 2.2 Mechanics © Copyright 2016
Module 2.2 Mechanics
Collision problems
Whenever two bodies collide, momentum is always conserved. This is simply the result of
applying Newton’s second and third laws as we have done in the preceding discussion.

Sometimes kinetic energy is also conserved in a collision. This happens when the bodies are so
hard that there is very little deformation of the bodies in the actual collision process. Billiard balls
are a good example. These collisions are known as elastic collisions. We will derive a formula
for determining the velocities of the bodies after the collision has occurred.

Another type of collision that we will discuss is the perfectly inelastic collision. In this type of
collision, the bodies are deformed so much that they actually stick together after the collision.
An example would be the collision of two masses of putty. We will also do some problems for
this type of collision.

Inelastic collisions
We use the conservation of momentum for dealing with this type of collision. As we have said,
the colliding bodies stick together after impact. Therefore, the equation is simply:

miV? + m2V2’ = (mi + m2) V”

Note that we use the symbol V” for the common velocity of the two bodies (which are now one
body) after the collision.

It is important to include the signs of the velocities of the bodies in setting up momentum
equations. As usual, we use a positive sign for east and a negative sign for west, a positive sign
for north and a negative sign for south.

Example:

A truck of mass 1,550 kg is moving east at 60 m/sec. A car of mass 1,250 kg is travelling west
at 90 m/sec. The vehicles collide and stick together after impact.

What is the velocity of the combined mass after the collision has occurred?

VT =60 m/sec
mi = 1,550 kg

V2’ = -90 m/sec


m2 = 1,250 kg

miV? + m2V2’ = (mi + m2) V”

We will not include units in our substitution. However, we will note that the velocity, when we
obtain it, will be in m/sec.

(1,550) (60) + (1,250) (-90) = (1,550 + 1,250)V”

-19,500 = 2,800 V”

V” = -6.96 m/sec

Since the calculated velocity has a negative sign, we conclude that the combined mass is
travelling west after the impact has occurred.

Our answer is that the wreckage starts moving west with a speed of 6.96 m/sec.

Total Training Support Ltd 2-307


© Copyright 2016 Module 2.2 Mechanics Issue 2 - September 2016
Module 2.2 Mechanics
Sometimes the principle of conservation of momentum in the case of inelastic collision can be
used by the police to determine the speed of a vehicle engaged in a head - on collision.

Suppose that a large truck with a weight of 12,000 lbs (mass of 375 slugs) travelling east with
an unknown velocity enters into a head-on collision with a smaller truck of weight 6.400 lbs
(mass of 200 slugs) initially travelling west with a speed of 30 mph (44 ft/sec.). The trucks stick
together in the collision and marks on the highway indicate that the wreckage travelled a
distance of 120 feet east. The condition of the roadway (amount of friction) indicates that the
wreckage would travel for a time of 4 sec. Determine the initial speed of the large truck.

The equation:

can be used to determine the initial velocity of the wreckage. Note that the final velocity of the
wreckage is zero.

t 4 sec

Next, we can use the conservation of momentum equation to determine the velocity of the large
truck at the instant of the impact. We will use the symbol V to represent this velocity.

(375 slugs) (V) + (200 slugs) (-44 ft/sec.) = 575 slugs) (60 ft/sec.)

375V = 43,300

V - 115 ft/sec

V = 78.4 mph

2-308 Total Training Support Ltd


Issue 2 - September 2016 ©Copyright 2016
Module 2.2 Mechanics
Module 2.2 Mechanics
Elastic collisions
Elastic collisions are collisions that occur between bodies that deform very little in the collision.
Therefore, we assume that no energy is lost. An example of such a collision is the collision
between pool balls.
In elastic collisions, both kinetic energy and momentum are conserved. In an ordinary elastic
collision problem, we know the masses and the velocities of two bodies that will collide. We
want to predict, by a mathematical calculation, the velocities the bodies will have after the
collision has occurred, the two unknowns. If we write the two conservation equations, we have
two equations in these two unknowns. It is possible to solve these two equations for these two
unknowns. However, one of the conservation equations, the energy equation, is a ‘second
order’ equation. A ‘second order’ equation contains the squares of the unknowns. This makes
the solution more difficult. Instead, we will use an algebraic trick! The two conservation
equations can be solved together producing a third equation. This third equation and the
momentum conservation equation provide the two first order equations that we will use in
solving elastic collision problems.

We will write the two conservation equations:


Conservation of energy:

(1) % miVi’2 + m2V2’2 = % miVi”2 + % FT12V2”2

Conservation of momentum:

(2) rrnVT + m2V2’ = miVi” + m2V2”


Divide (1) by %:

(3) miVi’2 + m2V2’2 = miVi”2 + m2V2”2

Now in both (2) and (3), we will transpose some terms:

(4) miVi’ - rmv,” = m2V2” - m2V2’

(5) miVi’ - miVi” = + m2V2” - m2V2'


Factorise (4) and (5):

(6) rWi’~Vi”) = m2(V2"-V2’)

(?) mi(V?2 - Vi”2) = m2(V2”2 - V2’2)


In (7), factor again:

(8) mi(Vi’~
+ Vi”) = m2(V2” - V2’) (V2” + V2’)
Divide (8) by (6):

+ Vi”) (Va” + Va’)

Total Training Support Ltd


© Copyright 2016 2-309
Module 2.2 Mechanics Issue 2 - September 2016
Module 2.2 Mechanics
After cancelling common factors, we obtain:

(9) Vi’ + Vi”= V2” + V2’

Again we transpose terms:

(9) Vi’~V2” = V2”-~Vi’

In words, this equation says that the relative velocity of the balls before the collision is equal to
the relative velocity of the balls after the collision.

Equation (9) has been obtained algebraically from two equations, the conservation of
momentum and the conservation of energy. We use equations (2), the conservation of
momentum equation, and equation (9), called the relative velocity equation, to solve for the
velocities of the two bodies after an elastic collision.

We will rewrite these two important equations for future reference:

(2) miVT + ITI2V2’ = miVi” + 1TI2V2”

(9) Vi’-V2” = V2”-VT

In using these two equations, the two unknowns are usually Vi” and V2”, the velocities of the
two bodies after the collision has occurred. The known quantities are usually the two masses
and the velocities of the bodies before the collision. Also be careful to include the signs of the
velocities. If you forget to do this, you will always end up with incorrect results.

Example:

A billiard ball of mass 2 kg is moving east at 3 m/sec and undergoes an elastic collision with
another billiard ball of mass 3 kg moving west at 4 m/sec. Find the velocities of the two balls
after the collision.

rm = 2 VT = 3 (east)
m2 = 3 V2’ = 4 (west)

Substitute in equation (2):

(2) (3) + (3) (-4) - 2Vi” + 3V2”

-6 = 2Vi” + 3V2”

Substitute in equation (9):


3 „ (-4) = v2” - VT

7 = v2” - VT
Rewrite equations (10) and (11) putting the unknowns in the left members and in order.

(10) 2Vi” + 3V2” = -6


(11) -VT + VT = 7

2-310 Totai Training Support Ltd


Issue 2 - September 2016 © Copyright 2016
Module 2.2 Mechanics
Module 2.2 Mechanics
We now have two equations and two unknowns. There are several methods of solving such a
system of equations. We will use the method of addition. In this method we multiply either or
both of the equations by constants to make the coefficient of one of the unknowns in one of the
equations a positive number and to make the coefficient of this same unknown in the other
equation a negative number of the same magnitude. We then add the two equations to
eliminate one of the unknowns. We then solve for the other unknown by substituting in either
equation.

We will multiply (11) by the number 2.

-2Vi” + 2V2” = 14

Add (10) and (12):

5V2” = 8

V2” = 1.6m/sec

Substitute this value back into (11):

-Vi” + 1.6 = 7

Vi” = 5.4 m/sec

Note: we interpret a positive sign for the velocity as motion east and a negative sign as motion
west.

Our final result is that the 2 kg ball is moving west with a speed of 5.4 m/sec after the collision
and the 3 kg ball is moving east with a speed of 1.6 m/sec after the collision.

Total Training Support Ltd 2-311


©Copyright 2016 Module 2.2 Mechanics Issue 2 - September 2016
Module 2.2 Mechanics
Impulse
The concept of ‘impulse’ are merely an extension of Newton’s second law as discussed earlier,
Newton’s second law (F = m a) stated that the acceleration of an object is directly proportional
to the net force acting upon the object and inversely proportional to the mass of the object.
When combined with the definition of acceleration (a = change in velocity / time), the following
equalities result.

F=ma

or

F = m Av /1

If both sides of the above equation are multiplied by the quantity t, a new equation results.

Ft = m Av

This equation represents one of two primary principles to be used in the analysis of collisions.
To truly understand the equation, it is important to understand its meaning in words. In words, it
could be said that the force times the time equals the mass times the change in velocity. In
physics, the quantity Force x time is known as impulse. And since the quantity mv is the
momentum, the quantity mAv must be the change in momentum. The equation really says that
the

Impulse = Change in momentum

The law can be expressed this way:

® Ina collision, an object experiences a force for a specific amount of time that results in a
change in momentum. The result of the force acting for the given amount of time is that
the object’s mass either speeds up or slows down (or changes direction). The impulse
experienced by the object equals the change in momentum of the object. In equation
form,

Ft = m Av.

• Ina collision, objects experience an impulse; the impulse causes and is equal to the
change in momentum.

Example:

A force of 10,000 lbs is exerted for 3.00 sec, on a mass of 90,000 slugs.

(a) What is the impulse of the force for this 3.00 sec?
(b) What is the mass's change in momentum from this impulse?
(c) What is the mass’s change in velocity from this impulse?

a) Impulse = Ft = 10,000 x 3 = 30,000 lb s

b) Change in momentum = impulse = 30,000 lb s

c) Impulse = mAv

30,000 = (90,000)Av

Av = 0.333 ft/s
Issue 2-September 2016 2"312 Total Training Support lw
Module 2.2 Mechanics © Copyright 2016
Module 2.2 Mechanics
Worksheet

1. A gun of mass 5 kg fires a bullet of mass 20 grams. The velocity of the bullet after firing, is
750 m/sec. What is the recoil velocity of the gun?

2. An astronaut on a spacewalk has a mass of 5 slugs and is at rest relative to the space
station. She is working with a tool having a mass of 0.5 slug. She accidentally throws this
tool away from herself with a speed of 6 ft/sec. With what speed does the astronaut recoil?

3. A car having mass 1,500 kg is travelling east on a motorway at 30 m/sec. It overtakes a


truck of mass 2,000 kg also travelling east and moving with a speed of 25 m/sec. The car
rear-ends the truck. The vehicles become locked together in this collision and continue
east. What is the velocity of this combined mass?

4. Two balls of putty become one mass of putty in a collision. The first, of mass 6 kg, was
originally moving east at 10 m/sec, and the second, of mass 4 kg was originally moving
west at 9 m/sec. What is the velocity of the total mass after the collision has occurred?

5. Due to a controller’s error, two aircraft are directed to land in opposite directions on the
same runway in a fog. A Cessna 150 of mass 50 slugs and a Beechcraft Bonanza of mass
80 slugs undergo a direct head-on collision. The Beechcraft Bonanza was originally
travelling north at a speed of 30 mph and the Cessna was travelling south. The wreckage
travels a distance of 20 ft south during a time of 3.6 sec. What was the original speed of the
Cessna?

6. A 3 kg ball is moving right with a speed of 3 m/sec before a collision with a 2 kg ball
originally moving left at 2 m/sec. What are the directions and speeds of the two balls after
the collision?

7. A 2 kg bail moving right at 5 m/sec overtakes and impacts a 1 kg ball also moving right at
2 m/sec. What are the speeds and directions of the two balls after the impact?

8. A force of 30,000 N is exerted for 4.00 sec, on a 95,000 kg mass.

a) What is the impulse of the force for this 4.00 sec?

b) What is the mass’s change in momentum from this impulse?

c) What is the mass’s change in velocity from this impulse?

d) Why can’t we find the resulting change in kinetic energy of the mass?

Total Training Support Ltd 2-313


©Copyright 2016 Module 2.2 Mechanics Issue 2 - September 2016
Module 2.2 Mechanics

intentionally Blank

2-314 Total Training Support Ltd


Issue 2 - September 2016
Module 2,2 Mechanics ©Copyright 2016
Module 2.2 Mechanics
Answers

1. 3 m/sec

2. 0.6 ft/sec

3. 27 m/sec east

4. 2.4 m/sec east

5. 67.7 mph

6. The 3 kg ball is moving left at 1 m/sec and the 2 kg bail is moving right at 4 m/sec

7. The 2 kg ball is moving right at 3 m/sec and the 1 kg ball is moving right at 6 m/sec

a) 120,000 Ns

b) 120,000 Ns

c) 1.26 m/s

d) We do not know the initial velocity

Total Training Support Ltd 2-315


© Copyright 2016 Module 2.2 Mechanics Issue 2 - September 2016
Module 2.2 Mechanics
Gyroscopic principles

Rigidity
Whilst small, the rotor of a gyroscopic instrument must rotate at a very high RPM. Giving them
inertia, also called rigidity and they maintain this alignment to a fixed point in space. This
basically happens to every rotating object: wheel, propeller etc. For example: this rigidity gives
the moving bicycle its stability preventing it from failing over while riding it.

A number of factors have their influence on rigidity: the mass of the rotor, its RPM or angular
velocity and finally the distance of the mass to the axis of rotation. The larger the distance the
greater the rigidity with equal rotational speed. Again, a bike has large wheels and can rotate
slowly to obtain enough stability.

Precession
When you apply a force to a point around the spinning rim of the gyro, the rotor will tilt as if the
force was 90° further in the direction of motion as shown in the image. This apparent
displacement of the force is called precession.

The amount of precession depends on the following factors: strength and direction of the force
applied, the amount of inertia of the gyro (mass concentration on the rim), diameter and the
RPM or rotational velocity of the gyro.

To conclude: the rate of precession in a free gyro is directly proportional to strength of the force
and inversely proportional to the RPM and the moment of inertia. Thus the more mass and RPM
a gyro has the more stable it is and maintain its position to a fixed point in space.

Gyroscope
https://youtu.be/cquvAlpEsA

Gimbai rings
The gyro rotor is held in place by rings or better known as gimbal rings. These allow for freedom
of motion in three dimensional planes as required by the instruments of the aircraft.

Planes of movement
There are three possible motions for a gyroscope: the plane of rotation of the gyro; the plane of
applied force and as a result: the plane of precession. Please refer to the image opposite for
more detail and three-dimensional view.

Types of gyro
Depending on how you setup or mount the gyro in the gimbal rings it will have number of planes
the gyro can move in, each useful to the pilot in different instruments. Below you will find a list of
possible installations:

• A rate gyro can move in one plane (not the plane of rotation) and the movement in the
third plane is used to measure the precession. You will see this type of gyro in a turn
coordinator or turn and slip indicator.

• A tied gyro moves in all three planes but kept in one plane by an outside force, usually
air jets in case of the direction indicator (gyro compass)

• An earth gyro has freedom of movement in all three planes but is held in one plane by
Earth’s gravity. You will find this gyro in an attitude indicator.

2-316 Total Training Support Ltd


Issue 2 ~ September 2016
Module 2.2 Mechanics ©Copyright 2016
Module 2.2 Mechanics
• A space gyro moves in ail three planes and is stabilized to a fixed point in space. You
will apparently see this gyro move due to the Earth’s rotation while in fact it is not moving
at all, space wise.

Planes of movement of a gyroscope

Totat Training Support Ltd 2-317


©Copyright 2016 Module 2.2 Mechanics Issue 2 - September 2016
Module 2.2 Mechanics
Apparent drift (or wander)
The figure opposite illustrates the behaviour of a gyro. A perfect gyro would be one without any
external forces acting upon it, mounted in a perfect suspension system that would give it
complete freedom of movement in all three axes. All the gyros in this figure are perfect gyros.
Such gyros are called free gyros.

Only four gyros are represented - A, B, C, and D. The other gyro symbols shown illustrate the
various positions of B, C, and D as the earth rotates.

Gyro A has its spin axis parallel with the spin axis of the earth, sitting on top of the North Pole. It
could maintain that position indefinitely.

Gyro B has its spin axis parallel to the earth’s spin axis, and is located above the equator. The
other gyros in its group represent Gyro B as it would appear at different times of the day. If we
were to look at Gyro B sitting on a table in front of us, we would see that the upper end of its
spin axis is pointing off toward the north star. As time goes on and the earth turns 360°, we
would not see any change in its attitude on the table. Its spin axis would always point toward the
north star.

Gyro C is situated on the equator. The other gyros in its group represent Gyro C as it would
appear at different times of the day.

Let us say that we have the Gyro C in front of us on a table. Its spin axis is parallel to the earth’s
surface. As time goes on and the earth rotates, we would see its spin axis gradually tilting
upward at one end until, six hours later (90° of earth rotation), we would see it perpendicular to
the earth’s surface, illustrated by the gyro shown to the right of the earth. Six hours later (behind
the earth out of sight in this drawing) the spin axis would once again be parallel to the earth, but
with the end which was first pointing east now pointing west.

Another six hours later, the spin axis would once again be perpendicular, but this time the
opposite end of the axis would be another six hours later. When we get to the same time of day
at which we started, the gyro will again be occupying its original position.

Gyro D and its group illustrate another changing aspect of a gyro, in different positions as
viewed from the earth’s surface at different times of day.

These perfect gyros illustrate what any gyro tries to do but cannot because of its orientation of
the spin axis - always in the same direction in space.

Transport drift (or wander)


The outer ring of gyros in the figure opposite demonstrate that a completely free gyro in an
aircraft circling the earth would be perpendicular to the earth’s surface at only two points.

The gyros drawn in the aircraft are continuously being corrected to a vertical position as the
aircraft moves around the surface of the earth. The corrections are gentle and slow, since the
amount of correction needed in a ten-minute period, for example, is small The gyro is relatively
very stable during the pitch and roll manoeuvres of the aircraft. Such a gyro is called an earth
gyro or tied gyro.

The gyros stable position with respect to the movements of the aircraft makes it possible for the
pilot to know the actual attitude of his aircraft, nose up or down, and wings level or not. This is
quite important to him when all he can see out of the window is a grey fog.

2-318 Total Training Support Ltd


Issue 2 - September 2016
Module 2.2 Mechanics ©Copyright 2016
Module 2.2 Mechanics
The aircraft attitude information derived from the gyros is also used by such systems as the
autopilot, radar antenna stabilization, flight recorders and flight directors.

Total Training Support Ltd 2-319


© Copyright 2016 Module 2.2 Mechanics Issue 2 - September 2016
Module 2.2 Mechanics
Applications of gyroscopes in aircraft

Attitude indicators
With this instrument the pilot receives instantaneous indication about pitch and roll of the aircraft
relative to the horizon. It is shown on the face of the instrument with an aircraft image. Roll
indications are indicated at the top and pitch with the aircraft image relative to the background in
blue and brown.

Inside the instrument you will find an earth gyro spinning and it is kept in the horizontal plane
through rigidity and a pendulous unit mounted on the bottom of the gyro.

The aircraft image on the instrument is fixed (but adjustable up and down for different level flight
attitudes) and the blue/brown background is able to move up and down.

As shown on the diagram opposite, the gyro lies horizontally (vertical axis) in the inner gimbal
ring. This is known as a ‘vertical gyro’. The outer gimbal is pivoted wingtip to wingtip and moves
the background plate.

Heading indicator
The magnetic compass is the primary direction indicator in an aircraft, but it is prone to a
number of errors due to acceleration, turbulence and they are sometimes difficult to read. To
solve this problem, we use a direction indicator based on a gyro. These are stable, accurate,
easy to read and can be coupled to an autopilot and even synchronized to a magnetic compass.

The heading indicator or makes it possible to fly accurate turns and headings but has their own
distinctive features and characteristics the pilot needs to be aware of.

Inside the heading indicator you will find a tied gyro which has its rotor axis mounted
horizontally.

This is called a horizontal gyro. Thus it spins in the vertical plane. The rotor is mounted in two
gimbal rings. The outer ring is mounted vertically and rotates freely around 360° parallel with the
normal axis (vertical) of the aircraft. On this outer gimbal the index card is attached with the
headings in older models. Newer models have a vertical indicator card.

The inner gimbal ring is mounted horizontally inside the outer gimbal making sure that the inner
ring can bank as the aircraft pitches and rolls and this arrangement keeps the rotor it a vertical
position.

2-320 Total Training Support Ltd


Issue 2 - September 2016
Module 2.2 Mechanics © Copyright 2016
Module 2.2 Mechanics

Attitude indicator Heading indicator

Attitude indication and heading indication use position gyros

Total Training Support Ltd 2-321


© Copyright 2016 Module 2.2 Mechanics issue 2 - September 2016
Module 2.2 Mechanics
Turn indicator
There are two types of turn indicators in aircraft. These are; the turn and slip indicator and the
turn coordinator. Both are gyro driven and indicate the rate of turn but the turn coordinator can
also indicate rate of roll. The turn and slip indicator has a needle indicator where the turn
coordinator has an aircraft picture on the face of the instrument.

Neither of these instruments give a direct indication of bank angle. For that you will need the
speed of the aircraft and rate of turn to calculate the bank angle, or have an attitude indicator.

Turn indicators use a rate gyro (see figure opposite) to detect how fast the aircraft is changing
direction. The turn and slip indicator has a vertical needle and a slip ball; note that the Turn
Coordinator has an aircraft picture. Both indicate a two-minute turn, thus a 360° turn is
completed in two minutes or put differently: three degrees/second (3°/s). This is known as a
‘rate one’ turn.

This gyro has only one plane of freedom where it can move (not the plane of rotation). The
tilting of the vertical axis though the aircraft causes precession and this is used to indicate the
rate of turn about that axis. The amount of tilt (linear with the rate of turn) is balanced against a
restraining spring.

2-322 Total Training Support Ltd


Issue 2 ~ September 2016
Module 2.2 Mechanics © Copyright 2016
Module 2.2 Mechanics

Turn rate indication uses a rate gyro

Turn and slip indicator Turn coordinator

Totai Training Support Ltd 2-323


© Copyright 2016 Module 2.2 Mechanics issue 2 - September 2016
Module 2.2 Mechanics
Friction

Nature and effects of friction


When a body rests on a horizontal surface or is dragged or rolled on such a surface there is
always contact between the lower body surface and the horizontal surface. This contact results
in friction. Friction is work done as the surfaces rub against each other. This work heats the
surfaces and always results in wasted work.

We need to define a force known as the normal force. A body resting on a horizontal surface
experiences two forces, the downward force due to the gravitational pull of the earth on this
body (the weight of the body), and the upward push of the surface itself on the body (the normal
force).

The weight (w) and the normal force (N) are equal to each other.

There are three kinds of friction:

• Static friction
• Sliding friction
• Rolling friction

Static friction
Static friction (or ‘starting’ friction) is the force between two objects that are not moving relative
to each other. For example, static friction can prevent an object from sliding down a sloped
surface. The coefficient of static friction, typically denoted as ps, is usually higher than the
coefficient of kinetic friction. The initial force to get an object moving is often dominated by static
friction.

Another important example of static friction is the force that prevents a car wheel from slipping
as it rolls on the ground. Even though the wheel is in motion, the patch of the tire in contact with
the ground is stationary relative to the ground, so it is static rather than kinetic friction.

The maximum value of static friction, when motion is impending, is sometimes referred to as
limiting friction, although this term is not used universally

Sliding friction
Also called ‘kinetic’ or ‘dynamic’ friction, sliding friction occurs when two objects are moving
relative to each other and rub together (like a sledge on the ground).

The coefficient of sliding friction is typically denoted as psiwe and is usually less than the
coefficient of static friction.

Since friction is exerted in a direction that opposes movement, kinetic friction usually does
negative work, typically slowing something down. There are exceptions, however, if the surface
itself is under acceleration. One can see this by placing a heavy box on a rug, then pulling on
the rug quickly. In this case, the box slides backwards relative to the rug, but moves forward
relative to the floor. Thus, the kinetic friction between the box and rug accelerates the box in the
same direction that the box moves, doing positive work.

Rolling friction
Rolling friction, sometimes called rolling resistance, is the force resisting the motion when a
body (such as a ball, tire, or wheel) rolls on a surface. It is mainly caused by non-elastic effects;
that is, not all the energy needed for deformation (or movement) of the wheel, roadbed, etc. is
recovered when the pressure is removed.
2-324 Total Training Support Ltd
Issue 2 - September 2016 © Copyright 2016
Module 2.2 Mechanics
Module 2.2 Mechanics

Weight and its reaction Pushing force Motion


c

ooommooo
Roiling friction

Viscous friction Rolling friction

No
motion
Static Sliding
friction friction
Static
friction

Sliding
motion

Sliding

Roiling
motion

Rolling
friction

Friction forces

Total Training Support Ltd 2-325


©Copyright 2016 Module 2.2 Mechanics Issue 2 ~ September 2016
Module 2.2 Mechanics
Coefficient of friction
In all cases, the friction equation is the same.

F = pN

The symbol ‘p’ (the Greek letter mu) is called the coefficient of friction. Every pair of flat surfaces
has two different coefficients of friction:

The coefficient of starting (static) friction — P start

The coefficient of sliding friction — P slide

Some values for the coefficients of starting and sliding friction are given in the table.

We note that the coefficients of sliding friction are less than the coefficients of starting friction.
This means that the force needed to start a body sliding is greater than the force needed to
keep a body sliding with constant speed.

When we deal with a body that rolls over a flat surface, we have another coefficient of friction to
consider: the coefficient of rolling friction.

The coefficients of rolling friction (proll) are very small. Therefore, rolling friction is much smaller
than either starting or sliding friction. Some values are:

Rubber tires on dry concrete road 0.02

Roller bearings 0.001 to 0.003

Intuition on static and kinetic friction comparisons I Physics I Khan Academy


https://youtu.be/J9BWNiOSGIc

Example:

A steel body weighing 100 lbs is resting on a horizontal steel surface. How many pounds of force
are necessary to start the body sliding? What force is necessary to keep this body sliding at
constant speed?

W = N= 100 lbs

F = pN

Force to start sliding motion = (0.15) (100 lbs) = 15 lbs

Force to keep sliding motion = (0.09) (100 lbs) = 9 lbs

Static and kinetic friction example I Forces and Newton’s laws of motion I Physics I Khan
Academy
https://youtu.be/ZA_D4O6H Io

2-326 Total Training Support Ltd


Issue 2 - September 2016 ©Copyright 2016
Module 2.2 Mechanics
Module 2.2 Mechanics
Coefficients of friction
MaWnai'
Steel on steel 0.15 0.09
Steel on ice 0.03 .. 0.01
Leather on wood 0.5 0.4
Oak on oak 0.5 0.4
Rubber on dry concrete 1.0 0.7
Rubber on wet concrete 0.7 0,5

Ball and roller bearings have rolling friction

Amount of
friction force

Friction increases as the force pushing the surfaces together increases

Total Training Support Ltd


© Copyright 2016 Module 2.2 Mechanics Issue 2 - September 2016
Module 2.2 Mechanics

Intentionaliy Blank

Issue 2 - September 2016 2-328 Total Training Support Ltd


Module 2,2 Mechanics © Copyright 2016
Module 2.2 Mechanics
Worksheet

1. An aircraft with a weight of 85,000 lbs is towed over a concrete surface. What force must
the towing vehicle exert to keep the aircraft rolling?

2. It is necessary to slide a 200 lb refrigerator with rubber feet over a wet concrete surface.
What force is necessary to start the motion? What force is necessary to keep the motion
going?

3. A block weighing 200 N is pushed along a surface. If it takes 80 N to get the block moving
and 40 N to keep the block moving at a constant velocity, what are the coefficients of
friction pstart and psiide?

Totai Training Support Ltd 2-329


©Copyright 2016 Module 2.2 Mechanics Issue 2 - September 2016
Module 2.2 Mechanics

intentionally Blank

2-330 Total Training Support Ltd


Issue 2 - September 2016
Module 2.2 Mechanics ©Copyright 2016
Module 2.2 Mechanics
Answers

1. 1,700 lbs

2. 140 lbs 100 lbs

3. 0.4,0.2

Total Training Support Ltd 2-331


© Copyright 2016 Module 2.2 Mechanics issue 2 - September 2016
Module 2.2 Mechanics

Intentionally Blank

2-332 Total Training Support Ltd


issue 2 - September 2016
Module 2.2 Mechanics ©Copyright 2016
Module 2.2 Mechanics
2.2.4 Fluid dynamics
Specific gravity and density

Density
The density of a material is defined as the mass of a sample of the material divided by the
volume of the same sample. The symbol used for density is the Greek letter rho, (p).
rn
P= v

Other algebraic forms of this same equation are:

m = pV and V=™
P

Density is a very important and useful concept. If a body is made of a certain kind of material its
density is known. If the weight of the body is also known, it is possible to determine the volume
of this body. Similarly, if the kind of material and volume are known it is possible to determine
the weight of the body.

The table opposite is a table of densities. You can refer to this table when you solve the
problems dealing with mass, weight, and volume.

Example:

An order has been placed for 120 gallons of lubricating oil. How much will this oil weigh?
1 ft3
V = 120 gal x —~ = 16.0 ft3
7.481 gal

(The density of the lubricating oil has been obtained from the ‘Young’s Modulus and Bulk
Modulus of some common materials’ table opposite.)

M = pV
W 150 lbs
m = 4.69 slug
0 32 Ibs/slug

32 lbs
W = mg ~ (28 slugs) " 896 lbs
slug

Total Training Support Ltd 2-333


© Copyright 2016 Module 2.2 Mechanics Issue 2 - September 2016
Module 2.2 Mechanics

Liquids Kg/m3 Slug/ft3 Metals Kg/m3 Slug/ft3


WOlBS 1 °S0fiS8$8ftl#^
Sea water 1030 2 0Cast iron 7^014 0

Alcohol 789 1.53 Gold 19300 37.5


.«=.
HMISSI 800 lls Nickel 8850
Kerosene

Mercury 13600 26.3 " Steel 7800 2*15.7"”

lllilBIfiiSSiSIlSSiiOBi^^Hi^iSlililliSBI
....... „ Zinc 7140 13.9

ice (32°F, 6’C) 922 1 jq"'Baisa 130 0~25


480 0.93
Glass 2,600 "4.97 ’ Maple 640 V24

Ebony 1200 2.

Comparison of densities ~ liquids and solids

HOMfiSiOlI#
Aluminium 10x10® 10x10®
si^Mi
Copper 16x10® 17x10®

Iron 13x10® 1.45x10®


isnaiiiaioffiiMBia
Ethyl Alcohol 0.16x10®
gBlilliBBIHaaSHM
Water ~ 0.31x10®

Young’s modulus and bulk modulus of


some common materials

2-334 Total Training Support Ltd


Issue 2 - September 2016
Module 2.2 Mechanics © Copyright 2016
Module 2.2 Mechanics
Example:

An order has been placed for 150 lbs of turpentine. How many gallons of turpentine will be
delivered?
W 150 lbs
m = 4.69 slug
9 32 Ibs/slug

~ 4.69 slug __ 2 yg
p 1.69 slug/ft3
7.481 gal
V 2.78 ft3 x = 20.8 gal
1 ft

Specific gravity
The term ‘specific gravity’ is closely related to the idea of density. The definition is as follows:
Density of the substance
Specific gravity =
Density of water

The calculation will give the same result (for a given substance) no matter what units are used.
The example below will calculate the specific gravity of sulphuric acid (see the table above).

If we use the metric units (kg/m3) we obtain:

Specific gravity ~ ~ 1.83


1,000

If we use the imperial units (slug/ft3) we obtain:


_ 3.55 .
SpecficgraW —=1-83

The specific gravity number (1.83) is unit-less. It tells us that, for sulphuric acid, the density is
1.83 times that of water.

Total Training Support Ltd 2-335


© Copyright 2016 Module 2.2 Mechanics Issue 2 - September 2016
Module 2.2 Mechanics

Intentionally Blank

2-336 Total Training Support Ltd


Issue 2 - September 2016
Module 2.2 Mechanics ©Copyright 2016
Module 2.2 Mechanics
Worksheet

1. What is the specific gravity of kerosene?

2. What is the specific gravity of aluminium?

3. What is the specific gravity of ice?

4. What is the specific gravity of glass?

5. What is the weight of 85 gallons of kerosene?

Total Training Support Ltd 2-337


© Copyright 2016 Module 2.2 Mechanics Issue 2 - September 2016
Module 2.2 Mechanics

Intentionally Blank

2-338 Total Training Support Ltd


Issue 2 - September 2016
Module 2.2 Mechanics © Copyright 2016
Module 2.2 Mechanics
Answers

1. 0.8

2. 2.7

3. 0.922

4. 2.6

5. 544 lbs or 3,029 N

(Hint: calculate weight of water, 1 litre = 1kg, or 1 pint = 1 lb, then convert to kerosene by
multiplying by its specific gravity of 0.8)

Total Training Support Ltd 2-339


© Copyright 2016 Module 2.2 Mechanics Issue 2 - September 2016
Module 2.2 Mechanics
Viscosity
Viscosity is a measure of the resistance of a fluid to being deformed by either shear stress or
extensional stress. It is commonly perceived as ‘thickness’, or resistance to flow. Viscosity
describes a fluid’s internal resistance to flow and may be thought of as a measure of fluid
friction. Thus, water is ‘thin’, having a lower viscosity, while vegetable oil is ‘thick’ having a
higher viscosity. Most real fluids have some resistance to stress, but a fluid which has no
resistance to shear stress is known as an ideal fluid or inviscid fluid.

The study of viscosity is known as rheology.

Viscosity coefficients
When looking at a value for viscosity, the number that one most often sees is the coefficient of
viscosity. There are several different viscosity coefficients depending on the nature of applied
stress and nature of the fluid.

• Dynamic viscosity determines the dynamics of an incompressible fluid;


• Kinematic viscosity is the dynamic viscosity divided by the density;
• Volume viscosity determines the dynamics of a compressible fluid;
• Bulk viscosity is the same as volume viscosity

Shear viscosity and dynamic viscosity are much better known than the others. That is why they
are often referred to as simply viscosity.

Simply put, this quantity is the ratio between the pressure exerted on the surface of a fluid, in
the lateral or horizontal direction, to the change in velocity of the fluid as you move down in the
fluid (this is what is referred to as a velocity gradient).

For example, at ‘room temperature’, water has a nominal viscosity of 1.0 * 10-3 Pa-s and motor
oil has a nominal apparent viscosity of 250 * 10~3 Pa-s.

Viscosity measurement
Dynamic viscosity is measured with various types of viscometer Close temperature control of
the fluid is essential to accurate measurements, particularly in materials like lubricants, whose
viscosity can double with a change of only 5°C. For some fluids, it is a constant over a wide
range of shear rates. These are Newtonian fluids.

The fluids without a constant viscosity are called Non-Newtonian fluids. Their viscosity cannot
be described by a single number. Non-Newtonian fluids exhibit a variety of different correlations
between shear stress and shear rate.

One of the most common instruments for measuring kinematic viscosity is the glass capillary
viscometer.

In paint industries, viscosity is commonly measured with a Zahn cup, in which the efflux time is
determined and given to customers. The efflux time can also be converted to kinematic
viscosities (cSt) through the conversion equations.

Also used in paint, a Stormer viscometer uses load-based rotation in order to determine
viscosity. The viscosity is reported in Krebs units (KU), which are unique to Stormer
viscometers.

Vibrating viscometers can also be used to measure viscosity. These models use vibration rather
than rotation to measure viscosity.

2-340 Total Training Support Ltd


Issue 2 - September 2016 © Copyright 2016
Module 2.2 Mechanics
Module 2.2 Mechanics

Viscosity observed in the flow of different fluids,


e.g. water - oil - grease etc.

Total Training Support Ltd 2-341


©Copyright 2016 Module 2.2 Mechanics Issue 2 - September 2016
Module 2.2 Mechanics

Intentionally Blank

2-342 Total Training Support Ltd


Issue 2 - September 2016
Module 2.2 Mechanics © Copyright 2016
Module 2.2 Mechanics
Viscosity (dynamic/absolute viscosity)
Dynamic viscosity and absolute viscosity are synonymous. The symbol for viscosity is the Greek
symbol eta (r|), and dynamic viscosity is also commonly referred to using the Greek symbol
mu (p). The SI physical unit of dynamic viscosity is the pascal-second (Pa-s), which is identical
to 1 kg/(ms) (kilogram per metre-second). If a fluid with a viscosity of one Pa-s is placed
between two plates, and one plate is pushed sideways with a shear stress of one pascal, it
moves a distance equal to the thickness of the layer between the plates in one second.

Kinematic viscosity
In many situations, we are concerned with the ratio of the viscous force to the inertial force, the
latter characterised by the fluid density p. This ratio is characterised by the kinematic
viscosity (v), defined as follows:

P
where p is the (dynamic) viscosity, and p is the density.

Kinematic viscosity (Greek symbol: v) has SI units (m2/s). The CGS physical unit for kinematic
viscosity is the stokes (abbreviated S or St), named after George Gabriel Stokes. It is
sometimes expressed in terms of centistokes (cS or cSt). In US usage, stoke is sometimes used
as the singular form.

1 stokes = 100 centistokes = 1 cm2/s = 0.0001 m2/s.


1 centistokes = 1 mm2/s

Viscosity of air
The viscosity of air depends mostly on the temperature. At 15.0°C, the viscosity of air is
1.78 x 10-5 kg/(ms).

Viscosity of water
The viscosity of water is 8.90 x 10“4 Pa-s at about 25°C.

Total Training Support Ltd 2-343


© Copyright 2016 Module 2.2 Mechanics Issue 2 - September 2016
Module 2.2 Mechanics

Fluid resistance and streamlining

Drag
In fluid dynamics, fluid resistance (usually called ‘drag’) is the force that resists the movement of
a solid object through a fluid (a liquid or gas). Drag is made up of friction forces, which act in a
direction parallel to the object’s surface (primarily along its sides, as friction forces at the front
and back cancel themselves out), plus pressure forces, which act in a direction perpendicular to
the object’s surface. For a solid object moving through a fluid or gas, the drag is the sum of all
the aerodynamic or hydrodynamic forces in the direction of the external fluid flow. (Forces
perpendicular to this direction are considered lift). It therefore acts to oppose the motion of the
object, and in a powered vehicle it is overcome by thrust.

Types of drag are generally divided into three categories: profile drag (called ‘parasitic’ drag in
USA), lift-induced drag (also known as vortex drag or induced drag), and wave drag. Profile
drag includes form drag (drag due to shape), skin friction (drag due to surface roughness),
and interference drag (drag due to intermingling of airflows at changes in section of airframe).
Lift-induced drag is only relevant when wings or a lifting body are present, and is therefore
usually discussed only in the aviation perspective of drag.

Wave drag occurs when a solid object is moving through a fluid at or near the speed of sound
in that fluid.

The overall drag of an object is characterized by a dimensionless number called the drag
coefficient, and is calculated using the drag equation. Assuming a constant drag coefficient,
drag will vary as the square of velocity. Thus, the resultant power needed to overcome this drag
will vary as the cube of velocity. The standard equation for drag is one half the coefficient of
drag multiplied by the fluid density, the cross sectional area of the specified item, and the
square of the velocity.

Air flowing over a skier causes


form drag and skin friction drag

*
2-344 Total Training Support Ltd
Issue 2 - September 2016
Module 2.2 Mechanics ©Copyright 2016
Module 2.2 Mechanics
'
Total drag is made up of the drag from ----
----
different parts of the aeroplane
----

<—
Q ----
& ----
Fuselage drag o Ps
= 2,000 N

Wing drag
= 3,000 N

Total drag
= 3,500 N

Interference drag
-SOON Effect of airflow on a flat plate

Some examples of Cd

--- .
A smooth brick

A rough sphere 0.4

A flat plate parallel to the flow 0.001

A man (upright position) 1.0-1.3

A skier 1.0-1.1

Drag coeffici entsi of jN:T


Aircraft type ■■■ T;'N“ ■■
Cessna 172/182 0.027
-Ce^srt^3|fe:;>r^>:;a;
Learjet 24 0 022
Bqeinfl;O®gW
X-15 0.95

Total Training Support Ltd 2-345


©Copyright 2016 Module 2.2 Mechanics Issue 2 - September 2016
Module 2.2 Mechanics
Drag coefficient
The drag coefficient (Cd) is a dimensioniess quantity that describes a characteristic amount of
aerodynamic drag caused by fluid flow, used in the drag equation. Two objects of the same
frontal area moving at the same speed through a fluid will experience a drag force proportional
to their Cd numbers. Coefficients for rough unstreamlined objects can be 1 or more, for smooth
objects much less.

Aerodynamic drag = Cd 1/z p V2 A

Where

Cd = drag coefficient (dimensionless)


p = fluid density (slug/ft3 or kg/m3)
V = Velocity of object (ft/s or m/s)
A = projected frontal area (ft2 or m2)

The drag equation is essentially a statement that, under certain conditions, the drag force on
any object is approximately proportional to the square of its velocity through the fluid,

A Cd equal to 1 would be obtained in a case where all of the fluid approaching the object is
brought to rest, building up stagnation pressure over the whole front surface. The figure
opposite (top) shows a flat plate with the fluid coming from the right and stopping at the plate.

Effects of streamlining
Streamlining is the shaping of an object, such as an aircraft body or wing, to reduce the amount
of drag or resistance to motion through a stream of air. A curved shape allows air to flow
smoothly around it. A flat shape fights air flow and causes more drag or resistance. Streamlining
reduces the amount of resistance and increases lift.

To produce less resistance, the front of the object should be well rounded and the body should
gradually curve back from the midsection to a tapered rear section.

The figure opposite shows how the drag of a flat plate can be reduced if its shape is changed to
a sphere, and more still if it is streamlined with fairings.

2-346 Total Training Support Ltd


Issue 2 - September 2016 © Copyright 2016
Module 2.2 Mechanics
Module 2.2 Mechanics

Learjet 24 0.022

0.024

Airbus A380 0.0265

0.027

Cessna 310 0,027


Sphere with
Boeing 747 0.031
a fairing

Drag coefficients of various


aeroplanes
Air-flows around a flat plate, a sphere, and a
faired-sphere, and their relative drag forces

Cd = 0.5 CD = 0.05

Drag coefficients of various shapes

Drag = Cp Vfe p V2 S

A wheel spat is a form of fairing, used to


streamline a wheel and reduce drag

Total Training Support Ltd 2-347


©Copyright 2016 Module 2.2 Mechanics Issue 2 - September 2016
Module 2.2 Mechanics
Effects of compressibility on fluids
Fluids are defined as any substance which flows readily. Gases and liquids are such
substances.

Gases
A gas is relatively easy to compress, and the effects of which have already been discussed in
the section on pressure and force.

Liquids
Many people think that a liquid is incompressible. However, liquids are, like any material, to a
certain amount compressible. In calculations, the amount of compressibility of liquid is
considered to be 1 volume-% per 100 bar. This means that for example when there is liquid
supplied to a 200 litre oil drum which already is completely filled with liquid, the pressure
increases with 100 bar for each 2 litre of extra supplied liquid. When we supply 3 litre of extra oil
the pressure increases with 150 bar. The compressibility of liquid plays a key role in for example
fast hydraulic systems like servo-systems of a flight simulator. To obtain a maximum dynamic
performance, the compressibility should be as little as possible. This is achieved by mounting
the control valves directly on the hydraulic motor or cylinder. In that case the amount of liquid
between the control valve and the motor/cylinder is minimised.

In some situations, the compressibility of liquids is made use of in design. A ‘liquid spring’ for
example, is the principle of a particular type of landing gear leg, which uses no gas. The leg is
completely filled with oil. The oil is compressed under the extremely large forces encountered
on landing, and the shock of the landing is absorbed by the compressibility of the liquid.

2-348 Total Training Support Ltd


Issue 2 - September 2016 © Copyright 2016
Module 2.2 Mechanics
Gas Liquid
Gases compress easily under a force
while liquids are difficult (but not impossible) to compress

Compressibility of liquids can cause problems in hydraulics that are required to


move quickly under very high load, such as this flight simulator platform

Total Training Support Ltd 2-349


©Copyright 2016 Module 2.2 Mechanics Issue 2 - September 2016
Module 2.2 Mechanics
Static, dynamic and total pressure

Static pressure
Static pressure is felt when the fluid is at rest or when the measurement is taken when traveling
along with the fluid flow, it is the force exerted on a fluid particle from ail directions, and is
typically measured with gauges and transmitters attached to the side of a pipe or tank wall.
Since static pressure is what most pressure gauges measure, static pressure is usually what is
implied when the term “pressure” is used in discussions.

Dynamic pressure
Dynamic pressure (sometimes called ‘stagnation’ pressure) is the pressure of a fluid that results
from its motion.

It is the difference between the total and static pressure, and represents the kinetic energy of
the flowing fluid. Dynamic pressure is a function of the fluid velocity and its density and can be
calculated from:

Pdynamic ~ p -----
2g
Pressure is the continuous physical force exerted on or against an object by something (a fluid
such as air) in contact with it.

Pilots rely on instruments that measure dynamic pressure to determine their airspeed.

Total pressure
The total pressure (also called ‘ram pressure’ or ‘Pitot pressure’) is the force per unit area that is
felt when a flowing fluid is brought to rest and is usually measured with a pitot tube type
instrument, shown opposite. The total pressure is the sum of the static pressure and the
dynamic pressure.

Ptotal = Pstatic *** Pdynamic

Total pressure is often referred to as the stagnation pressure.

Static pressure, stagnation pressure, and total pressure


https://youtu.be/pmcnvDIYmOg

Static vs. dynamic pressure


https://youtu.be/YqHOpSowDbw

2-350 Total Training Support Ltd


Issue 2 - September 2016
Module 22 Mechanics © Copyright 2016
Module 2.2 Mechanics

Flow
rate

Manometers measure pressure

a) Measures static pressure

b) Measures total pressure

Total Training Support Ltd 2-351


© Copyright 2016 Module 2.2 Mechanics Issue 2 - September 2016
Module 2.2 Mechanics
Bernoulli’s theorem and the venturi

Basic definitions
Before we begin our discussion of the lift and drag on an aircraft wing, the following definitions
must be understood.

The Pitot tube (named after Henri Pitot in 1732) measures a fluid velocity by converting the
kinetic energy of the flow into potential energy. The conversion takes place at the stagnation
point, located at the Pitot tube entrance (figure opposite). A pressure higher than the free­
stream (i.e. dynamic pressure) results from this conversion.

This static pressure is measured at the static taps (known as static ports or vents). The static
pressure is not affected by the speed of the aircraft, but is dependent upon the surrounding
atmospheric static pressure.

Pitot pressure is the sum of static and dynamic pressures, thus:

Ptotal ~ Pstatic *** Pdynamic

Bernoulli’s principle applies the ideas of work and energy and the conservation of energy to a
mass of fluid (liquid or gas). Since it is not as easy to think of a mass of fluid as it is to think of a
discrete body, the derivation of this principle requires some thought and effort.

It is worth the thought and effort, however, since this principle is the basic principle of the flight
of heavier-than-air aircraft.

We review that the density of a fluid (p) is related to the mass and volume of the sample of fluid
by the relation:

m = pV

The fluid flows from a region where the cross-sectional area is less (labelled with 1 ’s in the
diagram) to a region where the cross-sectional area is greater (labelled with 2’s in the diagram).
We assume that the volume of fluid in the left cylindrical shape of fluid (labelled with 1 ’s) is
equal to the volume of fluid in the right cylindrical shape of fluid (labelled with 2’s).

Hence, the volume flow rate in any part of the tube is constant, regardless of the tube diameter.
And, since the density of the fluid is constant (and air flowing at subsonic speed is considered
incompressible) than the mass flow rate is also constant, regardless of the diameter of the tube.

Bernoulli’s equation (part 1) I Fluids I Physics [ Khan Academy


https://youtu.be/uqyLOuAzbvo

2-352 Total Training Support Ltd


Issue 2 - September 2016 ©Copyright 2016
Module 2.2 Mechanics
Module 2.2 Mechanics

Higher
pressure

2 1 2

Fluid flowing through a tube of increasing cross sectional area

Total Training Support Ltd 2-353


©Copyright 2016 Module 2.2 Mechanics Issue 2 - September 2016
Module 2.2 Mechanics
Venturi
A venturi tube is a tube constructed in such a way that the cross sectional area of the tube
changes from a larger area to a smaller area and finally back to the same larger area. As a fluid
flows through this tube the velocity of the fluid changes from a lower velocity to a higher velocity
and finally back to the same lower velocity. We note that, if the rate (volume per second) of fluid
flow is to remain constant, the fluid must flow faster when it is flowing through the smaller area.

A diagram of a venturi tube is shown in the figure opposite.

The height of the fluid column in the vertical tubes at the three places shown in the diagram, is
an indication of the fluid pressure. As we expect from Bernoulli’s principle, the pressure is
greater where the velocity is lower and vice versa.

Venturi tubes in different shapes and sizes are often used in aircraft systems.

If we consider the types of energy involved in the flowing fluid, we find that there are three types
- potential (gravitational), pressure and kinetic energies.

Now if we consider only two positions in the venturi - the wide part (marked *1 ’) and the narrow
part (the throat, marked ‘2’), and consider the conservation of energy principle, we have:

Potential energy at 1 Potential energy at 2


+ +
Pressure energy at 1 = Pressure energy at 2

Kinetic energy at 1 Kinetic energy at 2

The above is assumed since the total energy in the fluid cannot change, only transferred from
one form to another. This then, is the basis for Bernoulli’s formula.

Venturi effect and Pitot tubes I Fluids I Physics I Khan Academy


https://youtu.be/Qz1g6kqvllG8

2-364 Total Training Support Ltd


Issue 2 - September 2016 © Copyright 2016
Module 2.2 Mechanics
Module 2,2 Mechanics

2 1 2
A venturi tube

Potential energy at 1 Potential energy at 2

Pressure energy at 1 - Pressure energy at 2


+ +
Kinetic energy at 1 Kinetic energy at 2

Total Training Support Ltd 2-355


©Copyright 2016 Module 2.2 Mechanics Issue 2 - September 2016
Module 2.2 Mechanics
Since the venturi in this case is horizontal, there is no change in potential energy, and so the
potential energies can be cancelled from the formula thus:

Pressure energy at 1 Pressure energy at 2


4* = +
Kinetic energy at 1 Kinetic energy at 2
Since kinetic energy _1_ is
2
where

m = mass of fluid, and V = velocity of fluid

and
p
Pressure energy is

where

P = pressure, and p = density of the fluid

Thus:
Pi 1 p2 1
m — + mV? k m — + “-mV22
p 2 p 2
Note that the mass, m, has no suffix, since the mass flow rate is constant regardless of the area
of the flow. The density, p, is also a constant since the fluid is considered incompressible (even
air, providing its velocity is subsonic).

Cancelling the mass, m, from each equation, and multiplying each term by the density, p, gives
us

Pi + % p Vi2 = P2 + % p V22

This is the standard mathematical form of the Bernoulli’s equation. It can be rearranged to give
the pressure difference (for example between the upper and lower surfaces of a wing) thus:

Pi - P2 = % p V22 - % p V12

Factorising gives:

P1-P2 = 1/2p(V22~Vi2)

2-356 Total Training Support Ltd


Issue 2 - September 2016 ©Copyright 2016
Module 2.2 Mechanics
Module 2.2 Mechanics

Pressure energy at 1 Pressure energy at 2

Kinetic energy at 1 Kinetic energy at 2

Total Training Support Ltd 2-357


©Copyright 2016 Module 2.2 Mechanics issue 2 - September 2016
Module 2.2 Mechanics
Application of Bernoulli’s principle to aerofoil sections
The relative wind direction is the direction of the airflow with respect to the wing and is
opposite to the path of flight (Figure opposite).

The chord line of a wing is a straight line connecting the leading edge of a wing to its trailing
edge (Figure opposite).

Angle of attack is the angle between the chord line of a wing and the relative wind direction
(Figure opposite).

The figure opposite shows the cross section of a wing at rest and subject to atmospheric
pressure which on the average is 14.7 lb/in2.

A force of 14.7 lbs can be imagined as acting perpendicular to every square inch of the wing.
The resultant of these 14.7 lbs force vectors is zero and therefore has no effect on the dynamics
of the aeroplane.

It is the motion of air past the wing that alters the pressure pattern. Whether the wing is in
motion through the air or the air is flowing past a stationary wing the result is the same.

For example, if an aeroplane is moving through stationary air at a speed of 200 mph, the effect
is the same (as far as the aeroplane and air are concerned) as if the aeroplane were stationary
and the air was moving with velocity 200 mph past the aeroplane.

There is a thin layer of air in direct contact with the wing surface, which, due to skin friction, is
actually stationary (relative to the wing). This is called the boundary layer. In these discussions
we will disregard the boundary layer and assume that the airflow is unaffected by friction.

As air streams past the wing of an aeroplane, the speed of the air past the upper surface of the
wing is greater than the speed of the air past the lower surface of the wing.

These exact speeds are determined by the shape of the wing and the angle of attack.

2-358 Total Training Support Ltd


Issue 2 - September 2016 © Copyright 2016
Module 2.2 Mechanics
Module 2.2 Mechanics

Chord Sine

Total Training Support Ltd 2-359


© Copyright 2016 Module 2.2 Mechanics Issue 2 - September 2016
Module 2.2 Mechanics
For example, if the speed of the relative wind (equal to the speed of the aeroplane) is 200 mph,
the speed of the air past the upper surface of the wing may be 210 mph and the speed of air
past the lower surface of the wing may be 195 mph. As indicated above, the exact values for a
given case depend on the shape of the wing and the angle of attack.

In this example, we could say that the speed past the upper surface of the wing is
[1.05 x (200 mph)] and the speed past the lower surface if the wing is [0.975 x (200 mph)].

In the figure, the following symbols apply:

P1 = pressure on the upper surface of the wing


P2 = pressure on the lower surface of the wing
Vo = relative wind velocity
V1 = wind velocity over upper surface
V2 = wind velocity over lower surface
p = density of the air

We apply Bernoulli’s principle

Pi + 1/2 p Vi2 = P2 + 1/2 p V22

We note that the ones refer to the upper surface and the twos apply to the lower surface of the
wing.

Pi + % p Vi2 = P2 + % p V22

% p Vi2 - y2 p v22 =p2-Pi

% p (Vi2 - V22) =P2-P1

When finding the lift on a wing, the pressure difference between the upper and lower surfaces is
found from the above equation, and, since Force = Pressure x Area, simply multiply the
calculated pressure difference by the area of the wing, thus:

AP = 1/2 p (V22 - Vi2), and Lift (or Force) = AP x Area

Lift = AP x Area

Note: In some questions, the weight of the aircraft will be quoted. Thus, if the aircraft is flying
straight and level, the Lift = Weight.

2-360 Total Training Support Ltd


Issue 2 ~ September 2016 ©Copyright 2016
Module 2.2 Mechanics
Module 2.2 Mechanics

Total Training Support Ltd 2-361


©Copyright 2016 Module 2.2 Mechanics Issue 2 - September 2016
Module 2.2 Mechanics

Intentionally Blank

2-362 Total Training Support Ltd


Issue 2 - September 2016
Module 2.2 Mechanics ©Copyright 2016
Module 2.2 Mechanics
Worksheet

1. An aeroplane having wing area 500 ft2 is moving at 300 ft/sec. The speed of the air moving
past the iop surface of the wing is 400 ft/sec and the speed of the air past the bottom
surface of the wing is 200 ft/sec. The density of the air is 0.0025 slug/ft3. What is the lift?

2. An aeroplane having wing area 400 ft2 is cruising at 230 ft/sec. The speed of the air moving
past the top surface of the wing is 240 ft/sec and the speed of the air past the bottom
surface of the wing is 230 ft/sec. The density of the air is 0.0025 slug/ft3. What is the weight
of the aeroplane?

3. An aeroplane is cruising at 310 ft/sec. The speed of the air moving past the top surface of
the wing is 340 ft/sec and the speed of the air past the bottom surface of the wing is
300 ft/sec. The density of the air is 0.001 slug/ft3. The weight of the aeroplane is 12,800 lbs.
What is the wing area?

Total Training Support Ltd 2-363


© Copyright 2016 Module 2.2 Mechanics Issue 2 - September 2016
Module 2.2 Mechanics

Intentionally Blank

2-364 Total Training Support Ltd


Issue 2 ~ September 2016
Module 2.2 Mechanics ©Copyright 2016
Module 2.2 Mechanics
Answers

1. 75,000 lbs

2. 2,350 lbs

3. 1,000 ft

Total Training Support Ltd 2-365


©Copyright 2016 Module 2.2 Mechanics Issue 2 - September 2016
Module 2.2 Mechanics

intentionally Blank

2-366 Total Training Support Ltd


Issue 2 - September 2016
Module 2,2 Mechanics ©Copyright 2016

You might also like